0% found this document useful (0 votes)
117 views

Pediatrics in Review 04-2016

Uploaded by

thiago
Copyright
© © All Rights Reserved
We take content rights seriously. If you suspect this is your content, claim it here.
Available Formats
Download as PDF, TXT or read online on Scribd
0% found this document useful (0 votes)
117 views

Pediatrics in Review 04-2016

Uploaded by

thiago
Copyright
© © All Rights Reserved
We take content rights seriously. If you suspect this is your content, claim it here.
Available Formats
Download as PDF, TXT or read online on Scribd
You are on page 1/ 65

Over 70%

of children are
admitted to
general APRIL 2016

hospitals
Vol. 37 No. 4
www.pedsinreview.org

rather than
Hematopoietic Stem Cell
children’s Transplantation in
Children and Adolescents
hospitals. Guilcher

Does your pediatric patient care fit? Physical Abuse of Children


Glick, Lorand, Bilka

Syncope
Cannon, Wackel
AN OFFICIAL JOURNAL OF THE AMERICAN ACADEMY OF PEDIATRICS
JANUARY 2016 • VOLUME 6 • NUMBER 1
w w w.hospi t alpedia trics.or g

Toward High-Value Care:


COMMENTARY
A Quality Improvement
Initiative to Reduce
AN OFFICIAL JOURNAL OF THE AMERICAN ACADEMY OF PEDIATRICS
N O V E M B E R 2 015 • V O L U M E 5 • N U M B E R 11
w w w.hospi t alpedia trics.or g
Unnecessary Repeat
Complete Blood Counts
and Basic Metabolic Panels
on a Pediatric Hospitalist
Service
JOHNSON ET AL
Zika Virus – Pediatricians
Parent and Provider
Perspectives on Pediatric
Readmissions: What Can
We Learn About Readiness
Reducing Overutilization
of Testing for Clostridium
difficile Infection in a
Pediatric Hospital System:
A Quality Improvement
Initiative
Be Aware
for Discharge?

Rathore
BRITTAN ET AL KLATTE ET AL

An Examination of Reasons for Refusal


Physician-, Caregiver-, and of Newborn Vitamin K
Disease-Related Factors Prophylaxis: Implications for
Associated With Readmission Management and Education
From a Pediatric Hospital HAMRICK ET AL
Medicine Service
WALLACE ET AL

The Med-Peds Hospitalist


Workforce: Results From
the American Academy of
Pediatrics Workforce Survey
DONNELLY ET AL

Subscribe to Hospital Pediatrics®, the


Monthly Peer-Reviewed Journal from
the American Academy of Pediatrics
Choose a 1- or 2-year individual
subscription terms in Online Only
OR Get a site license for your organization.
Order directly through the AAP.
now indexed
in MEDLINE or Print and Online formats. PHONE 847/434-4000
shop.aap.org/Hospital-Pediatrics EMAIL institutions@aap.org
ONLINE QUOTE go.aap.org/sitelicenseinquiryform
ASAP!
Every article. Any device.
Anytime. Anywhere.

Want A Better

Wrangle
Way to

Thousands of
Pediatrics-Focused Studies? AAP GRAND ROUNDS
www.aapgrandrounds.org
Mission: To provide pediatricians with timely synopses and critiques of important new studies relevant to
pediatric practice, reviewing methodology, significance, and practical impact, as part of ongoing CME activity.

PEDIATRIC PULMONOLOGY & SLEEP MEDICINE


Vol. 30 No. 5 | Pages 49-60 | November 2013

Desktop. Tablet. Mobile.

Adenotonsillectomy for Obstructive Sleep Apnea


It’s powerful and it’s here, now! It’s a smart new benefit, free
Source: Marcus CL, Moore RH, Rosen CL, et al. A randomized behavior, and quality of life among school-aged children with OSA. Watchful
trial of adenotonsillectomy for childhood sleep apnea. N Engl J observation may be an alternative for childhood OSA given the absence of
Med. 2013;368(25):2366-2376; doi:10.1056/NEJMoa1215881 significant cognitive improvement and the fact that polysomnography normal-
ized in about half of the children in the watchful observation group.

I
nvestigators from multiple in-
stitutions conducted a random- PICO Commentary by
ized controlled trial to assess Question: Among children aged 5 to 9 Catherine Kier, MD, FAAP, Pediatrics, Stony Brook School of
the efficacy of adenotosillectomy in years with obstructive sleep apnea, does Medicine, Stony Brook, NY

A single, integrated network of journals and periodicals — AAP News,


adenotonsillectomy, compared to watchful Dr Kier has disclosed no financial relationship relevant to this commentary. This commentary does not contain a
children with obstructive sleep ap-

to all AAP members. Log on


observation, result in improved outcomes? discussion of an unapproved/investigative use of a commercial product/device.
nea (OSA) as part of the Childhood
Question type: Intervention Research on pediatric OSA has exponentially increased in the past decade.
Adenotonsillectomy Trial (CHAT).
Study design: Randomized controlled A recent systematic review of the epidemiology of pediatric OSA indicated a
Children 5 to 9 years old with a
score ≥2 on an obstructive apnea- prevalence of 4% to 11% based on parent questionnaire and 1% to 4% based
on diagnostic studies, with higher prevalence among boys, obese children,

Nearly 100 medical journals scanned


hypopnea index (AHI) or ≥1 on the obstructive apnea index were enrolled.
and black children.1 Neurocognitive, cardiovascular, metabolic, and quality

Pediatrics, Pediatrics in Review, Hospital Pediatrics, NeoReviews and


Study participants were randomized to early adenotonsillectomy (within
1 month after randomization) or to watchful observation. The primary of life consequences have been associated with pediatric OSA, especially if
outcome measure was change in attention and executive function based left untreated. Adenotonsillectomy is considered the first line of treatment.
on the Developmental Neuropsychological Assessment score. Secondary About half a million tonsillectomies in children are performed in the United

using your AAP ID and visit


outcome measures included polysomnographic, behavioral, symptomatic, States annually, many of these for OSA.2 The CHAT is the first multicenter
and quality of life measures collected at the beginning of the study and at randomized trial comparing adenotonsillectomy and watchful observation
7 months. Outcomes were compared between the 2 groups; outcomes were for pediatric OSA. CHAT has replicated the findings of a meta-analysis of 14

AAP Grand Rounds — all available in one intuitive interface that


also compared in several “high-risk” subgroups. case series, which showed normalization of polysomnography in about 80%

monthly by 49 pediatric subspecialty A total of 464 children were enrolled; outcome data were collected on 397
including 194 participants randomized to early adenotonsillectomy. There
was no significant difference between the 2 study groups with regard to
change in attention or executive function scores at 7 months post interven-
tion. However, the adenotonsillectomy group demonstrated significantly
of children following adenotonsillectomy.3
The multicenter CHAT study was well-designed, utilized standardized
measurements, and closely followed participants. Since younger children
and those with very severe OSA, prolonged desaturations, and ADHD on
medication were excluded, the findings cannot be generalized to these
groups. In addition, since reevaluation occurred at 7 months, it is possible

gateway.aap.org.
greater improvement in behavioral, quality of life, polysomnographic, and

works on all of your devices.


symptomotology measures than the watchful observation group. Polysom- that improvements in attention and executive function may not be evident
nography normalized in 79% of the adenotonsillectomy group compared to in that short time.4

experts resulting in 134 subject collections 46% of the watchful waiting group. While the CHAT study certainly provides promising evidence that
Analyses in higher risk subgroups including black children, obese children, adenotonsillectomy improves behavioral, symptom, quality of life, and
and children with AHI scores above the median showed that polysomnogra- polysomnographic measures in at least some children with OSA, it leaves
phy did not normalize as frequently many questions unanswered. Future studies will need to assess the value of
compared to those in the correspond- adenotonsillectomy in younger children and those with more severe OSA as
ing lower risk group. This was true in well as evaluating the impact over a longer follow-up period.
INSIDE both adenotonsillectomy and watch- References
Cranial Autonomic Symptoms in ful observation groups. However, the 1. Lumeng JC, et al. Proc Am Thorac Soc. 2008;5(2):242-252; doi:10.1513/pats.200708-

and an archives vault with over 2,000 articles.


Pediatric Migraine adenotonsillectomy group improved 135MG
Long-term Benefits of Selective Dorsal more with regard to polysomnogra- 2. Baugh RF, et al. Otolaryngol Head Neck Surg. 2011;144(1 Suppl):S1-S30;
Rhizotomy doi:10.1177/0194599810389949
phy, behavior, and clinical measures as
Juvenile Idiopathic Arthritis and Live 3. Brietzke SE, et al. Otolaryngol Head Neck Surg. 2006;134(6):979-984; doi:10.1016/j.
Virus Vaccines
compared to the watchful observation otohns.2006.02.033

It’s tailored to your interests.


Cognitive and Behavior Outcome 8
group in these higher risk subgroups. 4. Brouillette RT. N Engl J Med. 2013;368(25):2428-2429; doi:10.1056/NEJMe1305492
Years Following ECMO in the NICU The authors conclude that although
Key words: obstructive sleep apnea, adenotonsillectomy, watchful observation
Disparity in Receipt of Educational adenotonsillectomy did not result in
Services for Autism significantly greater cognitive improve-
Weighing the Evidence: Logistic ments compared to watchful observa- Visit www.GrandRoundsBlog.org
Regression
tion, it resulted in significant improve- to read a post about this article
Is School an Infection Risk for Children appearing this month.
With Cancer? ments in polysomnography, symptoms,
The Costs of Child Maltreatment

AAP Gateway will help you manage AAP journals and periodicals,
What Is Different About Children Who
The AAP Policy on Disclosure of Financial Relationships and Resolution of Conflicts of Interest for AAP CME Activities is designed to issue of AAP Grand Rounds unless noted on the article or below. The AAP has taken steps to resolve any potential conflicts of interest.
Receive Palliative Care? ensure quality, objective, balanced, and scientifically rigorous AAP CME activities by identifying and resolving all potential conflicts of
Joseph Geskey, DO (Editorial Board Member) disclosed a Speaker’s Bureau with GlaxoSmithKline.
interest prior to the confirmation of service of those in a position to influence and/or control CME content.
Lane Palmer, MD (Editorial Board Member) disclosed a Speaker’s Bureau with Laborie.
Back Page: New Guidelines for All individuals in a position to influence and/or control the content of AAP CME activities, including editorial board members, authors, Philip Rosenthal, MD (Editorial Board Member) disclosed Research Grants with Bristol-Myers Squibb and Roche.
Diagnosis and Treatment of C difficile and staff, are required to disclose to the AAP and subsequently to learners that the individual either has no relevant financial Robert R. Wittler, MD (Editorial Board Member) disclosed a Speaker’s Bureau with Sanofi Vaccines.
relationship or any financial relationships with the manufacturer(s) of any commercial product(s) and/or provider(s) of commercial
services discussed in CME activities. None of the editors, authors, or staff had any relevant financial relationships to disclose for this

news, updates and clinical research that’s most relevant and


Subscribe to 2016 AAP Grand Rounds, important to you, based on your specialty and interests.
the Monthly Pediatric Literature Review
from the American Academy of Pediatrics It’s simple, intuitive and accessible.
Take full advantage of AAP Gateway and easily find, filter and sort
Choose an individual
subscription in Print and
OR Get a site license for your organization.
Order directly through the AAP.
over 50,000 articles with the all new search platform — now
Online or Online Only formats. PHONE 847/434-4000
including PubMed.
shop.aap.org/2016-AAP- EMAIL institutions@aap.org
Grand-Rounds ONLINE QUOTE go.aap.org/sitelicenseinquiryform
contents

Pediatrics in Review ®
Vol. 37 No. 4 April 2016

COMMENTARY Editor-in-Chief: Joseph A. Zenel, Sioux Falls, SD


Deputy Editor: Hugh D. Allen, Houston, TX
133 Zika Virus – Pediatricians Be Aware Associate Editor: Philip R. Fischer, Rochester, MN
Mobeen H. Rathore Associate Editor, Index of Suspicion: Deepak M. Kamat,
Detroit, MI
ARTICLES Associate Editor, In Brief: Henry M. Adam, Bronx, NY
Associate Editor, In Brief: Janet Serwint, Baltimore, MD
135 Hematopoietic Stem Cell Transplantation Associate Editor, CME: Paula Algranati, Longmeadow, MA
Editorial Fellow: Mark F. Weems, Memphis, TN
in Children and Adolescents Editor Emeritus: Lawrence F. Nazarian, Rochester, NY
Gregory M.T. Guilcher Founding Editor: Robert J. Haggerty, Canandaigua, NY
Managing Editor: Luann Zanzola
146 Physical Abuse of Children Editorial Associate: Sara Strand
Jill C. Glick, Michele A. Lorand, Kristen R. Bilka Medical Copyediting: Deborah K. Kuhlman

158 Addendum for Meningitis EDITORIAL BOARD


Robert D. Baker, Buffalo, NY
159 Syncope Peter F. Belamarich, Bronx, NY
Theresa Auld Bingemann, Rochester, NY
Bryan Cannon, Philip Wackel Stephen E. Dolgin, New Hyde Park, NY
Lynn Garfunkel, Rochester, NY
INDEX OF SUSPICION
Rani Gereige, Miami, FL
169 Case 1: Diplopia in a 15-year-old Boy Joseph Gigante, Nashville, TN
Nupur Gupta, Boston, MA
Christine Puthawala, Shana Hansen
Gregory A. Hale, St. Petersburg, FL
172 Case 2: Acute Onset of Lower Extremity Thomas C. Havranek, Bronx, NY
Jacob Hen, Bridgeport, CT
Weakness in a 16-year-old Korean Boy Jeffrey D. Hord, Akron, OH
Sandy Aikara, Srividya Naganathan, Santhosh Eapen Neal S. LeLeiko, Providence, RI
Michael Macknin, Cleveland, OH
175 Case 3: Altered Mental Status in a 14-Year-Old Girl Susan Massengill, Charlotte, NC
Jennifer L. Miller, Gainesville, FL
Allison Lowe Guimera, Deepa Kulkarni Carrie A. Phillipi, Portland, OR
Peter Pizzutillo, Philadelphia, PA
IN BRIEFS Mobeen Rathore, Jacksonville, FL
177 Chagas Disease Jennifer S. Read, Rockville, MD
E. Steve Roach, Columbus, OH
Aaron W. Tustin, Natalie M. Bowman Sarah E. Shea, Halifax, Nova Scotia
Andrew Sirotnak, Denver, CO
179 Dengue and Chikungunya Miriam Weinstein, Toronto, ON
Paul J. Lee, Leonard R. Krilov
PUBLISHER: American Academy of Pediatrics
ONLINE Mark Grimes, Director, Department of Publishing
Joseph Puskarz, Director, Division of Journal Publishing
e10 Complementary, Integrative, and Holistic
Medicine: Integrative Approaches to Pediatric Pediatrics in Review offers 36 CME articles per year. A maximum
of one AMA PRA Category 1 Credit TM is earned after achieving a
Irritable Bowel Syndrome 60% score on each designated quiz.
Alycia Leiby, Minal Vazirani CME STATEMENTS:
The American Academy of Pediatrics (AAP) is accredited by

e16 Visual Diagnosis: Two Cases of Abdominal the Accreditation Council for Continuing Medical Education
(ACCME) to provide continuing medical education for
Pain after Trauma physicians.
The AAP designates this journal-based CME activity for a
Stephanie Hartman, Robin Petroze, Eugene McGahren maximum of 1.00 AMA PRA Category 1 Credit TM. Physicians
should claim only the credit commensurate with the extent
Pediatrics in Review® (ISSN 0191-9601) is owned and controlled by the American Academy of Pediatrics. of their participation in the activity.
It is published monthly by the American Academy of Pediatrics, 141 Northwest Point Blvd., Elk Grove Village, IL 60007-1098.
Statements and opinions expressed in Pediatrics in Review® are those of the authors and not necessarily those of the American This activity is acceptable for a maximum of 1.00 AAP credit.
Academy of Pediatrics or its Committees. Recommendations included in this publication do not indicate an exclusive course of These credits can be applied toward the AAP CME/CPD*
treatment or serve as a standard of medical care. Award available to Fellows and Candidate Members of the
Subscription price for 2016 for print and online/online only: AAP/CPS Member $204/$156; AAP National Affiliate Member $160/$110; AAP.
Nonmember $255/$198; Allied Health or In-training $190/$128. Institutions call for pricing (866-843-2271). For overseas delivery, add $120. The American Academy of Physician Assistants accepts
Current single issue price is $10 domestic, $12 international. Replacement issues must be claimed within 6 months from the date of issue certificates of participation for educational activities certified
and are limited to three per calendar year. Periodicals postage paid at ARLINGTON HEIGHTS, ILLINOIS and at additional mailing offices. for AMA PRA Category 1 Credit TM from organizations accredited
© AMERICAN ACADEMY OF PEDIATRICS, 2016. by ACCME. Physician assistants may receive a maximum of 1.00
All rights reserved. hour of Category 1 credit for completing this program.
Printed in USA. No part may be duplicated or reproduced without permission of the American Academy of Pediatrics.
POSTMASTER: Send address changes to PEDIATRICS IN REVIEW®, American Academy of Pediatrics Customer Service Center, 141 This program is accredited for 1.00 NAPNAP CE contact
Northwest Point Blvd., Elk Grove Village, IL 60007-1098. hour; pharmacology (Rx) and psychopharmacology contact
Pediatrics in Review® Print Issue Editorial Board Disclosures hours to be determined per the National Association of
The American Academy of Pediatrics (AAP) Policy on Disclosure of Financial Relationships and Resolution of Conflicts of Interest for Pediatric Nurse Practitioners (NAPNAP) Continuing Education
AAP CME Activities is designed to ensure quality, objective, balanced, and scientifically rigorous AAP CME activities by identifying Guidelines.
and resolving all potential conflicts of interest before the confirmation of service of those in a position to influence and/or control It has been established that each CME activity will take the
CME content. All individuals in a position to influence and/or control the content of AAP CME activities are required to disclose to the learner approximately 1 hour to complete.
AAP and subsequently to learners that the individual either has no relevant financial relationships or any financial relationships with
the manufacturer(s) of any commercial product(s) and/or provider(s) of commercial services discussed in CME activities. Commercial *Continuing Professional Development
interest is defined as any entity producing, marketing, reselling or distributing health-care goods or services consumed by, or used How to complete this activity:
on, patients.
Pediatrics in Review can be accessed and reviewed in print or
Each of the editorial board members, reviewers, question writers, PREP Coordinating Committee members and staff has disclosed, if
applicable, that the CME content he/she edits/writes/reviews may include discussion/reference to generic pharmaceuticals, off-label online at http://pedsinreview.aappublications.org. Learners
pharmaceutical use, investigational therapies, brand names, and manufacturers. None of the editors, board members, reviewers, can claim credit monthly online upon completion of each CME
question writers, PREP Coordinating Committee members, or staff has any relevant financial relationships to disclose, unless noted article. The deadline for completing this activity is December
below. The AAP has taken steps to resolve any potential conflicts of interest. 31, 2018. Credit will be recorded in the year in which it is
Disclosures submitted. It is estimated that it will take approximately 1 hour
• Robert Baker, MD, PhD, disclosed he has an author relationship with AstraZeneca. to complete each CME article. This activity is not considered
• Lynn Garfunkel, MD, FAAP, disclosed that her family member is an employee of Philips Healthcare. to have been completed until the learner documents
• Nupur Gupta, MD, MPH, disclosed she has a financial relationship with Springer US as co-editor for MassGeneral Hospital participation in that activity to the provider via online
for Children Handbook of Pediatric Global Health. submission of answers. Course evaluations are online.
• Gregory A. Hale, MD, disclosed that he is on the board of directors of Make A Wish, Suncoast Chapter, and the Sickle Cell
Disease Association of America, North Pinellas Chapter; and that he has research grants with Hyundai Hope on Wheels and
with the V Foundation for Cancer Research.
• Miriam Weinstein, MD, has disclosed she is on the advisory board of Tribute Pharmaceuticals; on the advisory board and has
a consensus meeting/author relationship with Pierre Fabre; is a paid consultant and advisory board member for Johnson &
Johnson and Valeant; on the advisory board of Valeo Pharma Inc.; is a paid consultant, advisory board member, and receives
honoraria and grants from LaRoche-Posay; and receives honoraria for lectures from Galderma and Pediapharm.
The journal extends special thanks to the following question writers and reviewers who contributed to this issue:
• Jorina Elbers, MD Answer Key appears on page 171.
• Sydney Rice, MD
• Richard Sills, MD
• Heather Symons, MD
*
ed iatric CME/CPD

2016 AAP CME SCHEDULE The Best P


iatric Care
For The Best Ped

Register Online: shop.aap.org/professional-education/live-activities Call Toll-Free: 866/THE-AAP1 (866/843-2271)

JANUARY FEBRUARY MARCH APRIL MAY JUNE

+NeoPREP® O P
 ractical Pediatrics O Practical Pediatrics +Workshop on O P
 ractical Pediatrics O P
 ractical Pediatrics
An Intensive Review CME Course CME Course Perinatal Practice CME Course CME Course
and Update of Paradise Island, Orlando, FL Strategies Hilton Head Island, SC Anaheim, CA
Neonatal-Perinatal The Bahamas March 18 – 20 Scottsdale, AZ May 27 – 29 June 16 – 18
Medicine February 19 – 21 April 8 – 10
Atlanta, GA O P
 ractical Pediatrics
January 23 – 29 +Imaging of Child Abuse: +Neonatal/Perinatal CME Course
Exam Room, Reading Coding Seminar Seattle, WA
Room, and Court Room Scottsdale, AZ May 27 – 29
Orlando, FL April 8
February 26 – 28

Y PREP® The Course


Kansas City, MO
February 27 – March 2

+Pediatric Academic
Societies Annual
Meeting (PAS)
Baltimore, MD
April 30 – May 3

OCTOBER DECEMBER
JULY AUGUST SEPTEMBER NOVEMBER
+Pediatric Hospital +PREP®:EM O Practical Pediatrics VNational Conference O Practical Pediatrics O Practical Pediatrics
Medicine An Intensive Review CME Course & Exhibition CME Course CME Course
Chicago, IL and Update of Pediatric New York, NY San Francisco, CA Naples, FL Chicago, IL
July 28 – 31 Emergency Medicine September 2 – 4 October 22 – 25 November 4 – 6 December 9 – 11
Chicago, IL Pre-conference Session
August 6 – 10 Y P REP® The Course +DB:PREP®
Philadelphia, PA October 21 An Intensive Review
September 10 – 14 and Update of
Developmental-
Behavioral Pediatrics
Anaheim, CA
November 30 –
December 4

OPractical Pediatrics CME Courses YPREP® The Course +Subspecialty/Section VNational Conference & Exhibition
Designed for pediatricians, family An intensive review and update of pediatrics featuring CME Courses Features educational sessions in nearly 60 topic areas to suit
physicians, nurses, nurse practitioners, course content based on the content specifications Appropriate for the pediatric a variety of learning styles. Home to the largest pediatric technical
physician assistants, and allied health issued by the American Board of Pediatrics (ABP) subspecialist or the general exhibit, the National Conference showcases the latest products and
professionals caring for children, for Maintenance of Certification™ (MOC). The course pediatrician with a particular services, as well as AAP educational and professional resources and
Practical Pediatrics CME Courses features a variety of educational formats to meet interest in the topic. programs. From hot-topic plenary sessions to small interactive,
feature nationally prominent faculty different learning styles including: lectures, case- hands-on workshops, this conference will offer you every opportunity
presenting topics that highlight based sessions, faculty panels, visual diagnosis, to enhance your clinical skills and expand your appreciation of our
current issues in pediatrics. pre/post-course self-assessment and hot topics. wonderful profession. The AAP welcomes you to be an active part of
our annual fall event—the AAP Experience!
The American Academy of Pediatrics (AAP) is accredited by the Accreditation Council for Continuing Medical Education (ACCME) to provide continuing medical education for physicians.
These activities have been approved for AMA PRA Category 1 Credit™.
*Continuing Professional Development.
Commentary
Zika Virus – Pediatricians Be Aware
Every few years, a “new” (not really) funny-
sounding infectious disease is in the news
and causing anxiety: first it was SARS
(severe acute respiratory syndrome), then
avian flu, swine flu, dengue, MERS (Middle
East respiratory syndrome), chikungunya,
Ebola, and now in 2016 it’s Zika virus.
Zika virus was first identified in 1947 at
the East African Virus Research Institute (now the Uganda Virus Research
Institute) in Entebbe, Uganda, as a cause of febrile illness in rhesus macaques.
(1) Until 2007, Zika virus caused only rare cases of human disease in Africa and
Southeast Asia. However, in April 2007, an outbreak was reported on Yap Island
that subsequently spread to other Polynesian islands. This was followed in 2015
by an explosive and widespread outbreak in South and Central America that
is ongoing. Brazil seems to be particularly severely affected.
Zika virus is transmitted by Aedes mosquitoes, the same mosquito that transmits
Dengue and Chikungunya viruses. The Aedes mosquitoes that are known to transmit
or can potentially transmit Zika virus are present in 32 States (2). Although so far
no autochthonous cases of Zika virus transmitted by mosquitoes have been
diagnosed in the United States, one sexually transmitted case of Zika virus has
been identified in the United States during the current pandemic. (3)
The incubation period for Zika virus infection is 2 to 14 days. The disease
has a wide spectrum and only 1 in 5 infected patients becomes symptomatic.
Hospitalizations are uncommon and death is rare. Clinically, Zika virus infec-
tion presents similarly to many other viral infections, with fever (often low-
grade), vomiting, maculopapular rash, arthralgias, myalgias, retro-orbital pain,
and conjunctivitis.
Serologic diagnosis is not dependable because of potential cross-reactivity
with dengue and chikungunya viruses. Polymerase chain reaction that can detect
the RNA of Zika virus is available from the Centers for Disease Control and
Prevention (CDC) and some state health departments.
There is no commercially available test for Zika virus and no specific antiviral
treatment; management is primarily supportive. There is also no vaccine to
protect against Zika virus infection. Prevention is largely dependent on avoidance
of areas where there is active Zika virus transmission (Figure) as well as mosquito
control and measures to prevent mosquito bites.
Compared to previous “new” emerging infections, Zika virus infection
has particular interest for pediatricians because of the major concern that
AUTHOR DISCLOSURE Dr Rathore has such infection may be responsible for microcephaly in infants born to infected
disclosed no financial relationships relevant to women. Although no causal relationship has been determined between Zika
this article. This commentary does not contain
a discussion of an unapproved/investigative virus infection during pregnancy and microcephaly in the newborn, the many-
use of a commercial product/device. fold increase in cases of microcephaly in the midst of a Zika virus epidemic

Vol. 37 No. 4 APRIL 2016 133


Figure. Areas of reported Zika virus transmission.

offers compelling epidemiologic suggestion of a link. (4) A Education and Service (UF CARES)
total of 2,401 suspected cases of microcephaly have been Jacksonville, FL
reported in Brazil during the period of outbreak. Of these,
134 were confirmed as being related to Zika virus infection, References
102 were considered not related, and 2,165 are still under 1. Dick GWA, Kitchen SF, Haddow AJ. Zika virus. I. Isolations
investigation. (5) Further careful research is needed to and serological specificity. Trans R Soc Trop Med Hyg. 1952;
determine if this temporal association is causative. 46(5):509–520

Because of this concern, the CDC recommends that 2. Fauci AS, Morens DM. Zika virus in the Americas – yet another
arbovirus threat. N England J Med. 2016;374(7):601–604
pregnant women avoid travel to areas of ongoing Zika virus
3. Oster AM, Brooks JT, Stryker JE, et al. Interim guidelines for
transmission. If travel is necessary, measures should be
prevention of sexual transmission of Zika virus - United States, 2016.
taken to prevent mosquito bites. Pregnant women returning MMWR. 2016;65(5):120–121, DOI: http://dx.doi.org/10.15585/
from areas of Zika virus activity should consider testing to mmwr.mm6505e1
determine if they have become infected. (6) 4. Oliveira Melo AS, Malinger G, Ximenes R, Szejnfeld PO, Alves
Sampaio S, Bispo de Filippis AM. Zika virus intrauterine infection
This “new” viral infection is another reminder that world
causes fetal brain abnormality and microcephaly: tip of the iceberg?
is becoming smaller, and infections once exotic and far off Ultrasound Obstet Gynecol. 2016;47(1):6–7
can reach our shores quickly and sometimes stealthily. We 5. European Centre for Disease Prevention and Control. Epidemiological
need to be vigilant in identifying potential emerging infec- Update: Outbreaks of Zika Virus and Complications Potentially Linked to
the Zika virus infection. 2015. Available at: http://ecdc.europa.eu/
tion threats quickly. Building public health infrastructure in
en/press/news/_layouts/forms/News_DispForm.aspx?ID¼
under-resourced parts of the world benefits not just local 1342&List¼8db7286c-fe2d-476c-9133-18ff4cb1b568&Source¼http%3A
populations but those of us in the resource-rich parts of the %2F%2Fecdc%2Eeuropa%2Eeu%2Fen%2Fpress%2Fepidemiological
world. %5Fupdates%2FPages%2Fepidemiological%5Fupdates%
2Easpx#sthash.oX5TQfDj.dpuf. Accessed February 5, 2016
6. Petersen EE, Staples JE, Meaney-Delman D, et al. Interim
Mobeen H. Rathore, MD, CPE
guidelines for pregnant women during a Zika virus outbreak –
Editorial Board member United States, 2016. MMWR Morb Mortal Wkly Rep. 2016;65
University of Florida Center for HIV/AIDS Research, (2):30–33

Parent Resources from the AAP at HealthyChildren.org


• https://www.healthychildren.org/English/ages-stages/prenatal/Pages/Zika-Virus.aspx
• Spanish: https://www.healthychildren.org/Spanish/ages-stages/paginas/Pages/Zika-Virus.aspx

134 Pediatrics in Review


Hematopoietic Stem Cell Transplantation in
Children and Adolescents
Gregory M.T. Guilcher, MD*
*Section of Pediatric Oncology/BMT, Alberta Children’s Hospital; Departments of Oncology and Pediatrics, University of Calgary, Calgary, Alberta, Canada.

Educational Gap
Hematopoietic stem cell transplantation (HSCT) indications and practices
have changed significantly over the last 20 years. Evolving hematopoietic
stem cell sources, less toxic conditioning regimens, and improving graft-
versus-host disease prophylaxis and therapy have broadened the
application of HSCT from malignant conditions to increasing numbers of
nonmalignant diseases.

Objectives After completing this article, the reader should be able to:

1. Understand general principles of allogeneic and autologous


hematopoietic stem cell transplantation (HSCT), including the variety of
hematopoietic stem cell sources.
2. Discuss the variability in intensity of current conditioning approaches,
which influences the risks and applicability of HSCT.
3. Recognize that HSCT involves acute and chronic complications and the
importance of general clinicians and subspecialists in their
management.
4. Review the pathophysiology of graft-versus-host disease, its
presentation, and its prevention and management.
5. Identify the increasing number of nonmalignant indications for HSCT in
children such that children who might benefit from this procedure are
considered for timely referral as appropriate.

CASE STUDY

A 1-year-old child is referred to your office for a developmental assessment due to


delayed speech and gross motor skills. You notice coarse facial features and on
physical examination document corneal clouding, hepatosplenomegaly, and
AUTHOR DISCLOSURE Dr Guilcher has numerous skeletal deformities. You suspect a metabolic disorder and request
disclosed no financial relationships relevant to
an urgent referral to a metabolic specialist. The specialist clinically diagnoses
this article. This commentary does contain a
discussion of an unapproved/investigative Hurler syndrome (mucopolysaccharidosis IH) and confirms a-L-iduronidase
use of a commercial product/device. deficiency with urinary glycosaminoglycan testing and subsequently by enzyme

Vol. 37 No. 4 APRIL 2016 135


deficiency in fibroblasts. While genetic testing results are
pending, you discuss the case with the metabolic specialist TABLE 1. Nomenclature for Hematopoietic
and agree that an urgent referral to a pediatric hematopoietic Stem Cell Donors
stem cell transplantation (HSCT) specialist is warranted
Matched Sibling Donor MSD
before genetic testing results are available. The best neuro-
Mismatched Sibling Donor MMSD
logic outcomes are seen when HSCT is performed as soon
as possible, preferably before age 2 years. Having general Matched Familial Donor (eg, parent) MFD
knowledge about HSCT planning and complications, you Mismatched Familial Donor MMFD
help the family prepare for their meeting with the pediatric
Matched Unrelated Donor MUD
HSCT specialist, allowing for a more productive consulta-
Mismatched Unrelated Donor MMUD
tion, and offer to share ongoing care of the child both before
HSCT and during subsequent follow-up. Matched indicates all tested human leukocyte antigen (HLA) loci are the
same between donor and recipient.
Mismatched means at least 1 HLA locus differs between donor and
NOMENCLATURE recipient (at either allelic or antigenic level of testing).

HSCT is the procedure of infusing blood stem cells from a


donor into a recipient. When the donor and recipient are haplotype, typically matched at 5 to 8 of 10 HLA loci.
different people, the procedure is termed an allogeneic Unrelated HSC products may come from UCB donations
HSCT; if the donor and recipient is the same person, it is or a living adult donor (not minors).
an autologous HSCT. Syngeneic HSCT describes a donation Conditioning refers to the preparative chemotherapy,
between identical twins. immunotherapy, and/or radiotherapy given to a recipient
Hematopoietic stem cells (HSCs) may be collected from before stem cell infusion to facilitate engraftment of allo-
bone marrow, peripheral blood, or the umbilical cord/ geneic donor HSCs and to prevent rejection. In this setting,
placental unit of a newborn (UCB). the HSCs are a primary component of the curative therapy;
Human leukocyte antigens (HLAs) are tested at major in autologous HSCT, the conditioning is the actual therapy
histocompatibility loci: Class I (A, B, and C) and Class II and the HSCs are administered to rescue the hematopoietic
(DR; DQ in some centers). At least 6 loci routinely are system. Myeloablative conditioning refers to intensive che-
analyzed for a UCB product and 8 to 10 loci for a live donor motherapy and/or radiation doses sufficient to cause bone
product (ie, bone marrow or peripheral blood). The degree marrow aplasia in the absence of HSC infusion. Reduced-
of matching is expressed as the numerator of matched loci intensity conditioning (RIC) describes nonmyeloablative or
over the denominator of loci tested. HLA matching may be less intensive conditioning regimens.
tested at low (antigenic), medium, or high (allelic) levels of HSCs for UCB and autologous donation must be cryo-
resolution. preserved, whereas most allogeneic live donor products are
Graft-versus-host disease (GVHD) is a serious and poten- donated during the conditioning of the recipient. Allogeneic
tially life-threatening complication of HSCT in which the products may also be manipulated to reduce plasma, red
donor T cells cause an inflammatory response in the recip- blood cells, or T cells, depending on the donor-recipient
ient tissues. This complication is described in detail later, blood group matching/mismatching, the stem cell source,
but the risk of its development has been historically reduced the routine practices of the HSCT center, and the indication
by the best possible HLA matching at major loci as well as for HSCT.
the use of a related donor due to closer matching at untested
minor histocompatibility antigens. Newer approaches to
TRENDS IN PRACTICE
haploidentical HSCT (see definition later) and novel GVHD
prevention strategies, however, are reducing GVHD rates, Internationally, more than 2,000 allogeneic HSCTs were
even in the setting of greater HLA disparity. reported to have been performed in recipients younger than
Allogeneic HSC donors are further characterized in age 20 years in 2012. (1) The use of UCB has increased over
terms of the relationship between the donor and recipient the last 20 years, as have donations from unrelated live
(Table 1). Fully matched related donations can come from a donors. These trends are affected by improvements in
minor or adult sibling or rarely a parent (often with a history supportive care (including GVHD prevention and treat-
of consanguinity). Haploidentical HSCT involves donation ment) as well as donor availability, with expanded live donor
from a first-degree relative (usually a mother) who shares 1 and UCB registries.

136 Pediatrics in Review


RIC was developed for older recipients who were ineli- unrelated donor with the same number of matched loci.
gible for myeloablative conditioning due to comorbidities. Unrelated donors may be identified through international
Its use has expanded to many nonmalignant indications for live donor registries or accredited public UCB banks. Iden-
children in whom a phenotype can be reversed with even tifying an unrelated donor and proceeding with HSCT
relatively low numbers of engrafted donor HSCs (mixed usually takes 1 or more months, depending on the rarity
donor chimerism) and there is a mix of hematopoietic cells of the recipient HLA-typing, donor availability to proceed
of donor and recipient origin. Several conditions, such as with donation, and medical clearance of both donor and
severe combined immune deficiency, hemophagocytic lym- recipient. This process is generally shorter for UCB prod-
phohistiocytosis, and hemoglobinopathies, are known to be ucts because the donation has already been made and the
cured with stable mixed-donor chimerisms as low as 20% to product has been cryopreserved.
30%. (2) The appeal of RIC lies in reduced rates of GVHD and Allogeneic stem cells can be donated as 1 of 3 stem cell
transplant-related mortality (TRM) in addition to fewer acute sources:
and late toxicities due to lower doses of conditioning agents. • Bone marrow
The increased use of RIC and haploidentical HSCT has • Peripheral blood stem cells
also influenced the growing proportion of HSCT recipients • Umbilical cord blood
with nonmalignant diseases. This trend toward HSCT for Table 2 describes the method of donation as well as
nonmalignant conditions is due to improved outcomes with advantages and disadvantages of each source of allogeneic
upfront non-HSCT childhood leukemia therapies as well as HSCs. Peripheral blood stem cells are less commonly used
advancements in the safety of HSCT. As the risks of mor- in pediatric HSCT recipients due to higher risks of chronic
bidity and mortality decrease, the potential application of GVHD; they are typically only used for malignant indica-
HSCT as a curative option for various nonmalignant dis- tions or as part of a RIC protocol. Many considerations are
eases broadens. balanced in choosing a stem cell source: the recipient’s
Expertise in haploidentical HSCT is increasing world- underlying condition, the degree of HLA matching, the
wide, particularly in Europe and the United States. Its appeal urgency of the HSCT, the risk to the donor (particularly
lies in the almost universal availability of a donor, particu- for minor sibling donors who cannot consent for them-
larly for potential recipients whose HLA haplotypes are selves), donor preference for method of donation, donor
underrepresented on existing volunteer registries. Risks health status (which may preclude a method of donation),
of GVHD and infection (due to T-cell depletion) as well ABO status of donor and recipient, and size discrepancy
as required laboratory infrastructure complicate its applica- between donor and recipient. The stem cell dose (ie, num-
tion, but improved supportive care options have increased ber of donor HSCs) required for the HSCT recipient is
the practice of haploidentical HSCT. Newer techniques such calculated based on recipient weight, which may not be
as the use of cyclophosphamide after HSC infusion have achievable based on the size of a prospective living donor.
resulted in markedly improved rates of engraftment and Donations from living donors are almost always collected
reduced rates of GVHD and infectious complications. (3) within 2 days of infusion to ensure that the HSCT is not
subsequently cancelled due to a change in the recipient’s
eligibility status and to avoid cell loss with cryopreservation.
PRINCIPLES OF HSCT
UCB products contain a fixed number of cryopreserved stem
Allogeneic HSCT involves the replacement of the deficient cells. A given UCB unit may have sufficient stem cells for a
recipient hematopoietic system with that of the donor. The smaller recipient but may be inadequate for a larger patient.
best possible HLA-matched donor is used, with a preference Additional considerations include the age of the donor,
for matched sibling, followed by matched related donors. the donor sex, and any pregnancies (if applicable). Younger
HLA testing and matching is currently limited to 8 to 10 donors generally have more cellular bone marrows and
major histocompatibility loci for living donors, yet minor produce greater HSC yields. In addition, their donations
histocompatibility (H) antigens also influence the risk of are associated with lower GVHD rates in recipients. Dona-
GVHD. Minor H antigens are potentially immunogenic tions from females, particularly with increasing parity, are
peptides genetically coded outside of the major histocom- associated with higher rates of GVHD. Male recipients with
patibility complex (MHC). (4) The coding loci for H antigens female donors are at highest risk. (10)
are scattered throughout the genome in contrast to the Autologous stem cell collections are almost always from
MHC being coded on chromosome 6. As a result, a related peripheral blood, with bone marrow harvests usually
fully HLA-matched donor is almost always preferred to an reserved for failed peripheral blood collections. Such

Vol. 37 No. 4 APRIL 2016 137


TABLE 2. Review of Hematopoietic Stem Cell Sources
HSC STEM CELL
SOURCE METHOD OF COLLECTION ADVANTAGES DISADVANTAGES

Bone Marrow Donor undergoes anesthesia, is High engraftment rates Pain after harvest for donor
placed prone, and marrow
harvested bilaterally from iliac crests
Collection proceeds until donor Lower rates of chronic GVHD Donor size limits volume of marrow
maximum volume collected compared to peripheral blood (6) that can be harvested (transfusion
(10-20 mL/kg) or target HSC of donor is discouraged)
dose achieved (whichever
comes first)
Research underway regarding High volumes of product can cause
benefits of G-CSF administration volume overload for recipient
before donation to improve yield (5)
ABO incompatibility warrants
processing of sample to reduce
red blood cells and/or plasma
(HSC loss occurs with each
processing step)

Peripheral Blood Donor receives G-CSF for 3-5 days High engraftment rates G-CSF exposure to donor can cause
prior to donation bone pain
Apheresis catheter placement for Higher stem cell yields Ongoing concern over long-term
donor (often a femoral venous line Possibly lower relapse rates risks of G-CSF exposure to donor
under anesthesia for pediatric donors) for malignant diseases (6)(7) bone marrow (although data
show no clear evidence of harm)
Higher rates of chronic GVHD (6)

1-2 days of donation on apheresis May allow for lower-intensity Smaller donors unable to undergo
machine (typically 4-8 hr/day) conditioning apheresis without blood product
exposure due to extracorporeal
blood volume (transfusion of
donor is discouraged)
Collection proceeds until target HSC Donor may not mobilize stem cells
dose achieved (diminishing yield peripherally (more common in
with ongoing time on circuit) adult donors)
Umbilical Cord Collected after clamping of umbilical Product can be procured Higher rates of nonengraftment
Blood cord blood quickly for HSCT (graft failure) (9)
Method of collection should not HLA mismatching more permissive Cell dose per recipient weight is
compromise mother or neonatal (ie, 4-6/6 match can be used) limited to existing cryopreserved
donor in any way due to lower rates of GVHD product (fixed) and may be
lowered, depending on viability
before freezing and after thawing
Sample is processed and cryopreserved May be superior for metabolic
disorders (8)
May obviate the need for minor Higher rates of viral infections
sibling donation if sibling UCB (delayed immune recovery) (9)
available
No donor risk Cannot access additional HSCs if
nonengraftment or early relapse
for donor lymphocyte infusion
Medical history of donor generally
unknown

G-CSF¼granulocyte colony-stimulating-factor, GVHD¼graft-versus-host disease, HLA¼human leukocyte antigen, HSC¼hematopoietic stem cell,
HSCT¼hematopoietic stem cell transplantation, UCB¼umbilical cord blood

collections are typically timed at the point of initial hema- cryopreserved to be used later to rescue the patient following
topoietic recovery following myelosuppressive chemotherapy, high doses of chemotherapy or radiation, allowing for more
in combination with granulocyte colony-stimulating factor rapid hematopoietic recovery.
(G-CSF). “Steady-state” collections can also be performed HSCs are infused into the recipient after conditioning
with G-CSF administration alone. The HSCs are then chemotherapy and/or radiation (see next section). Such cells

138 Pediatrics in Review


are infused into the venous system using a central vascular chemotherapy with or without radiation therapy targeted
access device but may also be infused into a peripheral at the underlying disease (usually malignant). The goal is to
intravenous catheter. No filters can be placed on the tubing, rescue the patient after otherwise intolerable doses of these
which could block the HSCs from entering the circulation. agents given to intensify therapy.
Premedication is required for cryopreserved products to avoid
reaction to the preservative required for the cells to tolerate
RISKS OF HSCT
freezing, and such premedication is also used for ABO
incompatibilities with bone marrow products. The HSCs HSCT is associated with numerous acute and long-term
enter the marrow via adhesion molecule recognition and toxicities. The conditioning, its intensity (myeloablative
start to grow and mature immediately. However, 2 to 3 weeks versus RIC), preexisting comorbidities, prior chemotherapy
generally is required for measurable neutrophil counts (or exposure, and the stem cell source all influence the risks of
engraftment) and for red blood cell and platelet transfusion complications and TRM. Children and adolescents gener-
independence. The fastest rates of HSC engraftment are seen ally tolerate myeloablative conditioning better than adults,
with autologous rescues and with peripheral blood stem cell but TRM rates are still typically 5% to 10%. RIC was initially
products; UCB products are often the slowest to engraft. (9) designed to offer HSCT to patients with comorbidities, so
TRM rates are inherently lower, as are rates of many
toxicities. HSCT adverse effects on growth, development,
CONDITIONING FOR HSCT
and fertility are especially pertinent in children and adoles-
Conditioning, or the preparative regimen, refers to the cents (Table 3). (11)(12) A detailed discussion of these late
combination of chemotherapy, immunotherapy, and/or effects is beyond the scope of this article, but comprehensive
radiation therapy given to an HSCT recipient before the follow-up by general pediatricians and a team with expertise
HSC infusion. Such conditioning is usually administered in HSCT late effects care and surveillance is recommended.
over 1 to 2 weeks before HSC infusion. Immune suppres- Surveillance guidelines have been published by the Chil-
sion, notably reduction or ablation of innate immune and T dren’s Oncology Group and other research bodies. (11)(12)(13)
cells, is necessary to prevent rejection of the HSCs in the
setting of allogeneic HSCT. Conditioning may also serve as Infections/Immune Reconstitution
disease-directed therapy in allogeneic HSCT for malignant HSCT usually involves myelosuppression as well as func-
disease. Serotherapy is a form of immunotherapy typically tional impairment of adaptive immunity. (14) As mentioned
involving antithymocyte globulin or alemtuzumab (mono- previously, neutrophil engraftment typically occurs 2 to 3
clonal antibody to CD52) that is intended to address host weeks after HSC infusion, which is an important milestone
immune cell depletion, although it is primarily adminis- for innate immune protection against bacteria and fungi.
tered for in vivo GVHD prophylaxis. Total body irradiation Natural killer cell recovery usually is complete by 1 month
(TBI) is highly myelosuppressive but is associated with post-HSCT, offering additional protection against infection.
many undesirable acute and late toxicities. T-cell function is impaired by intent during periods of
Myeloablative conditioning is standard for malignant prophylaxis or therapy for GVHD, and GVHD in itself is
disease HSCT indications and has been used historically a dysregulated immune state, with poor function and pro-
for nonmalignant conditions as well. The goal of myeloa- tection against infection. For those HSCT recipients who
blation is to replace all cell lines of the hematopoietic sys- can stop GVHD prophylaxis by 6 months post-HSCT,
tem (eg, lymphoid, myeloid) completely with donor HSCs. lymphocyte class switching (producing immunoglobulin
Although most experts consider eradication of all recipient [Ig]G after IgM production) can be seen between 6 and 8
blood cells to be essential for a person with leukemia, as few months after HSC infusion.
as 20% donor cells in the deficient cell line can reverse the Children must be monitored for opportunistic infections
abnormal phenotype in a nonmalignant condition. (2) The after HSCT. Bacteremia and sepsis are frequent, particularly
ability to cure a nonmalignant disease in the setting of during the neutropenic phase before engraftment. Fungal
mixed-chimerism following RIC has greatly increased the infections are also a concern during neutropenic phases or
safety and application of HSCT to a broader number of corticosteroid therapy. Respiratory viruses such as respira-
nonmalignant diseases. Graft failure after RIC often results in tory syncytial virus and adenovirus can be devastating in an
autologous recovery of the recipient’s original HSCs. immunocompromised host. Primary infection or reactiva-
Autologous HSCT conditioning regimens are almost tion with cytomegalovirus and Epstein-Barr virus (EBV)
exclusively composed of high-dose combinations of warrant preemptive surveillance and intervention based

Vol. 37 No. 4 APRIL 2016 139


for up to 1 year post-HSCT and may also confer some
TABLE 3. Late Effects of Pediatric Hematopoietic protection against varicella-zoster virus. (15)(16) BK virus
Stem Cell Transplantation (11)( 12) is a polyoma virus that is generally harmless in an immu-
nocompetent host. However, it can cause hemorrhagic
Endocrine Growth disturbance (including growth
hormone deficiency) cystitis and renal dysfunction in HSCT recipients if viremia
Hypothyroidism is present. Pneumocystis jiroveci prophylaxis is also indicated
Thyroid nodules
Hypogonadism until immune suppression has been withdrawn.
Delayed or precocious puberty The Centers for Disease Control and Prevention, in
Infertility
Obesity (including sarcopenic obesity)
collaboration with several international HSCT organizations,
Osteopenia/osteoporosis have established guidelines for infectious prophylaxis, and
Avascular necrosis international guidelines also exist for the management of
Metabolic syndrome
fever and neutropenia in pediatric HSCT recipients. (16)(17)
Ophthalmologic Cataracts
Xerophthalmia Finally, children require reimmunization after HSCT, but
clinicians must exercise caution regarding the timing of live
Auditory Hearing loss
vaccine administration. Recommendations for the timing of
Neurologic Neurocognitive impairment immunizations for children who have undergone HSCTcan
Cerebrovascular disease
be referenced and are updated regularly. (16)(18)
Pulmonary Pulmonary fibrosis
Bronchiolitis obliterans with or without
organizing pneumonia (usually chronic Mucositis
GVHD) Almost all children who undergo myeloablative HSCT
Cardiovascular Congestive heart failure experience mucositis. This painful inflammation of the
Conduction abnormalities
gastrointestinal mucosa is due to direct toxicity from con-
Valvular disease
ditioning agents and is compounded by a local inflam-
Renal Chronic kidney disease
Hypertension matory state in the setting of neutropenia. It can occur
Proteinuria anywhere between the oral mucosa and rectum, and inten-
Gastrointestinal Hepatic siderosis sive intervention with narcotic and adjuvant therapies is
Focal nodular hyperplasia of liver often required, with resolution typically occurring after
Esophageal strictures
GVHD of upper or lower tracts neutrophil engraftment. As the intensity of the conditioning
Hepatic GVHD is reduced, the severity of mucositis decreases. Nutrition
Secondary malignancy Acute myelogenous leukemia (almost support is commonly required while mucositis is present.
exclusive to autologous HSCT) The risk of bacterial translocation across the lining of the
Posttransplant lymphoproliferative
disease (non-Hodgkin lymphoma) mucosa and secondary HSV-1 and fungal infections are a
Solid tumors (skin, brain, thyroid, concern.
musculoskeletal, oral cavity, breast,
gynecologic)
Nutritional Support
Dental Disordered tooth eruption
Increased risk of caries Many children require nutritional supplementation post-
Xerostomia HSCT due to decreased intake, which may be related to
Psychosocial Depressed mood nausea, anorexia, malabsorption, or mucositis. Even when
Anxiety many other complications abate, many children and ado-
Posttraumatic stress disorder
Relationship difficulties lescents need support to ensure adequate hydration and
Vocational difficulties caloric intake. In addition, metabolic needs are often
Chronic fatigue
increased due to a catabolic state, with extensive tissue
GVHD¼graft-versus-host disease, HSCT¼hematopoietic stem cell healing required postconditioning. HSCT centers often
transplantation have strong preferences regarding the safest and most
beneficial method of nutritional supplementation. Intractable
nausea and gut integrity, with potential compromise due to
on international guidelines and institutional practices. EBV mucositis or GVHD, should be considered when deliber-
can be associated with posttransplant lymphoproliferative ating about enteral feeding. In the absence of contraindi-
disorder. Acyclovir prophylaxis for herpes simplex virus-1 cations, enteral feeding has potential benefits to the liver in
(HSV-1) in seropositive recipients is generally administered promoting biliary flow, which is important because the liver

140 Pediatrics in Review


is at risk of toxicity from conditioning, sinusoidal obstruc- agents. Although some degree of GVHD can be associated
tive syndrome (SOS), GVHD, and polypharmacy. with better overall survival for those with malignancy (due to
a graft-versus-leukemia effect), there is no benefit in non-
Transfusions malignant disease, and GVHD remains a barrier to the
Transfusion support is expected pre-engraftment, partic- application of HSCT for many nonmalignant conditions,
ularly for more intensive conditioning regimens. Opti- particularly if no matched family donor exists. (24)
mization of iron load pre- and post-HSCT may reduce Chronic GVHD is often seen months after HSCT (aver-
complications (such as SOS), and phlebotomy post-HSCT age onset at 6 months) and can be a devastating complica-
is employed for patients with high iron burdens post-HSCT tion. (25) Clinically this entity resembles systemic lupus
once stable engraftment has occurred. New research is erythematosus (SLE) or systemic sclerosis and can result in
exploring the role of iron burden on inflammation after debilitating skin, muscle, joint, liver, gut, and lung disease.
HSCT. (11) Dry eyes, dry mouth, and fatigue can also affect quality of
life. Prolonged immune suppression is required for more
Sinusoidal Obstructive Syndrome severe cases, which can result in opportunistic infection.
SOS is a serious hepatoxicity seen in 1% to 10% of HSCT (25) End-organ dysfunction, particularly of lung and liver, is
recipients, with risk increased for those with preexisting a major concern. Chronic GVHD can replace the condition
liver disease, allogeneic HSCT recipients, children with for which HSCT was performed, and although less common
high-risk neuroblastoma, and those who receive busulfan, in children than in adults, must always be considered during
cyclophosphamide, or TBI. (19) It involves occlusion of the decision-making process for HSCT.
sinusoidal venules due to microthrombi, with resulting liver Tolerance between the donor T cells and the HSCT
swelling and enlargement, pain, fluid retention, and recipient eventually results in the ability to reduce and
hepatorenal syndrome. Cholestasis is present, with variable usually discontinue immune-suppressive medications.
degrees of hepatic enzyme elevation. Although prevention Tapering of immune suppression occurs at scheduled time
is ideal, treatment can vary from diuresis in moderate cases points after HSC infusion in the absence of GVHD, with
to promising agents such as defibrotide in severe cases longer periods of prophylaxis and higher target levels for
with end-organ failure. (20) Defibrotide has yet to receive nonmalignant HSCTdisease indications. (26)( 27) For those
approval from the US Food and Drug Administration. who develop GVHD, once the GVHD is inactive for a
sufficient period of time, immune suppression is weaned
Pulmonary Complications
The differential diagnosis of pulmonary complications after
HSCT is broad, with common causes being infection and TABLE 4. Malignant Disease Indications for
volume overload. Other considerations include pneumoni- Allogeneic Pediatric HSCT
tis from radiation or alkylating agents, idiopathic pneumo-
Acute lymphoblastic leukemia
nia syndrome, and chronic GVHD. Respiratory failure
requiring intubation and ventilation is associated with sig- • Very high-risk features
nificant rates of mortality in immunocompromised recip- • Relapsed disease
ients of HSCT. (21) Acute myelogenous leukemia
• High-risk features
Graft-Versus-Host Disease
GVHD is an immune-based complication seen almost • Relapsed disease

exclusively in allogeneic HSCT. It involves tissue damage Myelodysplastic syndrome (preleukemic state)
and antigen exposure, antigen presentation, and alloim- Non-Hodgkin lymphoma
mune reactivity of donor T cells against recipient tissues.
• Relapsed/primary refractory disease
(22) Acute GVHD affects the skin, gastrointestinal tract
• Disease subtype may indicate allogeneic versus autologous HSCT
(typically colon, stomach, or duodenum), and liver. Notably,
these organs are prone to chemotherapy and radiation Hodgkin lymphoma
damage and are rich in antigen-presenting cells. Staging • Relapsed disease after autologous HSCT or primary refractory
systems describe the severity of each affected organ, with disease (usually second relapse) (28)
an overall grading assigned. (23) Grading determines the HSCT¼hematopoietic stem cell transplantation
need to treat with corticosteroids and potentially additional

Vol. 37 No. 4 APRIL 2016 141


TABLE 5. Nonmalignant Disease Indications for Allogeneic Pediatric
HSCT
Primary Immune Deficiencies
• Phenotype must be severe enough to justify HSCT
• Specific genetic mutation identification ideal (can support indication for HSCT as well as influence conditioning)

Hemoglobinopathies
• Thalassemia major

∘ Matched sibling or unrelated live donor


∘ Unrelated UCB and haploidentical HSCT experimental
• Sickle cell disease (Hg SS, Sß0, or SC)

∘ Matched sibling donor


∘ Unrelated donor and haploidentical HSCT experimental
∘ Indications vary among centers, often some evidence of prior sickle cell complications required
Inherited Bone Marrow Failure Syndromes
• Severe aplastic anemia
• Fanconi anemia
• Shwachman-Bodian-Diamond syndrome
• Diamond-Blackfan anemia
• Dyskeratosis congenita
• Amegakaryocytic thrombocytopenia

Metabolic/Genetic Disorders (29)


• Infantile osteopetrosis
• Mucopolysaccharidoses

∘ Hurler syndrome (MPS IH), standard of care


∘ Optional indications (after frontline enzyme replacement therapy, if available)
n Hurler/Scheie (MPS IH/S)
n Scheie (MPS IS)
n Maroteaux-Lamy (MPS VI)
n Sly (MPS VII)
• Leukodystrophies

∘ Cerebral X-linked adrenoleukodystrophy


n Before advanced disease
∘ Metachromatic leukodystrophy, late onset
∘ Krabbe disease, generally early onset
• Miscellaneous disorders, optional indications

∘ Fucosidosis
∘ a-mannosidosis
Continued

142 Pediatrics in Review


TABLE 5. (Continued )

∘ Aspartylglucosaminuria
∘ Farber
∘ Gaucher types 1 (non-neuronopathic) and 3 (Norrbottnian)
∘ Niemann-Pick type C-2
∘ Wolman syndrome

HSCT¼hematopoietic stem cell transplantation, MPS¼mucopolysaccharidosis, UCB¼umbilical cord blood

and subsequently stopped. For this reason, HSCT recipients Chronic transfusions for hemoglobinopathies are associ-
are not expected to receive lifelong immune suppression, in ated with significant risks of iron overload and resultant
contrast to patients who receive solid organ transplantation. complications. For some of these conditions, the risks of
HSCT are affected substantially by the type of donor avail-
able, and the resulting recommendation for HSCT may be
INDICATIONS FOR HSCT
affected. Primary immune deficiencies such as severe com-
Historically, most allogeneic HSCT procedures in children bined immune deficiency, X-linked chronic granulomatous
were for malignant diseases such as leukemias and lym- disease, and Wiskott-Aldrich syndrome are examples of
phomas. With improving cure rates using chemotherapy for nonmalignant diseases for which HSCT is commonly per-
such cancers, the proportion of nonmalignant disease indi- formed. A large body of evidence supports the safety and
cations for pediatric HSCT continues to increase. efficacy of HSCT for severe aplastic anemia, with increasing
data to guide clinicians in decision-making for inherited
Malignant Disease bone marrow failure syndromes. Thalassemia major has an
Common malignant disease indications for allogeneic established track record for related and unrelated HSCT,
HSCT in children are acute leukemias and some non- with a clear phenotype of lifelong transfusion dependence
Hodgkin and Hodgkin lymphomas. High-risk clinical/ and risk of iron overload.
biological features or relapse are usually present (Table Sickle cell disease (SCD) is increasingly recognized as a
4). Myelodysplastic syndrome, a preleukemic state with risk disease with limited life expectancy and variable quality of life
of conversion to acute myeloid leukemia, is almost always despite best supportive care. As a result, interest is growing in
treated with HSCT in children. Chronic myelogenous leu- the application of HSCT to those with sickling syndromes.
kemia is often managed with tyrosine kinase inhibitors Although a history of complications of SCD had been man-
alone, so fewer affected children and adolescents are rec- dated in the past to justify HSCT, the safer HSCT techniques
ommended to undergo HSCT. have prompted increasing interest from patients, hematolo-
Autologous HSCT is performed routinely for children gists, and HSCT practitioners to intervene before organ
with high-risk neuroblastoma and for relapsed lymphomas. dysfunction occurs, notably neurologic and lung injury.
Many brain tumor treatment plans are incorporating high- Some metabolic diseases such as mucopolysaccha-
dose chemotherapy and autologous HSCT, particularly for ridoses are routine indications for HSCT, although the poten-
children younger than age 3 years, in an effort to spare or tial benefits are less clear for other metabolic diseases. (29)
delay radiation therapy to the developing brain. Current Table 5 summarizes some of the more standard indications,
research is exploring the use of autologous HSCT in chil- with an acknowledgement that HSCT is performed in some
dren and adolescents with solid tumors, such as Ewing centers for life-threatening metabolic diseases with fewer
sarcoma, who have high-risk features. data regarding potential benefit. (29) HSCTcan help prevent
neurologic progression in a metabolic disease due to
Nonmalignant Disease replacement of the deficient enzyme by monocytes pro-
Allogeneic HSCT is increasingly performed for nonmalig- duced from the HSCs following engraftment. Because
nant disease indications as rates of TRM and GVHD are HSCT generally only halts and does not reverse neurologic
reduced. These diseases confer lifelong risks of morbidity or progression and knowing that HSC-derived enzyme
mortality and often require complex supportive care (Table 5). replacement can take months to reach the central nervous

Vol. 37 No. 4 APRIL 2016 143


system (CNS) due to slow migration of donor-derived
monocytes into the CNS, early HSCT is critical for indicated Summary
conditions. Generally other non-CNS manifestations of the
• Hematopoietic stem cell transplantation (HSCT) refers to the
metabolic disorders are not reversed with HSCT. Discus- infusion of either allogeneic or autologous hematopoietic stem
sions about the appropriateness of HSCT should happen cells.
relatively soon after making a diagnosis, and those who • Newer techniques to reduce the risk of complications are
manage such conditions routinely should be aware of expanding the applicability of HSCT.
evolving indications for this group of diseases. • Nonmalignant disease indications for HSCT are increasing.
The practice of autologous HSCT for nonmalignant • Observational and cohort studies (level C evidence) indicate that
conditions is relatively limited. Some encouraging results acute and long-term toxicities remain an important consideration
for those with severe SLE suggest that some patients may for patients, families, and clinicians in making a recommendation
for HSCT and warrant lifelong surveillance. (11)(12)(13)(21)
derive temporary benefit in terms of corticosteroid-sparing
or reduced disease activity. (30) Repopulation of the bone • Based on overwhelming evidence from observational studies
(level B evidence), graft-versus-host disease can be a significant
marrow and peripheral blood with fewer autoreactive
cause of morbidity and mortality in allogeneic HSCT. (22)(24)
clones, in addition to the use of disease-modifying agents
• General pediatricians and subspecialists should be aware of
such as cyclophosphamide as part of the conditioning, may
evolving and newly established nonmalignant indications for
explain this period of improvement. Gene therapy trials for HSCT to make appropriate referrals (level D evidence). (28)(29)
hemoglobinopathies are incorporating autologous HSC col- (30)
lection and ex vivo manipulation, with reinfusion following
conditioning designed to give the manipulated cells a survival
advantage. (31) The use of autologous HSCT for traumatic
brain injuries and cerebral palsy is an area of intense research, CME quiz and references for this article are at http://pedsinreview.
but these indications are experimental at present. aappublications.org/content/37/4/135.

Parent Resources from the AAP at HealthyChildren.org


• https://www.healthychildren.org/English/health-issues/conditions/cancer/Pages/Cancer-Therapies.aspx
• Spanish: https://www.healthychildren.org/Spanish/health-issues/conditions/cancer/Paginas/Cancer-Therapies.aspx

144 Pediatrics in Review


PIR Quiz
There are two ways to access the journal CME quizzes:
1. Individual CME quizzes are available via a handy blue CME link under the article title in the Table of Contents of any issue.
2. To access all CME articles, click “Journal CME” from Gateway’s orange main menu or go directly to: http://www.aappublications.
org/content/journal-cme.

1. A 13-year-old girl with acute myeloblastic leukemia has relapsed 6 months after REQUIREMENTS: Learners
completing her initial course of chemotherapy. You explain to the parents that the only can take Pediatrics in
potential cure will be hematopoietic stem cell transplantation (HSCT). Which of the Review quizzes and claim
following options is the best donor for this girl? credit online only at:
A. Allogenic transplant using a first cousin who matches at 8/10 loci. http://pedsinreview.org.
B. Allogenic transplant using a sibling who matches at 8/10 loci.
C. Allogenic transplant using an unrelated donor who matches at 8/10 loci. To successfully complete
D. Allogenic transplant using her mother who matches at 8/10 loci. 2016 Pediatrics in Review
E. Autologous transplant. articles for AMA PRA
2. Which of the following would be the best therapy for the child described in the previous Category 1 CreditTM,
question? learners must
A. Chemotherapy alone to attempt prolonged remission. demonstrate a minimum
B. Myeloablative conditioning prior to transplant. performance level of 60%
C. Reduced-intensity conditioning prior to transplant. or higher on this
D. Serotherapy prior to transplant. assessment, which
E. Total body irradiation prior to transplant. measures achievement of
the educational purpose
3. A 5-year-old boy underwent HSCT 12 days ago because of neuroblastoma. He is now
and/or objectives of this
complaining of increasing abdominal pain. You note that he has icterus and mild
activity. If you score less
generalized edema. Laboratory studies reveal a total bilirubin of 4.5 mg/dL (76.9 mmol/L)
than 60% on the
and conjugated bilirubin of 2 mg/dL (34.2 mmol/L) but only mild elevations in
assessment, you will be
transaminase values. The most likely cause of his symptoms is:
given additional
A. Cytomegalovirus. opportunities to answer
B. Hepatitis A. questions until an overall
C. Hepatitis B. 60% or greater score is
D. Sepsis. achieved.
E. Sinusoidal obstructive syndrome.
4. A 7-year-old girl with homozygous sickle cell anemia underwent HSCT from an unrelated,
This journal-based CME
human leukocyte antigen-identical donor 7 months ago. She has been complaining of
activity is available
fatigue for 2 weeks and now has developed a feeling of her mouth being dry. On physical
through Dec. 31, 2018,
examination she has a widespread nonspecific erythematous rash over her trunk and arms.
however, credit will be
There is no cyanosis or jaundice. She has shotty anterior cervical nodes but no other
recorded in the year in
significant adenopathy. The most likely cause of her symptoms is:
which the learner
A. Acute graft-versus-host disease. completes the quiz.
B. Chronic graft-versus-host disease.
C. Cytomegalovirus.
D. Epstein-Barr virus.
E. Human herpesvirus 6.
5. A 4-year-old girl presents with bruising and pallor. She is found to have pancytopenia. A
bone marrow aspirate and biopsy are diagnostic of myelodysplastic syndrome. Which of
the following is the most appropriate treatment for this child’s myelodysplastic syndrome?
A. Begin chemotherapy and evaluate the response long term.
B. Begin prophylactic antibiotics to prevent sepsis.
C. Maintain the patient on transfusions until she becomes unresponsive to them.
D. Observe the child until the pancytopenia becomes severe.
E. Proceed to HSCT once an appropriate donor is identified.

Vol. 37 No. 4 APRIL 2016 145


Physical Abuse of Children
Jill C. Glick, MD,* Michele A. Lorand, MD,† Kristen R. Bilka, MMS, PA-C‡
*Department of Pediatrics, University of Chicago; Medical Director, Child Advocacy and Protective Services, University of Chicago Comer Children’s Hospital,
Chicago, IL.

Division of Child Protective Services, Department of Pediatrics; Medical Director, Chicago Children’s Advocacy Center, John H. Stroger, Jr. Hospital of Cook
County, Chicago, IL.

Department of Pediatrics, University of Chicago; Physician Assistant, Child Advocacy and Protective Services, University of Chicago Comer Children’s Hospital, Chicago, IL.

EDITOR’S NOTE

This article stresses the importance of the “sentinel injury,” a physical injury that
is unusual for the age of the child and may herald more serious injuries, thereby
necessitating further evaluation.
Joseph A. Zenel, MD
Editor-in-Chief

Practice Gap
Before receiving a diagnosis of child abuse, 25% to 30% of abused infants
have “sentinel” injuries, such as facial bruising, noted by clinicians or
caregivers. (1)(2)(3)(4)(5)(6) Although easily overlooked and often considered
minor, such injuries are harbingers warning clinicians that pediatric patients
require further assessment. Appropriate intervention is critical, and the
clinician plays a major role in identifying children who present with signs
or symptoms concerning for child physical abuse by ensuring appropriate
and expeditious medical evaluations and reports to child protective services.

Objectives After completing this article, the reader should be able to:

1. Identify which injured children require a child abuse evaluation.


2. Recognize subtle signs and nonspecific symptoms of major trauma in
infants.
3. Understand sentinel injuries and their significance.
4. Know which laboratory and imaging studies to obtain when child
physical abuse is suspected.
5. Understand the legal obligation to report children with injuries that are
suspicious for physical abuse and develop a thoughtful approach to
informing parents of this legal obligation.

AUTHOR DISCLOSURE Drs Glick and Lorand


CASE PRESENTATION
and Ms Bilka have disclosed no financial
relationships relevant to this article. This
A private practice pediatrician receives a phone call from a community emergency
commentary does not contain a discussion of
department (ED) physician regarding one of her patients, a 4-month-old infant an unapproved/investigative use of a
being treated for bronchiolitis. The ED physician informs her that the baby’s chest commercial product/device.

146 Pediatrics in Review


radiograph has revealed multiple posterior rib fractures in Clinicians must appreciate that with few exceptions, almost
different stages of healing, and physical examination shows any injury can be either abusive or accidental.
a cluster of small bruises on her cheek. The mother denies a Once considered a strictly social problem, child abuse is
history of trauma and has no explanation for the findings. now also recognized as a medical problem. A recent survey
The ED physician is concerned that the baby has been by the Children’s Hospital Association revealed that more
abused and his plan includes admitting the patient to the than 90% of responding hospitals have child protection
hospital to obtain a head computed tomography (CT) scan, teams, and more than 50% have at least 1 of the 324 board-
skeletal survey, complete blood cell count, coagulation stud- certified child abuse pediatricians in the United States on
ies, electrolytes, and liver function tests. He also plans to staff. (8)
consult with the child abuse pediatrician and arrange for an Recognition of the profound impact of childhood expe-
evaluation of the patient’s siblings. Lastly, he tells the riences on adult health and well-being, beginning with
primary pediatrician that he will explain the clinical findings Feleitti’s landmark adverse childhood experiences study,
to the family and file a report with the child welfare system. further solidifies the need for clinicians to recognize
The primary pediatrician thanks him for contacting her and, possible maltreatment and intervene. (9) Adverse child-
recalling no significant medical history, pulls the patient’s hood experiences have wide-ranging, cumulative, and
chart. direct impacts on adult health, increasing the incidence
The baby’s most recent visit was slightly more than 1 of chronic diseases and early death. (9)(10) The role of
week ago for her routine 4-month health supervision the clinician is therefore not only limited to promoting
visit. She is a term infant who has no prior medical wellness but also to decreasing or eliminating long-term
complaints other than colic at 1 month of age that has health consequences resulting from childhood exposure to
resolved. On recent physical examination, the baby ap- trauma and violence.
peared well, with normal growth and development, and
the mother did not raise any concerns during the visit.
EPIDEMIOLOGY
The primary pediatrician now notes that she documented
a small circular bruise on the baby’s chest that the mother In 2014, over 3.5 million children were subjects of child
stated occurred when a 3-year-old sibling hit the baby with a maltreatment reports. Of those, 702,000 children (20%)
toy. Having had a longstanding relationship with this were found to have evidence of maltreatment. (11) This
mother and family, she accepted this explanation for the translates to an annual victimization rate of 9.4 children
bruise. per 1,000 in the United States and a prevalence rate of 1 in
After reviewing the chart, she explores the current 8 children by age 18 years. (12) Neglect is the most
literature and management of suspected child physical common form of child maltreatment, constituting 75%
abuse, including the American Academy of Pediatrics of indicated reports; 7% are attributable to physical abuse.
clinical report on evaluation of suspected child physical In 80% of child physical abuse cases, a biological parent is
abuse. (7) She now understands that the bruise she noted the perpetrator. Children in their first postnatal year have
on examination was a sentinel injury that should have the highest victimization rate (24.4 per 1,000), and chil-
prompted further evaluation. As a result of the case, her dren younger than age 3 years have the highest fatality
practice group plans to review and implement guidelines rate, comprising over 70% of the nationally estimated
for the identification and evaluation of children present- 1,580 child maltreatment deaths in 2014. Child welfare
ing with signs or symptoms concerning for physical data and trends, however, are dubious because of a lack of
abuse. standardized terminology and differences in report and
response types across states.

INTRODUCTION
RISK FACTORS FOR CHILD PHYSICAL ABUSE
Child physical abuse is a difficult diagnosis to entertain
primarily because clinicians are hesitant to accept that Risk factors for abuse are commonly categorized into
caretakers can injure children. The diagnosis is further parental, child, and social characteristics. Identification of
complicated by the reality that caretakers rarely disclose risk factors aids in the assessment of abuse but more
maltreatment, preverbal or obtunded children cannot provide importantly aids in the ability to counsel parents and
a history, and signs and symptoms of physical abuse may be develop preventive strategies. Risk factors are not, in
subtle and confused with other common pediatric diagnoses. and of themselves, diagnostic. Many families have risk

Vol. 37 No. 4 APRIL 2016 147


factors and never abuse their children, while others have As children become mobile, the incidence of expected
no apparent risk factors and do abuse their children. Child accidental trauma increases, and common childhood
abuse does not discriminate; it affects children of all ages, injuries such as bruises over bony prominences and
socioeconomic classes, and ethnic groups. toddler’s, clavicular, and skull fractures are seen. In
Parental/household risk factors include substance contrast to children with abusive injury, witnesses often
abuse, mental illness, interpersonal violence (IPV), single corroborate accidental injuries in ambulatory children,
and/or teen parent, and a nonrelated adult in the home. caregivers seek timely care, they provide a consistent
Among the social risk factors are social isolation, poverty, history, and the mechanism described explains the injury
lower levels of education, and large family size. Child- observed. Because the incidence of child physical abuse is
related risk factors include prematurity, low birthweight, highest in children younger than age 4 years, the clinician
intrauterine drug exposure, and developmental and phys- must have a high index of suspicion and add abusive
ical disabilities. The most significant risk factor for abuse is trauma to the differential diagnosis of the ill-appearing
the age of the child, with infants and toddlers being at young child.
greatest risk for serious and fatal child physical abuse. Determining which injured children require an evalua-
A clear association exists between particular developmen- tion for child physical abuse should account for the age and
tal stages and physically abusive injuries, such as excessive developmental ability of the child, the injury sustained, the
crying and abusive head trauma or toilet training and adequacy of the historical explanation provided, and
inflicted scald burns. Awareness of these developmental
triggers should guide anticipatory guidance, with the poten-
tial for preventing an abusive injury.
TABLE 1. Criteria for Consideration to Initiate a
IPV is a substantial risk factor for child abuse, and each Child Physical Abuse Assessment
health supervision visit should include IPV screening. Age and Development
Exposure to violence itself, even if the child is not physically
• Nonmobile infant with any injury
harmed, has significant and long-lasting effects.
• Injury in nonverbal child
• Injury inconsistent with child’s ability
WHEN TO CONSIDER THE DIAGNOSIS OF CHILD
• Statement of harm from a verbal child
PHYSICAL ABUSE
Injury
Injuries are common in childhood. Although most child-
• Any injury in a nonmobile infant
hood injuries are accidental, the clinician must appreciate
that almost any injury can be abusive. With the exception of • Uncommon in age group
patterned marks, very few injuries are pathognomonic for • Occult finding
abuse. In the nonverbal child, injuries may be apparent or • Mechanism not plausible
covert; many children present with nonspecific symptoms
• Multiple injuries, including involvement of multiple organs
and a lack of history. Child physical abuse should be
entertained in any infant displaying signs or symptoms • Injuries of differing ages

potentially explained by trauma, such as irritability, lethargy, • Pattern of increasing frequency or severity of injury over time
vomiting, apnea, seizures, or coma. • Patterned cutaneous lesions
Several studies of abused children have demonstrated
• Bruises to torso, ear, or neck in child younger than age 4 years
that antecedent sentinel injuries, such as bruises, intraoral
• Burns to genitalia, stocking or glove distribution, branding, or pattern
lesions, and skeletal trauma, were noted by medical pro-
fessionals or caregivers before a subsequent abusive act, History
while children presenting with accidental injuries were not • Chief complaint does not contain caregiver concern for an injury
found to have sentinel injuries. (1)(2)(3)(4)(5)(6) Because and plausible history

infants are essentially nonmobile and nonweight-bearing, • Caretaker response not commensurate to injury
they should never have bruising. Therefore, any injury in • Unexplained delay in seeking care
an infant must be viewed as significant and descriptive
• Lack of, inconsistent, or changing history
language such as “minor” should not be used. Identifying
• Inconsistencies or discrepancies in histories provided by
a sentinel injury with appropriate evaluation of the child may
involved caretakers
be lifesaving.

148 Pediatrics in Review


clinical findings (Table 1). Fundamentally, when injuries histories should be obtained. A complete social history
are not explained or historical data provided contain incon- identifies risk factors for maltreatment, and a family med-
sistencies or insufficiencies, a child abuse evaluation is ical history focusing on illnesses such as bone disease or
warranted. Any child younger than age 2 years who pre- bleeding tendencies allows for screening and identification
sents with a suspicious injury should have a skeletal of possible underlying medical problems in the patient.
survey. Other studies should be obtained based upon
clinical concern and findings. Negative studies do not rule
PHYSICAL EXAMINATION AND DIAGNOSTIC
out child abuse.
EVALUATION

A thorough and well-documented physical examination of


HISTORY OF THE PRESENT ILLNESS AND CHILD
any child with concerns for possible child abuse is imper-
PHYSICAL ABUSE
ative. The clinician should be aware that children may suffer
A thorough history of present illness is the single most more than one type of abuse; the physically abused child
useful piece of information to aid the clinician in making a may also be neglected or sexually abused. The child’s mental
correct diagnosis. The detailed history should be obtained status, affect, and level of activity should be noted. The child
in separate interviews with each caregiver, the child (if must be undressed and all skin surfaces examined with
possible), verbal siblings, and any other persons in the good lighting. The entire body must be evaluated, including
household. Interviews should be conducted such that each areas that may be overlooked, such as the pinnae, behind the
parent or caregiver can give a history in his or her own ears, the oral cavity including the teeth and frenula, the soles
words. He or she should be allowed to provide the entire and palms, the genitals, and the anus. Every cutaneous
history without interruption, decreasing the chance that injury should be described according to color, shape, size,
the interviewer unintentionally redirects or suggests a and location. Photographic documentation or drawings
mechanism. Details about the mechanism of injury, the should be completed and placed in the medical record.
events leading up to the injury, and whether the injury was The presence or absence of swelling and the ability to move
witnessed or unwitnessed should be elicited. For example, limbs should be noted. Paradoxical comfort (a baby who is
in injuries related to falls, having parents recreate the more comfortable when not being held but cries when
scene, describing the height of furniture, flooring, and picked up) may be observed in infants with occult injuries
the position of the child before and after the fall, is such as rib fractures. An assessment of the child’s nutri-
essential. tional status, including completion of a growth chart, is
A history of the onset and progression of symptoms crucial because neglect, malnutrition, and failure to thrive
since the child last appeared well should be obtained. may be comorbidities with physical abuse.
Determining who was caring for the child and asking each The diagnostic evaluation of suspected physical abuse
of the caretakers how the child appeared by focusing on should always be driven by the history, physical examination,
descriptions of activity and movement (particularly during and differential diagnosis. Clinicians must consider the pos-
feeding, bathing, and diaper changing) can aid in deter- sibility that multiple types of trauma may coexist and recognize
mining when a child may have been injured. For infants that injuries may be occult. Any nonverbal and nonambulatory
with intracranial injury, it may be difficult to develop a child with an injury should have a standard child abuse
timeline of when the child was last well because the infant evaluation (Table 2) no matter how “minor” the injury. The
may be thought of as “well-appearing” while asleep when most prudent approach is to rule out skeletal trauma in all
the child actually may be seriously injured. Important children younger than 2 years of age with a standard skeletal
features of the history that should raise concern for an survey and assess for occult central and/or internal injuries
abusive injury include: no history of trauma; a history of by choosing appropriate imaging and laboratory studies
trauma inconsistent with the severity, pattern, or timing of (Table 3).
the injury; injury inconsistent with the developmental
capabilities of the child; multiple or evolving histories;
ABUSIVE HEAD TRAUMA
discrepant histories from the same caregiver or between
caregivers; injury attributed to a sibling or pet; and a delay Abusive head trauma (AHT) has the highest mortality of
in seeking medical care. all forms of child physical abuse, with an estimated
In addition to a detailed history of the incident, the fatality rate greater than 20%. Survivors have irreversible
patient’s birth, past medical, developmental, and dietary sequelae of brain injury, ranging from minor behavioral

Vol. 37 No. 4 APRIL 2016 149


TABLE 2. Protocol for the Evaluation of Suspected Child Physical Abuse
History of Present Illness
• Interview primary caretakers separately; note historian’s ability to provide history
• Ask caretakers about age-appropriate developmental abilities of child. Observe child if possible
• Develop a timeline from when the child was last agreed upon to be in his or her usual state of good health and note the following:

B Onset of symptoms and progression


B The patient’s observed mental status and activity level. Ask specifically about how the child appeared at time of hand off between
caretakers
• Note if there were any witnesses, photos taken of child, or other corroborating information

Social History
• List all adults having access to the child, including age, relationship, and contact information
• List all children, including age and relationship; identify in which home they reside
• Note history of drug or alcohol abuse, intimate partner violence, mental illness, prior history of involvement with child protective services

Relevant Past Medical History


• Skeletal trauma: child or family history of bone disease, diet history
• Abusive head trauma (AHT) and cutaneous injuries: child or family history of bleeding diathesis, eg, prolonged bleeding after circumcision,
umbilical cord removal, or surgery or as a result of past injuries
Physical Examination
• Examine closely for possible intraoral injuries such as frenulum tears; explore all unexposed surfaces: behind ears, genital region, and bottoms of feet
• Growth chart: obtain prior growth data, and with regard to AHT, note trajectory of head circumferences

Photodocumentation
• If photos are obtained, document in the medical record details of the photos taken, including location of injuries, number of photos taken,
date, and photographer
• If photodocumentation is unavailable, use a body diagram noting all cutaneous lesions by size, location, and color

Evaluation
• Indicated laboratory and imaging studies for current illness or injury
• Studies to assess occult injuries, such as skeletal survey
• Communication with appropriate subspecialists regarding findings and treatment, including child abuse pediatricians when appropriate for
referral and consultation
Mandated Reporting and Safety
• Develop dialogue to inform parents about mandated reporting, safety, and reason for report
• Ensure that forms and phone numbers for reporting are accessible
• Establish office process for specific scenarios with regard to obtaining imaging and laboratory studies and process for transfer to appropriate
facility for evaluation and treatment, including protocol for accessing expertise of child abuse pediatrician
• Facilitate thorough sibling assessment, including appropriate imaging, laboratory studies, and interpretation; establish protocol to ensure
results of sibling assessments are communicated to others in the investigation, including primary care clinician
• Ensure medical record and photodocumentation accessibility for investigators (consent not required after report to child welfare)
• Discuss disposition of, medical follow-up, and supportive services for patient with child welfare case worker

issues and neurodevelopmental delays to significant neu- occurs most often in children younger than age 2 years
rodevelopmental delays, seizures, blindness, and paraly- and crying is the most commonly identified trigger.
sis. (13) The incidence of AHT is 15 to 30 cases per Recognizing that the phrase “shaken baby syndrome”
100,000 infants annually in the United States. AHT implies a specific mechanism, in 2009 the American

150 Pediatrics in Review


Academy of Pediatrics (AAP) recommended that AHT patient is stable, ideally a few days after admission, to
replace this terminology to acknowledge that multiple optimize visualization of the parenchyma and evaluate for
mechanisms, either separately or together, can cause edema, stroke, and thromboses. MRI can also elucidate the
calvarial, brain, and cervical injuries. (14) location of extra-axial fluid collections and aid in the aging of
Infants and young children who have AHT can present intracranial hemorrhages. MRI imaging of the spine is also
with signs and symptoms ranging from mild to life- indicated because studies have now demonstrated injury to
threatening, with a clinical spectrum that includes irritabi- the cervical spine, such as ligamentous injury and spinal
lity, vomiting, lethargy, seizures, apnea, coma, and death. subdural hemorrhage, in children with AHT. (16)(17) MRI is
Often there are no external findings suggestive of trauma also preferred over CT scan when clinical findings such as
and the history is lacking or misleading. Thus, depending on rapidly increasing head circumference or focal neurologic
the extent and severity of the injuries, traumatic brain injury issues suggest remote injury. MRI does not entail radiation
is often misdiagnosed as colic, viral syndrome, otitis media, exposure, but it is a longer study that most often requires
gastroenteritis, gastroesophageal reflux, or pyloric stenosis. sedation.
Clinicians must keep AHT in their differential diagnoses Retinal injuries such as hemorrhages, schisis or tearing,
and have a high index of suspicion to obtain a thorough and folds are associated with AHT and may be seen in up
history and perform the appropriate diagnostic tests. to 80% of cases. Hemorrhagic retinopathy from AHT is
Brain injuries seen in 80% of AHT cases include sub- classically described as multilayered, with hemorrhages
dural hemorrhage that is interhemispheric, posterior, that are too numerous to count and extend to the ora
often layering over the tentorium, and/or a thin subdural serrata. This very specific finding is unique to AHT and
layer over either or both of the convexities. Mass effect is not due to increased intracranial pressure, blunt head
results not from the subdural trauma itself but rather from trauma, or cardiopulmonary resuscitation. Retinoschisis
significant cerebral edema. The parenchymal damage and macular folds are reported almost exclusively in chil-
evolves into a clinical picture consistent with hypoxic- dren who have sustained violent craniorotational injury
ischemic encephalopathy. Although additional injuries and are specific to this mechanism. Any infant or child who
need not be present to diagnose AHT, these neurologic has intracranial injuries suspicious for abuse should be
injuries are frequently associated with other traumatic evaluated by an ophthalmologist who can meticulously and
findings, such as retinal hemorrhages, posterior rib frac- precisely document the ocular findings, preferably with
tures, and classic metaphyseal lesions (CMLs). Bruising to use of photo imaging. The rate of healing varies from days
the scalp or other parts of the body may or may not be present. to weeks and aging of retinal hemorrhages is imprecise. Of
The clinician must be mindful to ensure a thorough eval- note, an ophthalmologic examination is not a screening
uation for other occult injuries, including neck, internal, tool for AHT but is indicated when there is evidence of
and other skeletal trauma. intracranial injury.
A short fall leading to fatal head trauma is exceptionally
rare, with a calculated risk of less than 1 per 1,000,000
CUTANEOUS INJURIES
children annually. (15) A unique situation is the develop-
ment of an epidural hematoma after minor blunt trauma in The skin is the most frequently injured organ in child
which a temporal linear skull fracture may sever the middle abuse, with bruises, bites, and burns accounting for many
meningeal vessels and lead to an accumulation of blood that child maltreatment injuries. Although cutaneous injuries
results in mass effect. This is one circumstance in which a are very common in childhood, they are rare in the
child may be neurologically intact after minor or trivial preambulatory child: “those who don’t cruise don’t
trauma but experience deteriorating mental status and acute bruise.” (18)(19) Considerable data support that bruising
symptoms as a result of mass effect. is not only extremely uncommon in infants but highly
Head and neck imaging must be obtained for any child correlated with child abuse. (20)(21) Thirty percent or
for whom there are concerns for AHT. A CT scan is the more of seriously injured or fatally abused children have
initial imaging modality of choice because it can be per- been noted to have bruises, which are sentinel signs
formed quickly in the critically ill child. However, CT scan (Figure 1), reported on physical examination before sub-
does not reveal parenchymal injuries, cannot reliably dif- sequent severe or fatal abuse. These data support the
ferentiate between subdural and subarachnoid collections, directive that any nonmobile infant who has a bruise
and involves substantial radiation exposure. Magnetic res- must receive a full child abuse evaluation (Table 2) and a
onance imaging (MRI) should be performed once the report to child welfare for investigation. (3)(4)(5)(6)

Vol. 37 No. 4 APRIL 2016 151


TABLE 3. Child Physical Abuse Medical Evaluation: Imaging and
Laboratory Studies
STANDARD CHILD PHYSICAL ABUSE MEDICAL EVALUATIONS
Skeletal Injuries
• Skeletal survey (with views according to the collaborative practice parameter issued by the American College of Radiology and the Society for
Pediatric Radiology)
• Follow-up skeletal survey is indicated in 2 weeks when abuse is suspected on clinical grounds and/or initial findings are abnormal or
equivocal
• Core laboratory studies for bone health: calcium, magnesium, phosphate, and alkaline phosphatase
• If concerns for vitamin D deficiency (elevated alkaline phosphate, abnormal bone density, or dietary concerns), consider 25-hydroxyvitamin D
and parathyroid hormone level
Central Imaging
• Head computed tomography (CT) scan (useful for screening, and/or monitoring an ill child)
• Magnetic resonance imaging (MRI) of head and spine (useful for elucidating extra-axial spaces, parenchymal disease, and spinal injury)

Routine Trauma Laboratory Tests


• Hematologic: complete blood cell count and platelets
• Coagulation: international normalized ratio, prothrombin time, and activated partial thromboplastin time
• Metabolic: glucose, blood urea nitrogen, creatinine, calcium, magnesium, phosphate, albumin, and protein
• Urinalysis: urine toxicology screen, order myoglobin if urinalysis positive for blood and red cells are not seen on smear
• Liver function tests: aspartate aminotransferase and alanine aminotransferase (>80 U/L [1.34 mkat/L] is concerning for occult injury)
• Pancreatic enzymes: amylase and lipase

ADDITIONAL POTENTIAL TESTS


Ophthalmologic Examination
• Indicated if evidence of either acute or remote central nervous system trauma
• Not a screening tool for abusive head trauma

Abdominal (Thoracoabdominal) Imaging: CT Scan With Intravenous Contrast


• Elevated liver or pancreatic enzyme values
• Comatose patient
• Evidence of trauma with delay in care (liver function tests may have decreased to normal levels)

Concerns for Bleeding Diathesis (Family History or Clinical Concerns)


• von Willebrand antigen, von Willebrand activity (ristocetin co-factor), Factor VIII, Factor IX, platelet function assay
• Hematology consultation

Metabolic Diseases
• Genetics consultation

As children start to ambulate, the incidence of bruising “TEN 4” is useful to recall which bruise locations are
increases. Bruise location and morphology are important concerning for abuse: Torso, Ear, Neck, and 4 signifying
factors to consider when assessing for child physical abuse children younger than age 4 years and any bruising noted in
in ambulatory children. Accidental injuries tend to occur infants younger than 4 months. (20) Bruising and abrasions
over bony prominences (shins and elbows) in contrast to that occur on more than one body surface, are in multiple
bruises due to abuse, which are located on the face, head, stages of healing, and are patterned or well demarcated are
neck, torso, flanks, buttocks, and thighs. The mnemonic more likely to be the result of abuse. Patterned injuries

152 Pediatrics in Review


reflect the shape of the instrument, such as loop marks dialogue with parents that promotes alternative methods of
from a cord or cable, linear bruises from belts, or multiple discipline.
parallel linear bruises equally distributed from a slap with a Bite marks are another patterned skin injury noted in
hand. Contrary to some common beliefs, children do not abused children. Clinicians can discern between animal and
bruise more easily than adults and bruises cannot be aged human bites by assessing the shape: animal bite marks are
precisely. The appearance of a bruise is related to many puncture wounds with a sharply angulated arch, while
factors, including the state of hemoglobin degradation, the human bite marks are crush injuries consisting of an ovoid
color of skin pigment, the depth of the bruise, the location pattern of tooth marks that may surround an area of central
on the body, the lighting in the room, and the patient’s bruising. In general, adult bite marks measure greater than
metabolism and circulation. Bruises of differing colors do 2 cm between the maxillary canines. Consultation with a
not signify different times or incidents. Finally, because forensic odontologist may assist in the evaluation of well-
bruises or soft-tissue injury may be painful for days, the demarcated bite marks. Multiple bites on different body
presence of tenderness does not necessarily mean the planes, bites on soft-tissue areas, and bites on areas gener-
injury is acute. ally covered by clothing should raise a suspicion of abuse.
Determining whether marks or bruising from corporal Bites to the genitalia, buttocks, and/or breasts should raise a
punishment constitutes abuse is a difficult task. Legally, concern of possible sexual abuse. Acute bites to the genitalia,
some states condone corporal punishment as an acceptable buttocks, and/or breasts may warrant collection of forensic
form of behavioral modification while others define it as a evidence for DNA by swabbing the area with a cotton swab
form of child maltreatment and require reporting to child moistened with distilled water.
welfare. The AAP and the American Academy of Child and Hot liquid, grease, steam, hot objects, chemicals, elec-
Adolescent Psychiatry do not condone corporal punish- tricity, or microwave ovens may cause abusive burns. Com-
ment due to its limited effectiveness and potential delete- pared to accidental burns, abusive burns are more severe,
rious effects. Although known to be immediately effective, more likely to be full-thickness, and require more extensive
spanking and corporal punishments have significant treatment, including grafting. Children who are abusively
adverse outcomes, such as increased aggression and burned are most often younger than 4 years and inflicted
decreased development of appropriate behavior. (22)(23) immersion burns to the buttocks and genitalia are com-
(24) Both groups advise against the use of corporal pun- monly associated with toilet training.
ishment and encourage alternative methods of behavioral Abusive burns most often take the form of immersion
modification – such as time out, loss of privileges, positive scald burns, characterized by well-demarcated areas of
reinforcement, and opportunities for positive touch like confluent depth with no splash or cascading flow pattern.
hand holding and hugging – that have healthier, long-lasting Immersion burns may involve the buttocks, perineum,
effects. From a practical standpoint, each clinician must be extremities, hands, or feet. Circumferential burns affect-
versed in his or her state laws. More importantly, the ing the feet and/or hands are sometimes referred to as
clinician must develop a thoughtful and culturally sensitive having a “stocking” or “glove” distribution. In immersion
burns, the position in which the child was held may be
surmised by the burn pattern and depth. If the child’s
buttocks come into contact with the tub surface, a “dough-
nut” type pattern may be noted with relative sparing of the
part of the anatomy coming into contact with the tub. Sparing
of the flexion creases is often observed. Persons who inflict
these burns generally do not suffer burns themselves. A
careful scene reconstruction and investigation, including
water temperature, may help determine the length of time
the child was held in the water. Generally, the hotter the water,
the shorter the duration of submersion. Partial-thickness
burns develop in minutes at 48.9°C (120°F) but take mere
seconds at 65.6°C (150°F). (25)
Children often come into contact with hot objects, such
Figure 1. Infant displaying bruising that represents a sentinel sign for
as irons, hair tools, radiators, and stovetops. Resultant burns
physical abuse. are related to the heat of the object and period of contact with

Vol. 37 No. 4 APRIL 2016 153


the skin. Both abusive and accidental contact burns can bones, resulting from twisting that causes a planar fracture
result in a patterned mark, making discernment based on through the spongiosum of the metaphysis.
appearance difficult. Suspicious contact burns require a Some uncommon fractures, such as sternal, spinal, and
thorough scene investigation and corroboration, particularly scapular fractures, are also highly suggestive of child abuse
if the child is nonverbal. In general, abusive contact burns in the absence of a credible and corroborated trauma history.
are deeper and leave a clear imprint while those that result Any child who presents with multiple fractures or fractures
from grazing against a hot object are not as deep or well- in differing stages of healing should raise concern for
demarcated. nonaccidental trauma.
The most common differential diagnosis of inflicted The most common differential diagnosis of abusive
cutaneous injury is accidental injury. Dermatologic and fracture is accidental fracture. Some fractures, such as
other conditions such as congenital dermal melanocyto- clavicular fractures, may be due to birth trauma. Underlying
sis (mongolian spots), phytophotodermatitis, Henoch- medical conditions and metabolic bone diseases should be
Schönlein purpura, Ehlers-Danlos syndrome, vasculitis considered in the differential diagnosis of skeletal trauma
syndromes, bleeding dyscrasias, eczema, malignancy, and and include osteogenesis imperfecta, Menkes syndrome,
cultural practices such as cupping and coining (cao gio or hyperparathyroidism, hypophosphatasia, and Fanconi syn-
gua sha) may lead to cutaneous lesions that appear initially drome. Although vitamin D deficiency is prevalent, rickets
concerning for abuse. Some conditions that may be con- is not, and research demonstrates that the incidence of
fused with or mimic burns are impetigo, staphylococcal fractures in skeletal trauma is not influenced by vitamin
scalded skin syndrome, herpes, and contact dermatitis. A D deficiency. (27)(28)
careful history, physical examination, and diagnostic tests
should clarify whether an accident or one of these condi-
IMAGING OF SKELETAL INJURIES
tions is the cause of the findings. As always, clinicians
should be mindful that the child who has an underlying Skeletal injuries may be clinically silent in children younger
medical problem might also be physically abused. than age 2 years and in developmentally delayed or non-
verbal children. A skeletal survey to identify acute or healing
fractures should be performed when there is concern for
SKELETAL INJURIES
any form of child physical abuse. If an abused child has
Fractures are the second most common type of child siblings who are younger than 2 years, skeletal surveys
physical abuse. Accidental fractures are common in ambu- should be performed to evaluate these siblings. The current
latory children but not in nonambulatory children. Most recommendation by the American College of Radiology for
abusive fractures occur in nonambulatory children, rep- skeletal survey consists of 21 dedicated views, including
resenting 55% to 70% of fractures in children younger than oblique views of the chest to aid in the detection of rib
age 1 year and 80% of all abuse fractures found in children fractures. (29) Although the clinician must be mindful of
younger than 18 months of age. (26) Age is the single most radiation exposure, the discovery of an occult injury is a
important risk factor for abusive skeletal injury. major determinant in the diagnosis of child abuse and, thus,
Understanding that different types of fractures result outweighs radiation risk. Infants may have positive skeletal
from different forces applied to the bone aids in determin- surveys in up to 20% of cases. (30)
ing if a given history is plausible. Transverse fractures are Some fractures with high specificity for child abuse, such
due to forces that are perpendicular to the bone or bend the as rib fractures and CMLs, may not be apparent on the initial
bone, torus or buckle fractures are due to axial loading or skeletal survey. With healing and new bone formation, these
compression along the long axis of the bone, spiral fractures injuries become more obvious. Thus, a repeat skeletal survey
are due to twisting, and oblique fractures are due to a obtained 10 to 14 days after the initial survey is recommended
combination of transverse and twisting forces. to re-evaluate for fractures. Bone scans are no longer obtained
Abusive skeletal injury may involve any part of the as a complementary test to detect possible injury because of
skeleton, but fractures of the extremities are most common. their serious limitations, including problems with motion
Any fracture can result from abuse, and no fracture is artifact, inability to estimate age of injury, and lack of spec-
pathognomonic for abuse. Some fractures, however, have ificity. Due to radiation exposure concerns, CT scan is not a
a higher specificity for abuse, such as posterior or lateral preferred imaging modality to detect fractures. If chest or
rib fractures or CMLs, also known as “corner” or “bucket abdominal CT scan is performed for other reasons, it can be
handle” fractures. These fractures occur at the ends of long useful in detecting nondisplaced rib fractures. Skull fractures

154 Pediatrics in Review


in the line of the axial plane are often missed on head CTscan. liver and spleen, followed by duodenal and proximal jejunal
The addition of 3-dimensional CT scan reconstruction ruptures or hematomas and pancreatic, vascular, and/or
enhances the identification and morphology of skull fractures renal trauma. Other injuries may involve the bladder, large
and helps ensure their detection and diagnosis. intestine, stomach, mesentery, and adrenals. The most com-
Aging of bone fractures is imprecise and must be based mon mechanism for abdominal injury is focused blunt force
upon history and clinical examination findings as well as trauma to the abdomen that compresses, crushes, or tears
radiographic known patterns of healing. In young children the viscera. Because the forces are focused internally, abdom-
with long bone fractures, new bone is visible within 1 to 2 inal bruising is rare, and clinicians should not be lulled into a
weeks, followed by callous development, disappearance of false sense of security by the absence of abdominal bruising.
the fracture line, and finally resolution. Some fractures, such The major diagnostic differential diagnosis for abusive
as skull fractures or CMLs, do not follow a predictable healing abdominal injury is accidental abdominal injury, which is
process and cannot be aged based on radiographs alone. readily elucidated by history (or lack thereof).
Any child in whom abdominal trauma is suspected
should undergo screening, including liver function tests,
CHEST AND ABDOMINAL INJURIES
pancreatic enzymes, hemoglobin to assess for blood loss,
Abdominal trauma is the second leading cause of fatalities and urinalysis to check for hematuria. Plain radiographs are
due to child physical abuse. This is likely due to delays in rarely diagnostic but may reveal indirect evidence of visceral
seeking medical care, a misleading history of no or only injury, such as dilated loops of bowel, air fluid levels, free air,
trivial trauma, and the greater severity of injury in abusive as bowel wall air, periportal tracking, or mass effect on the
compared to accidental abdominal trauma. Early signs and bowel. Abdominal CT scan with contrast is the preferred
symptoms, such as loss of appetite, vomiting, and abdom- imaging modality to assess for intra-abdominal injury and
inal pain, are nonspecific and may be misdiagnosed. Fur- is indicated when hepatic enzymes aspartate aminotrans-
thermore, physical examination generally does not reveal ferase and alanine aminotransferase measure more than
bruising to the chest or abdomen. Some children with 80 U/L (1.34 mkat/L). (31) Liver enzyme values acutely rise
abdominal trauma may be battered and have “distracting but then rapidly fall after blunt trauma; liver enzymes due to
injuries,” such as AHT or a fracture, that may delay recog- infection or liver disease do not resolve in this pattern. In
nition of the abdominal injury. Because signs and symp- children who have subacute abdominal trauma, liver enzymes
toms of chest and abdominal injuries may be subtle or may have normalized by the time of evaluation. (32)
overlooked, meticulous physical examination is imperative. Thoracic injuries are mostly related to crush or major
Signs and symptoms of occult abdominal trauma vary, blunt trauma and may involve the heart, lungs, rib cage, and
depending on the age of child and presence of associated mediastinum. Pulmonary hemorrhage and edema due to
injuries such as intra-abdominal hemorrhage or peritonitis. airway obstruction, shearing injury to the thoracic duct
The clinician should palpate the chest wall to examine for rib resulting in a chylothorax, pneumomediastinum, pneumo-
deformities. Chest wall tenderness and/or palpable callus pericardium, and commotio cordis have all been described
may suggest the presence of healing rib fractures. Auscul- as a result of abusive trauma.
tation for equal breath sounds as well as clear heart tones
and bowel sounds and palpation of the abdomen must be
REPORTING TO CHILD WELFARE AND THE ROLE OF THE
performed. Because the physical examination results can be
CLINICIAN
misleading, with a lower sensitivity for trauma in younger
children, laboratory screening should be considered, par- As mandated reporters, clinicians are required to make a
ticularly in any nonverbal or nonambulatory child. report to child welfare when there is reasonable cause to
Relative to accidental abdominal trauma, abusive abdom- believe that an injury is the result of abuse or neglect. The
inal trauma occurs more often in infants and toddlers, is clinician need only suspect that maltreatment has occurred
more severe, and requires a higher rate of surgical inter- to initiate a report. The clinician’s role is to initiate evalu-
vention. The peak age for abusive abdominal injury is ation and ensure appropriate treatment and safety of the
between ages 2 and 3 years. This may be due to child-related patient. The final determination of child abuse requires a
behavioral risk factors, such as increased activity level, coordinated interdisciplinary investigation. Access to the
normal exploration, normal negativism, and toilet training. expertise of a child abuse specialist is ideal because he or she
Abusive injuries to almost every visceral structure have is accustomed to collaborating with law enforcement, child
been reported. The most commonly injured organs are the welfare, and the legal system.

Vol. 37 No. 4 APRIL 2016 155


Primary care clinicians may be challenged by their significant contemporary source of morbidity and mor-
responsibility to inform the caregiver or parent(s) of a report tality. Appreciation of the adverse impacts of early stres-
suspicious for child maltreatment. They are understandably sors on adult health has been transformational in
concerned about the safety and disposition of their young validating the role of the pediatric clinician in promoting
patients as well as the potential stress and reaction of the wellness not only in childhood but also into adulthood.
family to an investigation. Anticipating a negative family Knowledge of child physical abuse continues to evolve,
reaction may cause hesitation in reporting. However, the providing more clarity, as demonstrated by the recent
clinician must keep in mind his or her role as the child’s understanding of the significance of sentinel injuries.
advocate and recognize that the report is a medical inter- Increased awareness of child maltreatment by primary
vention that may be lifesaving. Developing a thoughtful care clinicians along with timely intervention ideally can
dialogue to inform a parent of the need to make a lead to effective prevention of the adverse outcomes of
report can ease the stress on the reporter as well as the child maltreatment and, along with more dedicated research,
parent. to effective primary prevention.
Clinicians often wonder what happens after a report is
made. In every state, a child welfare organization is
responsible for investigation of the child maltreatment Summary
allegation. The investigative agency may substantiate • On the basis of research and consensus, the diagnosis of child
(indicate) or not substantiate (unfound) a particular alle- physical abuse must be entertained whenever an infant or
nonverbal child presents with any injury. Substantial evidence
gation. Unsubstantiated findings do not necessarily mean
supports that any form of trauma in a baby is significant and
that child maltreatment did not occur. Most investigations
deserves complete evaluation. (1)(2)(3)(4)(5)(6)
do not result in removal of a child from a home and may, in
• Clinicians must consider child abuse in the differential diagnosis
fact, provide opportunities for augmenting the family by of any young child with injuries or symptoms where there are
offering support to caregivers, such as home visits, par- discrepancies between the sustained injuries and the history and/
enting classes, access to transportation for follow-up care, or patient’s developmental capabilities. On the basis of strong
and individual or group therapy. research and consensus, child abuse is recognized not only as a
major source of mortality and morbidity in childhood but also as a
Some cases may progress in the legal system. When there
direct cause of increased adult morbidity and early death. (9)(10)
is urgent and immediate concern for a child’s safety, the case
• On the basis of consensus, primary care clinicians are in a position
may be heard in juvenile/family court and result in the
to identify children with injuries concerning for child abuse, initiate
temporary removal of a child to ensure his or her safety, an appropriate and thoughtful medical evaluation, report to child
health, and well-being. These children are assigned their welfare, and appropriately seek child abuse pediatric consultation.
own attorney, known as a guardian ad litem.
Criminal court testimony is requested when a specific
person has been charged with an act leading to an abusive CME quiz and references for this article are at http://pedsinre-
injury. Unlike juvenile court, in which evidence is based view.aappublications.org/content/37/4/146.
upon preponderance (more likely than not), criminal court
requires evidence beyond a reasonable doubt. “Managing
Child Abuse: General Principles” is an excellent reference To view PowerPoint slides that accompany this article,
(33) that provides a very thoughtful stepwise approach to visit http://pedsinreview.aappublications.org
reporting and navigating the subsequent legal process. and click on the Supplemental tab for this article.
No matter which direction a report may take, the clinician
must advocate for his or her patient to have necessary
resources and an appropriate medical home.

PREVENTION AND FUTURE HORIZONS

The monetary cost to society of child abuse has been


estimated to be $80.3 billion per year. (34) However, this
does not adequately reflect the true loss of a child due to
untimely and preventable death. Child maltreatment is
now recognized as a major public health problem and a

156 Pediatrics in Review


PIR Quiz
There are two ways to access the journal CME quizzes:
1. Individual CME quizzes are available via a handy blue CME link under the article title in the Table of Contents of any issue.
2. To access all CME articles, click “Journal CME” from Gateway’s orange main menu or go directly to: http://www.
aappublications.org/content/journal-cme.

1. A 14-month-old girl is brought to the emergency department with history of a fall from a REQUIREMENTS: Learners
couch to a tile floor. Her mother and father have accompanied her. The mother believes can take Pediatrics in
the fall was about 2.5 feet. She reports that the girl cried immediately and after a few Review quizzes and claim
minutes, she seemed to act in a typical manner. She also reports that after about 15 credit online only at:
minutes the girl started vomiting and was sleepy. You obtain a computed tomography http://pedsinreview.org.
scan of the brain that shows a 9-mm right-sided epidural hematoma with mass effect.
What is the most appropriate next step in management?
To successfully complete
A. Obtain social work consultation. 2016 Pediatrics in Review
B. Request consultation from the child abuse specialist. articles for AMA PRA
C. Request magnetic resonance imaging (MRI) of the brain. Category 1 CreditTM,
D. Request a neurosurgical consultation. learners must
E. Separate the parents and obtain histories from each of them. demonstrate a minimum
2. You see a 3-month-old boy for a health supervision visit. His mother reports that he spits up performance level of 60%
after most feedings. He has eczema over his face, arms, and chest. He has a nickel-sized or higher on this
bruise behind his left ear. His mother reports that he rolled onto a toy in his crib and this assessment, which
caused the bruise. What is the most appropriate next step in management? measures achievement of
A. Obtain complete blood cell count and iron levels. the educational purpose
B. Request MRI of the brain. and/or objectives of this
C. Request referral for a dermatology evaluation. activity. If you score less
D. Request referral to a gastroenterologist. than 60% on the
E. Submit a report to child welfare. assessment, you will be
given additional
3. A 1-year-old boy has bruising over his back and upper arms with several parallel lines of
opportunities to answer
bruising on both upper arms. Several of the bruises are green; other bruises are yellow and
questions until an overall
purple. You are asked to testify in court regarding the cause and timing of these injuries.
60% or greater score is
You testify that:
achieved.
A. Children bruise more easily than adults.
B. The parallel lines of bruising likely represent a hand print injury.
C. These sites of bruising are common in accidental injuries. This journal-based CME
D. Varied colors of bruising show that there were injuries at varied times. activity is available
E. You are unable to provide any information related to the boy’s bruises. through Dec. 31, 2018,
however, credit will be
4. You see a 2-year-old boy with vomiting and weight loss over the past several weeks. His
recorded in the year in
father relates that several other family members have had gastrointestinal illness in the
which the learner
past month. On physical examination, the boy’s abdomen is mildly distended with bilateral
completes the quiz.
upper quadrant tenderness. He appears mildly dehydrated and drinks small amounts of
water during the visit without emesis. He wants to be held and cries throughout the
examination. What is the most appropriate next step in management?
A. Obtain complete blood cell count, liver transaminases, and pancreatic enzymes.
B. Prescribe antacid daily.
C. Request MRI of the brain.
D. Request referral for ophthalmology evaluation.
E. Request referral to a gastroenterologist.
5. You see a 3-year-old girl for a respiratory illness. Her mother states that the girl refuses to
go to sleep and wakes multiple times during the night. The girl is resistant to toilet training
and her mother reports that she holds her on the toilet to help her potty train. She has had
a few successful voids on the toilet with this method. She has several bruises on her right
scapula that her mother reports occurred when she fell from a dining room chair. She has a
faint bruise on her right facial cheek and her mother is not sure of how this injury occurred.
Her physical examination findings are otherwise normal. What is the most appropriate next
step in management?

Vol. 37 No. 4 APRIL 2016 157


A. Obtain laboratory studies for possible bleeding disorder.
B. Request polysomnography evaluation of her sleep.
C. Request renal ultrasonography and cystometrography.
D. Request referral for ophthalmology evaluation.
E. Submit a report to child welfare.

Parent Resources from the AAP at HealthyChildren.org


• https://www.healthychildren.org/English/safety-prevention/at-home/Pages/What-to-Know-about-Child-Abuse.aspx
• Spanish: https://www.healthychildren.org/Spanish/safety-prevention/at-home/Paginas/What-to-Know-about-Child-Abuse.aspx

Addendum for Meningitis


Regarding the December 2015 Pediatrics in Review article “Meningitis” (Pediatr Rev. 2015;36(12):514–526, doi: 10.1542/
pir.36-12-514:
The incidence of Haemophilus influenzae type b (Hib) invasive disease, including meningitis, has decreased
tremendously with the increased use of Hib conjugate vaccine in infants. Unfortunately, there is concern that similar
to other vaccine-preventable diseases, such as measles and pertussis, an upsurge in Hib meningitis could follow a
decrease in Hib vaccine use. Therefore, it is important for clinicians to recognize possible Hib meningitis promptly and
treat it effectively.
The footnote to the listing for Escherichia coli in Table 6 of the article on meningitis published in Pediatrics in Review
clearly states “Or other Gram-negative enteric bacilli. Choice of antibiotic is directed by the results of susceptibility
testing.” What may not be clear to all readers is that a very small percentage of Hib that are beta-lactamase-negative still
have a sufficiently high minimum inhibitory concentration (MIC) for ampicillin to make Hib resistant to ampicillin.
Thus, ampicillin is not considered a preferable antibiotic until susceptibility (based on MIC) is available. This is an
essential point because the consequences of initial ineffective therapy can be disastrous.
Out of an abundance of caution, we want to remind readers (and have updated the online version of the article with
the notation) that The Committee on Infectious Diseases of the American Academy of Pediatrics recommends, “Initial
therapy for children with H influenzae meningitis is cefotaxime or ceftriaxone. Ampicillin should be substituted if the
Hib isolate is susceptible.”
– Mobeen H. Rathore, MD, FAAP
Pediatrics in Review Editorial Board
AAP Committee on Infectious Diseases

158 Pediatrics in Review


Syncope
Bryan Cannon, MD,* Philip Wackel, MD*
*Mayo Clinic, Rochester, MN

Educational Gap
Syncope is a common problem in children and adolescents, but the
diagnostic yield for most tests used in its evaluation in pediatric patients is
low, and testing should be guided by a careful history and physical
examination. (1)

Objectives After completing this article, readers should be able to:

1. List the most common causes of syncope in pediatric patients.


2. Understand the elements in the history or physical examination that
may be related to a life-threatening cause.
3. Describe the basic evaluation for a patient who presents with syncope.
4. Discuss basic education principles for patients who present with
uncomplicated syncope.
5. Screen and identify patients who require referral to a pediatric
subspecialist.

CASE PRESENTATIONS

Case 1
A 14-year-old boy with no significant past medical history presents to the clinic
following a syncopal episode. He reports that he had been standing in church and
felt lightheaded before passing out. The event was witnessed and his parents
describe brief seizure-like activity when he was syncopal. He woke up 2 seconds
after he passed out and was alert and oriented. What is the most likely cause of the
syncope? What further evaluation is necessary? What recommendations would
you make for this patient? Would you refer him to a pediatric cardiologist or
neurologist?

Case 2
AUTHOR DISCLOSURE Dr Cannon has
A 14-year-old girl with no significant past medical history presents to the clinic
disclosed that he is a consultant for Medtronic,
USA, and serves on the Board of Directors for following a syncopal episode. She explains that she was running a 100-meter dash
Mayo Support Services-Texas. Dr Wackel has and passed out in the middle of running. She had no symptoms before her
disclosed no financial relationships relevant to
syncope. She woke up after 4 to 5 seconds but was very confused and did not
this article. This commentary does not contain
a discussion of an unapproved/investigative recognize her track coach or teammates. What are the potential causes of the
use of a commercial product/device. syncope? What further evaluation is necessary? What recommendations would

Vol. 37 No. 4 APRIL 2016 159


you make for this patient? Would you refer her to a pediatric reflex syncope, which accounts for approximately 75% of all
cardiologist or neurologist? cases of pediatric syncope. (3) Reflex syncope can be divided
into 3 basic types: vasovagal syncope, postural orthostatic
tachycardia syndrome (POTS), and orthostatic hypotension.
DEFINITION

Syncope is a sudden and transient loss of consciousness and Vasovagal Syncope


postural muscle tone that reverses without intervention. Vasovagal syncope is also referred to as neurocardiogenic
Normal brain function depends on a constant supply of syncope, vasodepressor syncope, or simple/common faint.
oxygen and glucose. A temporary decrease in cerebral blood Typically, a prodrome lasts a few seconds to 1 minute and is
flow or glucose supply may result in transient loss or near followed by syncope that usually lasts less than 1 minute.
loss of consciousness. Upright posture results in venous pooling in the lower
extremities (up to 25% of the total blood volume). Decreased
venous return results in lower blood pressure and stroke
EPIDEMIOLOGY
volume (up to 40%). Specialized receptors, known as
Syncope occurs in up to 50% of the general population, and C-fibers, in the ventricle and atrium as well as mechanore-
approximately 15% to 25% of children and adolescents ceptors in the carotid sinus and pulmonary arteries sense
experience at least 1 episode of syncope before adulthood. stretch and pressure. C-fiber/mechanoreceptor activation
The incidence peaks in the late teenage years and occurs results in a reflexive increase in parasympathetic tone
more commonly in females. Syncope accounts for as many (decreased heart rate and blood pressure), thereby reducing
as 3% of all emergency department visits in the pediatric pressure and stretch on the heart. Similarly, less C-fiber/
population. Approximately 60% of girls and 50% of boys mechanoreceptor activation from a decrease in blood
have more than 1 episode of syncope. (2) Syncope may result pressure results in a reflexive increase in sympathetic
from circulatory, metabolic, psychiatric, or neurophysio- tone (increased heart rate and blood pressure). However,
logic processes, and the causes range from benign to life- in susceptible individuals, reduced venous return (ie,
threatening. Although a substantial portion of syncope in decreased preload) results in a large increase in the force
the adult population is due to cardiac causes, most syncope of ventricular contractions to maintain adequate cardiac
in the pediatric population is benign. output. The combination of increased contractility and a
decrease in ventricular blood volume creates the so-called
“empty heart syndrome.” An increase in endogenous cat-
PATHOGENESIS
echolamines may further accentuate this response. Forceful
Syncope can be divided into several broad categories (Fig 1). contractions by an underfilled heart result in inappropriate
The most common form of syncope is autonomic-mediated activation of the C-fibers in the heart. The C-fibers increase

Figure 1. Causes and categorization of


syncope.

160 Pediatrics in Review


parasympathetic activity and inhibit sympathetic activity rhythm (sinus arrhythmia) is present after a spontaneous
(Bezold-Jarisch reflex), which results in bradycardia, vaso- rhythm returns. Typical symptoms of presyncope are diz-
dilation, and hypotension. This cascade of events may occur ziness, vision changes, a warm feeling, nausea, or an
suddenly, with the consequent reduction of cerebral perfu- irregular heartbeat before complete loss of consciousness,
sion producing loss of consciousness. although on rare occasions there are no presyncopal warn-
Often, the patient assuming an upright posture initiates ing signs. Patients frequently have isolated episodes of
the previously described cascade of events. A normal phys- presyncope without subsequent loss of consciousness. Dur-
iologic response initially may occur after assuming an ing the syncopal event, after losing postural tone and
upright posture, but susceptible individuals may develop assuming a supine position, venous return from the lower
an irregular sinus rate (sinus arrhythmia). They then can extremities increases. The increased cardiac blood volume
abruptly develop symptomatic hypotension and bradycar- combined with a spontaneous increase in heart rate results
dia, with the systolic blood pressure decreasing up to 40 to in increased cardiac output and a return to full conscious-
80 mm Hg. Bradycardia may progress to asystole that lasts 3 ness within a brief period of time (typically less than 1 minute,
to 40 seconds in 4% to 6% of patients (Fig 2). Frequently, although unconsciousness may last longer than 5 minutes).
sinus tachycardia is seen before the pause and an irregular (3) Repeated episodes of syncope may occur if the patient

Figure 2. Asystolic pause characteristic of


reflex syncope.

Vol. 37 No. 4 APRIL 2016 161


attempts to sit up or stand too quickly or is held in an upright vasovagal syncope, with a prodrome and brief period of
position. Because vasovagal syncope is related to blood pool- unconsciousness.
ing in the lower extremities due to gravity, this type of syncope
almost never happens in the supine position, although it may Cardiac Syncope
occur with sitting or standing or more commonly with a The most concerning type of syncope is true cardiac syn-
change in position from supine to sitting, supine to standing, cope. Cardiac syncope is characterized by inadequate cere-
or sitting to standing. bral perfusion due to failure of the heart as a pump. Longer
episodes of cardiac pump failure can result in sudden death
Postural Orthostatic Tachycardia Syndrome or neurologic damage from prolonged ischemia, making
POTS is a condition in which the heart rate increases recognition of this type of syncope very important. True
more than 30 beats per minute, with a heart rate of more cardiac syncope can be divided into 3 basic categories:
than 120 beats per minute without hypotension (inap- structural heart disease, arrhythmias (both tachyarrhyth-
propriate sinus tachycardia) within 10 minutes of assum- mias and bradyarrhythmias), and myocardial dysfunction.
ing an upright position. Symptoms can be syncopal or The causes of cardiac syncope are detailed in Figure 3.
presyncopal. Cardiac syncope frequently occurs without any warning
POTS has been associated with chronic fatigue, gas- signs, and any episode of loss of consciousness without
trointestinal problems, headaches, poor sleep, difficulty symptoms of presyncope should be assumed to have a
concentrating, and decreased exercise tolerance. In some cardiac cause until proven otherwise. Cardiac syncope can
patients, there is a substantial psychological influence also be preceded by severe chest pain or very rapid heart
to the symptoms. POTS has a female-to-male ratio of rates. Any patient who has no pulse during syncope or
5:1. Many affected patients do not have true syncope requires cardiopulmonary resuscitation (CPR) or defibril-
but may have marked debilitation from presyncopal lation should undergo a full evaluation to exclude a cardiac
symptoms. cause. One of the most concerning signs for underlying
cardiac pathology is syncope while actively exercising.
Orthostatic Hypertension During exercise, the contracting leg muscles act as a pump
Orthostatic hypotension represents a decrease in systolic that helps return venous blood to the heart. Because of the
blood pressure of more than 20 mm Hg or in diastolic augmented venous return, vasovagal syncope during active
pressure of more than 10 mm Hg within 3 minutes of exercise can occur, although this is rare and cardiac syncope is
assuming an upright position. (4) In this condition, the much more likely. In contrast, syncope immediately after
normal adrenergic vasoconstriction of arterioles and veins is exercise can have a cardiac cause, but it is more likely to be
absent or inadequate, which results in hypotension without vasovagal because the increased blood flow to the muscles
a reflex increase in heart rate. This can occur with prolonged during exercise combined with the abrupt withdrawal of the
bed rest or prolonged standing or in patients who have increased venous return caused by muscular contractions may
chronic medical conditions. Pure orthostatic hypotension set up the cascade of events leading to a vasovagal episode.
is a rare cause of syncope; frequently some element of
heart rate change accompanies the symptoms. Causes of Neurologic Syncope
orthostatic hypotension include volume depletion, acute or Neurologic conditions such as seizures or migraines can
chronic illness, medications (eg, calcium channel blockers, also present with syncope. Patients who exhibit tonic-clonic
angiotensin-converting enzyme inhibitors), and autonomic movements or have prolonged periods of confusion or
dysfunction. postictal states should undergo neurologic evaluation. How-
ever, brief posturing or myoclonic jerks that may be inter-
Situational Syncope preted as seizures are common after an episode of reflex
A second form of syncope that is closely related to reflex syncope. Such myoclonic jerks are typically of a short
syncope is situational syncope. The pathophysiology is duration; longer episodes are more likely related to seizure
similar to vasovagal syncope, but a specific trigger or action activity. Loss of bowel or bladder control is rare in reflex
initiates the cascade of events leading to syncope. Common syncope but more common in neurologic syncope. Cere-
forms of this syncope include syncope induced by the sight brovascular occlusive disease (stroke) is extremely rare in
of blood, pain, or fear. Hair brushing and micturition have children but may be seen in hypercoagulable states or
also been well described as triggers for syncope. The conditions that affect the arteries supplying blood to the
episodes have a clinical presentation that is the same as brain.

162 Pediatrics in Review


Figure 3. Causes of cardiac syncope.

Psychogenic Syncope patient can appear cyanotic or pallid during the episode, but
Psychogenic syncope, also called “pseudoseizures” or the actual breathholding may be difficult to see. Typically
“behavioral spells,” occurs when no physiologic alter- within seconds of syncope, a patient with a breathholding
ations lead to the syncopal episode. Psychogenic syncope spell begins breathing and regains consciousness. Breath-
typically occurs in the presence of an audience or at a holding spells usually occur in young children beginning as
specific time (before school). These episodes frequently young as 6 to 18 months of age but generally resolve by 5 to 6
occur in an emotionally charged setting or during times years of age. Failure of venous return as a mechanism of
of stress. Typically, the patient has no pallor or hypoten- syncope can result from increased intrathoracic pressure
sion during the episode, and the episodes may be pro- (pneumothorax, pericardial effusion) or severe hypo-
longed, lasting up to 1 hour. Psychogenic syncope is very volemia, but these are very rare causes of syncope in the
rare before age 10 years. Usually the episodes are due to a pediatric population. Finally, teenagers may deliberately
conversion disorder, in which adolescents express emo- engage in games designed to cause syncope, such as the
tional feelings through physical symptoms, an expression “fainting lark,” in which the teenager squats, hyperventi-
that is not a conscious or deliberate act. However, some- lates, stands up suddenly, and forcefully exhales against a
times adolescents deliberately feign syncope to gain closed glottis. Persistent hyperventilation can also result in
attention or avoid a particular circumstance. Both emo- syncope.
tional and sexual abuse may be the precipitating factor for
psychogenic syncope, and questions related to the poten-
CLINICAL ASPECTS
tial for abuse should be raised in all patients with psy-
chogenic syncope. In the pediatric population, the overwhelming majority of
syncopal events are benign. However, a small subset of
Other Causes of Syncope patients is potentially at risk for sudden cardiac death,
Other causes of syncope include alcohol intoxication or drug making the evaluation of a patient with syncope a potential
effects. Eating disorders such as anorexia may also result in diagnostic challenge. Clinicians should try to determine
syncope related to a combination of bradycardia, hypovole- an underlying cause to address the mechanism-specific
mia, and hypoglycemia. Breathholding spells can cause method of treatment and determine the potential risk for
syncope in young patients. The patient who is angry, in a life-threatening event. A suggested algorithm is shown in
pain, or upset holds his or her breath until passing out. The Figure 4.

Vol. 37 No. 4 APRIL 2016 163


Figure 4. Diagnosis and treatment algorithm for true syncope.

History physiologic and should raise concern for a psychological


The most important tool in diagnosing syncope is the mechanism.
history. A detailed history of all the events surrounding the An equally important part of the history is the family
episode is frequently time-consuming but critical in deter- history. Many life-threatening arrhythmias are caused by an
mining the potential underlying cause. It is important for autosomal dominant genetic condition. A history of sudden
the patient to describe everything that he or she felt before cardiac death or death in a relative younger than age 50 years
and after the episode. Talking to any witnesses can help to warrants further investigation of the specific circumstances
determine the specifics about the syncope, including the and cause of the death. A family history of cardiomyopathy,
duration. Of note, though, observers frequently overesti- placement of an implantable cardioverter-defibrillator or
mate the amount of time that a patient is unconscious. pacemaker, long QT syndrome (or congenital deafness,
Other important factors include time of day the syncope which has been associated with long QT syndrome), or
happened and whether the patient had anything to eat or congenital heart disease should prompt evaluation for a
drink that day because reflex syncope may be brought on potential heritable cardiac cause for the syncope. A family
by dehydration or fasting. Episodes of self-resolving syn- history of syncope may also be helpful because multiple
cope lasting longer than 10 minutes are almost never members in the same family may have reflex syncope.

164 Pediatrics in Review


Signs and Symptoms Routine blood studies rarely are helpful in determining
Distinguishing presyncope from true syncope is important. the cause of syncope unless the physical examination or
Presyncope involves symptoms such as weakness, sweating history suggests a specific underlying cause. Hypoglycemia
or a warm “rushing” feeling, dizziness, visual disturbances, can cause syncope, but serum glucose values have fre-
headache, palpitations, abdominal discomfort/nausea, or quently returned to normal at the time of evaluation. As-
pallor without a complete loss of consciousness. Although sessing glucose concentrations in close proximity to the
these symptoms frequently precede true syncope, they are syncopal episode may be helpful but often has minimal
not as concerning when true complete loss of consciousness value. Serum electrolytes, including calcium, are also of little
does not occur. If a patient did not become completely utility in establishing a cause of syncope. Although iron
flaccid or remembers all the events that happened (includ- deficiency anemia has been implicated in autonomic dys-
ing falling to the ground), he or she may not have experi- function and reflex syncope, it is a rare primary cause of
enced true syncope and may require further evaluation to syncope, and a complete blood cell count is typically not
determine the underlying cause of presyncopal signs and helpful in establishing a diagnosis. In adolescent females,
symptoms. screening for pregnancy may be indicated because the
physiologic adaptations of pregnancy may result in a syn-
Laboratory and Imaging Studies copal event. A serum or urine drug screen should also be
Electrocardiography (ECG) is important in the evaluation of considered. In the absence of a neurologic cause or focal
patients who have syncope because findings may suggest an neurologic findings, imaging of the brain with computed
underlying cardiac cause that may not otherwise be evident. tomography scan or magnetic resonance imaging is rarely
When evaluating the ECG, the clinician must measure the helpful, although these frequently are performed. Electro-
QT interval. Because the QT interval varies with heart rate, it encephalography is also rarely indicated and may yield
must be adjusted to obtain a corrected QT interval (QTc). abnormal results immediately after an episode of reflex
The most commonly used equation is Bazett’s formula (the syncope, which can create the impression of an underlying
QT interval in seconds divided by the square root of the seizure disorder.
previous QRS-to-QRS interval in seconds). The upper limit
of normal for a corrected QT interval is 440 milliseconds in Diagnostic Tests
males and 450 milliseconds in females. However, most One means of potentially assessing for reflex syncope is a
individuals with a corrected QT between 440 and 470 head-upright tilt table study, but its routine use is contro-
milliseconds with a normal T wave morphology do not have versial, and results in most instances of routine syncope are
an underlying cardiac pathology and are not at risk for life- of limited value. In this study, the patient lies on a table that
threatening arrhythmias. QTc values greater than 470 mil- can be rotated from 0 to 90 degrees from the horizontal
liseconds should prompt an evaluation for long QTsyndrome position. During this time, the patient is monitored with a
and values greater than 500 milliseconds are almost never continuous ECG tracing and blood pressure readings.
seen in the absence of a pathologic condition. The onset of Recordings are taken with the patient supine and when
the QRS should be scrutinized for the pre-excitation delta the patient is angled to 60 to 90 degrees by tilting the table.
wave seen in Wolff-Parkinson-White syndrome (Fig 5). Other A positive test result consists of reproduction of symptoms
important findings are atrioventricular conduction distur- usually accompanied by hypotension or bradycardia. Other
bances or atrioventricular block, evidence of ventricular hyper- methods to assess for autonomic dysfunction (such as the
trophy, or other abnormalities that may suggest underlying response to a Valsalva maneuver or response to facial
heart disease (eg, T-wave inversion in leads V5 and V6, which immersion in ice water) can be performed during the
may indicate left ventricular pathology). evaluation. The sensitivity and specificity of tilt table testing
varies widely, depending on the protocol being used. The
false-negative rate ranges from 14% to 30% but can be
higher, depending on the protocol. (5) Using different
protocols for the study can increase the sensitivity but
subsequently decrease the specificity, resulting in up to a
45% false-positive rate in some cases. (6)(7) Because patient
history can diagnose most cases of reflex syncope, a tilt table
Figure 5. Pre-excitation: The first 3 beats show the characteristic delta
study may not be necessary in all patients, and most pediatric
wave seen in Wolff-Parkinson-White with the last 3 beats (after the *)
showing normal conduction. electrophysiologists do not believe that it is beneficial in the

Vol. 37 No. 4 APRIL 2016 165


diagnostic evaluation of syncope. (8) However, in cases of artery abnormalities can develop during exercise stress test-
psychogenic syncope, it can be helpful to observe episodes of ing, this modality is not sensitive for detecting ischemia in
syncope that have no heart rate or blood pressure changes. pediatrics, and definitive imaging of the coronary arteries
Tilt table studies may also be useful in determining a cause of and ventricular function with echocardiography, computed
recurrent syncope that does not have clear antecedents or in tomography scan, or magnetic resonance imaging or cardiac
cases with atypical symptomatology. Although a formal tilt catheterization is indicated if there is concern for a coronary
table test may not be necessary, monitoring a patient’s heart artery abnormality.
rate and blood pressure in the supine position and after Single isolated episodes of syncope with a history typical
standing for 3 minutes may be beneficial. for reflex mechanism syncope do not require extensive
If the clinician is concerned about an arrhythmic mech- evaluation. Multiple episodes of syncope or syncope with
anism of syncope, documenting the heart rhythm during an unusual historical details not classic for reflex syncope may
episode of syncope is important and can be accomplished by require more extensive evaluation, including blood labora-
several different methods. An event monitor is a small tory tests, neurologic imaging, or subspecialty consultation.
electronic device that can record an ECG when activated.
There are two types of ambulatory event monitors. For the
MANAGEMENT
first type, the patient wears leads that are continuously
connected to a device that records the heart rhythm. When An important part of management is deciding who should
the device is activated, it documents the rhythm recorded be referred to pediatric cardiology or neurology. If there are
several minutes before and after activation. For the second specific concerns for a neurologic or particularly a poten-
monitor type, the patient’s rhythm is recorded by a monitor tially life-threatening cardiologic cause, the referral should
that is connected to the patient during episodes of syncope. be immediate. Any patient with a syncopal episode who
Another method for recording a patient’s heart rhythm is required resuscitation by CPR or received defibrillation
the 24-hour Holter monitor, which is useful if the syncopal from an automated external defibrillator or defibrillator in
episodes occur on a daily basis. the hospital should undergo a thorough evaluation by a
In extreme cases, hospitalization with continuous heart pediatric cardiologist before leaving the hospital or clinic.
rhythm, blood pressure, and/or ECG monitoring may be Fundamental to the treatment of reflex syncope is
indicated. Another diagnostic approach is using an ECG increasing water and salt intake, particularly upon waking.
monitor that is implanted under the skin (implantable loop A regular exercise program is also very important in therapy
recorder), which can monitor the rhythm continuously for of reflex syncope. Exercise is frequently challenging because
periods of up to 3 years. With this approach, the patient patients may fatigue easily and feel exhausted after minimal
wirelessly triggers an ECG recording through an activator exertion. If exercise is not continued, despite its difficulty,
button when he or she feels symptomatic. In addition, the patients can become more deconditioned, which exacer-
device automatically records heart rates above or below a bates the underlying mechanism of their symptoms. Patients
predetermined set rate or if there is a rhythmic pause of should also avoid diuretics such as caffeine and alcohol. They
more than 3 seconds. This approach has been shown to be should be instructed to lie down with their feet elevated when
very effective in determining arrhythmic or reflex causes of symptoms start or perform counterpulsation measures in an
syncope. However, it is rarely indicated, except when a attempt to avoid syncope. Counterpulsation measures involve
cardiac arrhythmia is suspected but cannot be documented purposeful contraction of the muscles in the arms and legs in
by other means. (9) Continuous ambulatory blood pressure an effort to mechanically force blood return to the heart.
monitoring also can be used to document hypotension. Patients should also avoid circumstances that predispose to
An exercise stress test (bicycle or treadmill) is indicated syncope, such as dehydration or prolonged standing.
for episodes of syncope that occur during exercise. The heart Medications may be considered for patients with refrac-
rate and blood pressure response to exercise can help tory reflex syncope. However, medications are frequently
determine if there is a reflex cause of syncope. In addition, ineffective and may have adverse effects. In addition, they
arrhythmias during exercise may be observed. In a pa- may improve symptoms but only rarely completely elim-
tient with catecholamine-sensitive polymorphic ventricular inate them. One of the more effective medications is
tachycardia, the baseline physical examination and ECG midodrine. This a-receptor agonist constricts arterioles
findings are normal, but the patient develops increasing and veins and increases peripheral vascular resistance.
ventricular ectopy and frequently ventricular tachycardia Oral midodrine is rapidly absorbed. The dose is normally
during exercise. Although ischemic changes due to coronary 2.5 to 10 mg. The drug reaches peak blood concentrations

166 Pediatrics in Review


in 1 to 2 hours and the half-life is approximately 3 to 4 hours. they should be warned that their syncope may recur and be
For this reason, midodrine is usually taken 3 times a day. instructed to lie down with their feet up if they feel dizzy.
However, the dosing schedule should be shifted toward Syncope is a very frightening event for most families, and
earlier in the day because the blood pressure-elevating they frequently need significant reassurance about its
effects are not desirable when the patient is supine at benign nature. Many patients with recurrent reflex syncope
bedtime. The typical dosing schedule is to take 3 doses improve after 1 to 2 years or in late adolescence and may
4 hours apart beginning in the morning (eg, 7 AM, 11 AM, even have their symptoms completely resolve, although
and 3 PM). Because midodrine may increase blood pressure patients may have recurrences into their third decade.
substantially, several blood pressure readings should be Recurrent syncope frequently responds to conservative
obtained 1 to 2 hours following the initial dose. Other measures such as diet and exercise. However, patients
adverse effects include blurred vision; headache; pounding may require medication for even a few years until their
in the ears; burning; itching; chills; piloerection; and symptoms improve. For some patients, the reflex syncope
urinary frequency, retention, or urgency. can become a debilitating condition with a significant
b-blockers have also been used but are ineffective in component of depression. For these patients, a chronic pain
many patients. (10) In fact, some patients with reflex syn- rehabilitation program that includes psychiatric behavioral
cope actually feel worse when receiving these medications. therapy may be beneficial. Clinicians must explain to the
b-blockers may also cause fatigue and should be used with family that there is no quick “cure” and that complete
caution in patients who have diabetes (because they may resolution of reflex syncope and presyncope symptoms is
block hypoglycemic symptoms) or reactive airways disease. frequently not possible. The goal is to minimize symptoms
b-blocker therapy is frequently initiated with a very low dose; so that patients can be functional in their everyday lives.
even subtherapeutic doses may be sufficient to reduce
symptoms. The dose can be slowly titrated upward to achieve
the desired effect. Fludrocortisone is a synthetic corticoste-
roid that acts similarly to aldosterone, causing sodium and
water retention and increasing potassium excretion. In the- Summary
ory, fludrocortisone should increase the blood volume (and, • On the basis of strong evidence, syncope is a typically benign
entity in young patients and usually has a good prognosis. (3)
thus, decrease venous pooling and symptoms) in patients
with reflex syncope, thereby improving symptoms. However, • On the basis of good evidence and expert opinion, syncope
during active exercise, syncope that occurs without warning, and
fludrocortisone was ineffective in a double-blind, placebo-
syncope preceded by severe chest pain or rapid palpitations
controlled study in children with reflex syncope. (11) possibly have cardiac origins and require further evaluation. (12)
Although reflex syncope may result in heart rate pauses • On the basis of good evidence and expert opinion, most causes of
greater than 10 to 20 seconds, a pacemaker is almost never syncope can be determined by a thorough history, but
indicated. In rare cases where severe bradycardia is the electrocardiography and physical examination are also important
overwhelmingly predominant symptom or in small children parts of the evaluation to rule out cardiac causes. (1)
with multiple breathholding spells occurring on a daily or • On the basis of good evidence and expert opinion, the primary
weekly basis, a pacemaker may be beneficial. However, therapies for reflex syncope are conservative measures such as
increased fluid and salt intake, a routine exercise program, and
caution should be exercised in teenage patients because
counterpulsation measures. (13)
the reflex syncope response is rarely only bradycardia, and a
• On the basis of good evidence, medications are not particularly
pacemaker may not be beneficial for patients in whom both
effective at preventing recurrent episodes of syncope, with the
hypotension and bradycardia play a role in their syncope. exception of midodrine, which may be beneficial in children with
a prominent hypotensive response. (14)

PROGNOSIS

Patients who have a single episode of reflex syncope with


normal physical examination findings, family history, and CME quiz, references, and suggested readings for this article are at
ECG results require no further evaluation or follow-up, but http://pedsinreview.aappublications.org/content/37/4/159.

Vol. 37 No. 4 APRIL 2016 167


PIR Quiz
There are two ways to access the journal CME quizzes:
1. Individual CME quizzes are available via a handy blue CME link under the article title in the Table of Contents of any issue.
2. To access all CME articles, click “Journal CME” from Gateway’s orange main menu or go directly to: http://www.
aappublications.org/content/journal-cme.

1. The autonomic reflex responsible for vasovagal syncope results in which of the following REQUIREMENTS: Learners
physiological changes? can take Pediatrics in
A. Bradycardia, vasoconstriction, and hypertension. Review quizzes and claim
B. Bradycardia, vasodilation, and hypertension. credit online only at:
C. Bradycardia, vasodilation, and hypotension. http://pedsinreview.org.
D. Tachycardia, vasoconstriction, and hypertension.
E. Tachycardia, vasodilation, and hypotension. To successfully complete
2. A 10-year-old boy presents to the emergency department following a period of loss of 2016 Pediatrics in Review
consciousness. His parents report that he was playing tag with his brother when he articles for AMA PRA
suddenly dropped to the ground. The episode lasted less than 1 minute, after which time Category 1 CreditTM,
he quickly returned to baseline activity. He does not describe any aura or symptoms before learners must
the event and there were no abnormal movements during the event. Which of the demonstrate a minimum
following diagnoses is most likely? performance level of 60%
A. Cardiogenic syncope. or higher on this
B. Migraine. assessment, which
C. Psychogenic syncope. measures achievement of
D. Seizure. the educational purpose
E. Vasovagal syncope. and/or objectives of this
activity. If you score less
3. A 5-year-old girl was standing in line at a grocery store with her mother when she suddenly
than 60% on the
fell to the ground. Her mother reports that the girl had shaking of her arms and legs that
assessment, you will be
lasted for 20 seconds and urinary incontinence. Following this event, she was disoriented
given additional
and sleepy for 1 hour. Which of the following characteristics is most suggestive of seizure
opportunities to answer
as a cause for her loss of consciousness?
questions until an overall
A. The episode is followed by a 1-hour postictal period characterized by disorientation 60% or greater score is
and sleepiness. achieved.
B. The episode is associated with urinary incontinence.
C. The episode is described as shaking of her arms and legs.
D. The episode occurred in the absence of physical activity. This journal-based CME
E. The episode is 20 seconds long. activity is available
through Dec. 31, 2018,
4. A 14-year-old girl presents to your clinic with 5 episodes of syncope over the last 2 months.
however, credit will be
These episodes usually occur before school but without specific triggers. Her episodes of
recorded in the year in
loss of consciousness typically last 20 to 30 minutes, after which time she quickly returns to
which the learner
baseline activity. On review of systems, she has a 2-year history of abdominal pain,
completes the quiz.
headache, and intermittent blurred vision, which have resulted in many missed days of
school over the last 3 months. What should your next step be?
A. Order electroencephalography.
B. Order magnetic resonance imaging.
C. Order a tilt table study.
D. Refer her to psychiatry.
E. Tell her this is due to a virus and that it will get better.
5. Which of the following is an inappropriate recommendation for patients with reflex/
neurally mediated syncope?
A. Avoid all physical activity.
B. Avoid caffeine and alcohol.
C. Avoid periods of prolonged standing.
D. Increase salt and water intake.
E. Lie down with feet elevated to treat feelings of lightheadedness.

168 Pediatrics in Review


1 Diplopia in a 15-year-old Boy

Christine Puthawala, DO,* Shana Hansen, MD†


*Submarine Base New London, Groton, CT.

Adolescent Clinic, Fort Sam Houston, TX.

PRESENTATION

A 15-year-old previously healthy boy presents to the adolescent clinic with bilateral
EDITOR’S NOTE
We invite readers to contribute Index of frontal headaches over the last 4 months. He was initially treated with antibiotics
Suspicion cases at: Submit and Track My for presumed sinusitis. The headaches improved after 2 weeks of therapy,
Manuscript. decreasing in frequency to once a month. They are sometimes present upon
awakening but never wake him from sleep. At the time the headaches began, the
AUTHOR DISCLOSURE Drs Puthawala and
Hansen have disclosed no financial
patient also developed progressively worsening double vision. He also reports
relationships relevant to this article. This hearing his heartbeat in his left ear for several weeks starting 9 months ago. For
commentary does not contain a discussion of the last 10 days he has experienced impaired balance when his eyes are shut. He
an unapproved/investigative use of a
denies travel, trauma, fevers, weight loss, mood changes, weakness, sensory
commercial product/device.
deficits, vomiting, nausea, or a history of Lyme disease or meningitis. He has no
significant past medical history or history of developmental delay.
On physical examination, his temperature is 36.7°C (98.1°F), heart rate is 95
beats per minute, respiratory rate is 16 breaths per minute, and blood pressure is
112/77 mm Hg. He is a healthy-appearing teenager in no acute distress. Mental
status is normal. Pupils are equally reactive, but bilateral optic disc edema is noted.
No nystagmus, ptosis, or visual deficits are present. Cranial nerves are normal, with
the exception of an inability to abduct the right eye with horizontal movement,
which is exacerbated with gaze to the right. The teen also has binocular diplopia.
He has 3þ patellar reflex on the left and the rest of the deep tendon reflexes are 2þ.
His strength is 5/5 in the upper and lower extremities, muscle tone is normal,
and sensation to light touch is intact. His gait, including tandem gait, is normal.
A Romberg test and further cerebellar testing yield normal results.
Laboratory evaluation is deferred for an emergency computed tomography
(CT) scan.

DISCUSSION

CT scan revealed hydrocephalus with bilaterally enlarged lateral ventricles and


third ventricle. Signs of hemorrhage, mass effect, and infarction were not present.
Magnetic resonance imaging (MRI) documented narrowing of the cerebral
aqueduct with right displacement of the pineal gland and dilation of the vein of
Galen. No vein of Galen aneurysm was noted. MRI was negative for any space-
occupying lesions. The patient was diagnosed with aqueductal stenosis and
ultimately received a ventriculoperitoneal shunt, which resolved his hydrocephalus
and diplopia.

Vol. 37 No. 4 APRIL 2016 169


The Condition displacement. Monocular diplopia usually has an intraocu-
Late-onset idiopathic aqueductal stenosis (LIAS) is easily lar pathology, which necessitates expedited ophthalmologic
missed because it may present without a history of trauma, assessment.
meningitis, or an intracranial hemorrhage. Although hy- Periorbital pain or pain with eye movements may
drocephalus is generally diagnosed in neonates and young suggest an inflammatory process. Ptosis and worsening
children, its incidence has a bimodal distribution, with the diplopia with intensive use of the eye may indicate
most frequent diagnoses seen in infants younger than age myasthenia gravis. Weakness of proximal limb muscles
12 months and adolescents. may be due to mitochondrial myopathy or congenital
The pathophysiology of LIAS is not completely known. myopathy. Progressive headache and papilledema sug-
The hypothesis is that the process begins with decreased gest increased ICP.
compliance of the intracranial veins, which results in The differential diagnosis for hydrocephalus and sub-
increased development of venous collateral flow. This sequent increased ICP is extensive, with traumatic brain
impairs brain compliance and results in aqueductal occlu- injury resulting in cytotoxic edema being the most com-
sion followed by ventriculomegaly. Pediatric patients, such mon cause in the pediatric population. Increased ICP is
as the one presented, develop symptoms of increased intra- also commonly caused by infections, such as bacterial
cranial pressure (ICP), while older adults develop ataxia, meningitis, mumps, cytomegalovirus, influenza A, and
cognitive impairment, and incontinence more typical of in the fetus, toxoplasmosis. Hemorrhage within the ven-
normal-pressure hydrocephalus. tricles or brain matter can also lead to elevated ICP and may
Of note, hydrocephalus can be stable for years. Such result in hydrocephalus if debris obstructs the flow of
“arrested hydrocephalus” can acutely decompensate after cerebrospinal fluid. Space-occupying lesions, including
head trauma, hemorrhage, and infections or after compro- tumors, aneurysms of the vein of Galen or basilar artery,
mise of the compensatory capacities of the brain. Alterna- and arteriovenous malformations, can cause increased
tively, hydrocephalus may develop more gradually as the ICP. Among the additional causes of elevated ICP are
increased pressure places greater sheer stress on the aq- genetic diseases such as the X-linked L1 syndrome
ueduct, which eventually causes symptomatic damage. In (Bickers-Adams-Edwards syndrome) and neurofibromato-
addition, increased pressure in the supratentorial ventricles sis type-1. Central nervous system malformations such as
may cause kinking of the aqueduct. Chiari and Dandy-Walker malformations or spina bifida
Signs of hydrocephalus include macrocephaly, visual also may result in hydrocephalus. Finally, hydrocephalus
disturbances, papilledema, Collier sign (“setting sun sign”), may be caused by “functional stenosis” that causes large
and Parinaud syndrome (upward gaze paralysis). If in- pressure differences between the supra- and infratentorial
creased ICP is present, patients may have symptoms such spaces.
as headache, as seen in this patient. Pupillary dilation and a There are several mechanisms by which the previously
paralysis of the light reflex is a late finding. Ten percent of listed causes trigger aqueductal stenosis and, thereby,
adolescents with aqueductal stenosis exhibit symptoms hydrocephalus. For example, an external mass may simply
related to compression of the hypothalamic-hypophyseal force the aqueduct shut. Intrinsic pathology of the aque-
axis, including obesity, diabetes insipidus, precocious puberty, duct may cause an occlusion by stenosis of the aqueduct,
and amenorrhea. forking, septum formation, or gliosis following inflamma-
tion from infection, toxins, or hemorrhage. Experimental
Differential Diagnosis evidence also suggests that certain toxins (trypan blue,
Diplopia has numerous causes. In many cases, the diag- salicylates, and cuprizone) and dietary deficiencies (vita-
nosis can be reached with a detailed history and physical min A, vitamin B, and folic acid) can cause stenosis of the
examination. cerebral aqueduct.
The first step is to determine whether the diplopia is
monocular or binocular. If the diplopia is only present with Management and Prognosis
both eyes open, the patient has binocular diplopia. The The management of hydrocephalus is highly dependent on
clinician must determine the cause of “ocular misalign- the cause. All cases warrant MRI. Corticosteroids, furose-
ment.” Ascertaining whether the diplopia is horizontal, mide, and acetazolamide have limited success in treating
vertical, or oblique is helpful. The most common causes acute hydrocephalus. No data support the concept that
of diplopia generally involve extraocular muscle dysfunc- medications decrease the number of patients eventually
tion, although binocular diplopia may also be due to eye requiring ventriculoperitoneal shunting.

170 Pediatrics in Review


One of the most common treatments for hydrocephalus is Lessons for the Clinician
ventriculoperitoneal shunting. Although the mortality rate • Hydrocephalus can present in the pediatric population
associated with the procedure is low, 40% of shunts fail within without a history of trauma, infection, or tumor. The
12 months, after which the failure rate stabilizes at 5% annually. presentation may include visual and gait disturbances and
Half of failures are due to obstruction, but other complications headache.
include infection (most commonly Staphylococcus epidermidis in • Imaging with computed tomography scan or magnetic
the first 6 months), subdural hematomas due to overshunting, resonance imaging is recommended for cases of sus-
headaches, and nausea. Craniosynostosis is a potential com- pected elevated intracranial pressure.
plication of ventriculoperitoneal shunting specific to infants.
Approved for public release; distribution is unlimited. The
Prognosis depends on the cause of hydrocephalus, the
views expressed herein are those of the authors and do not
time from onset until diagnosis, and the effectiveness of
necessarily reflect the official position of the Department of
treatments. Hydrocephalus caused by genetic conditions has
Defense or its components.
the worst prognosis. For nontumor-associated hydrocephalus,
as seen with this patient, the 10-year mortality rate is between Suggested readings for this article are at http://pedsinreview.
5% and 15%. aappublications.org/content/37/4/169.

ANSWER KEY FOR APRIL 2016 PEDIATRICS IN REVIEW


Hematopoietic Stem Cell Transplantation in Children and Adolescents: 1. B; 2. B; 3. E; 4. B; 5. E.
Physical Abuse of Children: 1. D; 2. E; 3. B; 4. A; 5. E.
Syncope: 1. C; 2. A; 3. A; 4. D; 5. A.

Vol. 37 No. 4 APRIL 2016 171


Acute Onset of Lower Extremity Weakness in
2 a 16-year-old Korean Boy

Sandy Aikara, MD,* Srividya Naganathan, MD,* Santhosh Eapen, MD*


*Jersey Shore University Medical Center, Neptune, NJ.

PRESENTATION

A 16-year-old Korean boy presents to the emergency department with acute onset
AUTHOR DISCLOSURE Drs Aikara and
Naganathan have disclosed no financial of lower-extremity weakness. He woke up from sleep, had several episodes of
relationships relevant to this article. Dr Eapen emesis, and found he was unable to move his lower extremities. He recalls eating
has disclosed that he owns common shares of
half a dozen doughnuts earlier in the day. His parents found him struggling to get
Dexcom, Roche Holdings, Omnipod, and
Illumina. This commentary does not contain a up and brought him in for evaluation.
discussion of an unapproved/investigative He reports intermittent palpitations and an unintentional weight loss of 40 lb
use of a commercial product/device. over the past year. He has had similar episodes of lower-extremity weakness with
no loss of sensation. These past episodes would last approximately 3 to 4 hours,
resolve spontaneously, and always occur at night. The patient was adopted at 6
months of age and, therefore, family history is unavailable.
On physical examination, his vital signs are temperature of 36.5°C (97.8°F),
heart rate of 78 beats per minute, respiratory rate of 16 breaths per minutes, and
blood pressure of 142/65 mm Hg. His body mass index is greater than the 95th
percentile. He is very combative and anxious. A grade 1 systolic ejection murmur
is auscultated at the left sternal border. He exhibits 5/5 muscle strength in the
upper extremities and 2/5 strength in the lower extremities. His sensation and
proprioception are intact, and his deep tendon reflexes are difficult to elicit. The
rest of the physical examination findings are within normal limits.
Initial laboratory evaluation documents potassium of 1.3 mEq/L (1.3 mmol/L)
and a urine drug screen positive for cannabinoids. His complete blood cell count,
creatinine phosphokinase, hepatic function panel, chest radiograph, and urinal-
ysis results are within normal limits. Electrocardiography shows a normal sinus
rhythm with right bundle branch block and a prominent U wave consistent with
severe hypokalemia. Further diagnostic evaluation reveals the cause of the
hypokalemia and the explanation for the recurrent episodes of weakness.

DISCUSSION

The combination of muscle weakness and severe hypokalemia prompted an


evaluation for periodic paralysis. The typical signs and symptoms of thyrotoxicosis
occur as a result of the excess thyroid-stimulating hormone affecting the body’s
function and metabolism. Common symptoms include weight loss, nervousness
or irritability, diaphoresis, hyperthermia, heat intolerance, muscle weakness,
tachycardia, and tremors. This patient presented with a history of weight loss
and intermittent palpitations suggesting hyperthyroidism. Thyrotropin (TSH)

172 Pediatrics in Review


measured during the paralytic event is low at 0.032 IU/mL include conditions and substances that cause a transcellular
(normal range, 0.34–5.6 IU/mL). Further evaluation documents shift or loss of potassium (Table 2).
elevated free triiodothyronine of 1,020 pg/dL (15.7 pmol/L)
(normal range, 227–357 pg/dL [3.5–5.5 pmol/L]), free thyroxine The Condition
(T4) of 4.03 ng/dL (51.9 pmol/L) (normal range, 0.5–1.26 ng/dL Periodic paralysis is a muscle disease due to a channelopathy.
[6.4–16.2 pmol/l]), and a total T4 of 16.85 mg/mL (288 nmol/L) Patients usually present with an episode of painless
(normal range, 5.28–9.87 mg/dL [90.3–168.7 nmol/L]). He is muscle weakness. Common precipitants include heavy
treated with potassium supplements and discharged from the exercise, fasting, and high-carbohydrate meals. Most cases
hospital with methimazole and propranolol. Follow-up evalu- are hereditary, with an autosomal dominant inheritance
ation shows elevated thyroid-stimulating immunoglobulin and pattern. The acquired cases are usually associated with
TSH receptor antibodies, supporting a diagnosis of Graves’ hyperthyroidism.
disease. Since his admission, he has met with an endocrinol- Periodic paralysis is classified as hypokalemic and hyper-
ogist. He is currently taking 5 mg of oral methimazole daily and kalemic, based on the serum potassium concentration.
has had no further attacks; his repeat thyroid function test Hypokalemic periodic paralysis occurs within the first or
results are within normal limits. second decade, with attacks occurring infrequently but
lasting hours to days. Precipitants include exercise, carbohy-
Differential Diagnoses drate load, and stress. Hyperkalemic periodic paralysis is seen
The inability to move muscles actively and voluntarily within the first decade, with attacks occurring frequently and
against resistance is defined as weakness. Acute weakness lasting minutes to hours. Among the precipitants are exercise,
can be categorized based on location from central to pe- fasting, stress, and potassium-rich food. Both forms of paralysis
ripheral (Table 1). Differential diagnoses for hypokalemia are associated with later-onset myopathy.

TABLE 1. Differential Diagnoses for Weakness


CENTRAL SPINAL CORD PERIPHERAL NERVE

Intracranial hemorrhage Cord trauma Guillain-Barré syndrome


Subdural hematoma Hematoma Heavy metal toxins (arsenic,
mercury, thallium)
Epidural hematoma Vertebral column fracture Ciguatera fish poisoning
Rupture of an arteriovenous malformation or aneurysm Spinal tumor Paralytic shellfish poisoning
Subarachnoid hemorrhage Paraspinal infection or inflammation
Stroke Epidural abscess
Brain tumor Discitis
Seizure with Todd paralysis Transverse myelitis
Hemiplegic migraine Anterior horn cell disease
(ie, poliomyelitis)
Alternating hemiplegia of childhood
NEUROMUSCULAR JUNCTION MUSCLE OTHER
Botulism Rhabdomyolysis Electrolyte disturbances
Myasthenia gravis Myositis Medication-induced
Organophosphate poisoning Pyomyositis Conversion disorder
Carbamate poisoning Familial periodic paralysis
Neurotoxic snake envenomation Thyrotoxic periodic paralysis
Tick paralysis Trichinellosis

Vol. 37 No. 4 APRIL 2016 173


which falls within the reference range. However, he may
TABLE 2. Differential Diagnoses for have had a lower baseline resting heart rate. The patient
Hypokalemia also presented 4 hours after initiation of the attack, and
compensatory mechanisms may have allowed normaliza-
RENAL LOSSES TRANSCELLULAR SHIFTS
tion of his heart rate and temperature. Attacks are char-
Interstitial nephritis Thyrotoxic periodic paralysis acterized as recurrent, transient episodes of muscle
Barter syndrome Familial periodic paralysis weakness of the proximal muscles, primarily of the lower
Gitelman syndrome Sporadic periodic paralysis extremities, that range from mild to complete flaccid
paralysis. The severity of the weakness usually corre-
Renal tubular acidosis Barium poisoning
sponds to the degree of hypokalemia but not to the
Primary hyperaldosteronism Alkalosis thyrotoxic state.
Aminoglycosides Tocolytics The presence of hypokalemia in the setting of acute
Amphotericin Theophylline toxicity weakness is a clue to evaluate for periodic paralysis. When
there is no family history, the possibility of thyrotoxicosis
Chemotherapeutic agents Chloroquine toxicity
must be evaluated.
Licorice Insulin
Liddle syndrome Diuretics Treatment and Management
EXTRARENAL LOSSES DECREASED INTAKE Acute treatment may include potassium supplementation,
Infectious diarrhea Anorexia with careful monitoring for rebound hyperkalemia. Oral
potassium supplementation is the preferred mode of treat-
Short bowel syndrome
ment for this reason. However, if serum potassium is less
Sweating than 2 mEq/L (2 mmol/L) and electrocardiographic changes
Laxative abuse characteristic of hypokalemia are evident, intravenous
potassium is indicated. The suggested protocol is to admin-
Thyrotoxic periodic paralysis occurs more commonly in ister 30 mEq of oral potassium every 2 hours until improve-
males, with a higher prevalence in Asians, usually of Chinese, ment begins, with a maximum of 90 mEq in 24 hours. Both
Japanese, Vietnamese, Filipino, or Korean descent. Although serum potassium and heart activity should be monitored.
attacks can occur at any time during the day, most tend to Propranolol (1 mg intravenously or 3 mg/kg orally) is in-
occur at night or during the early morning hours, which has dicated for patients with symptoms of hyperthyroidism
prompted the terms “nocturnal paralysis” or “night palsy”. and hypokalemia that persist despite potassium supplemen-
Attacks occur in the presence of elevated thyroid hormone tation. Maintaining a euthyroid state and avoiding high-
and cease once values normalize. The exact mechanism of glycemic meals can prevent repeated attacks.
thyrotoxic periodic paralysis is not well understood but is
believed to be related to thyroid hormone increasing tissue Lessons for the Clinician
responsiveness to b-adrenergic stimulation. • The vast array of differential diagnoses for acute paralysis
Most cases of hyperthyroidism are due to Graves’ disease, in children warrant a thorough evaluation.
but other potentially causative conditions are thyroiditis, • Hypokalemia can result from a variety of causes and can
toxic nodular goiter, toxic adenoma, TSH-secreting pituitary cause acute weakness. It is often a signal for the physician
tumor, and exogenous ingestion of thyroid hormone or to consider periodic paralysis.
iodine. The age of onset is typically 20 to 40 years but • The severity of paralysis correlates to the degree of
has been described in adolescents. Clinical features of hypokalemia, irrespective of the thyroid hormone
hyperthyroidism may precede the onset of periodic paralysis values.
by months to years, occur at the same time, or develop after • Thyrotoxicosis should be considered in the differential
the diagnosis is made. Although cases of thyrotoxic periodic diagnosis when adolescents present with recurrent epi-
paralysis are characterized by tachycardia and hyperther- sodes of weakness and hypokalemia.
mia, this patient did not exhibit these changes in vital Suggested readings for this article are at http://pedsinreview.
signs on presentation. His heart rate was 78 beats per minute, aappublications.org/content/37/4/172.

174 Pediatrics in Review


3 Altered Mental Status in a 14-Year-Old Girl

Allison Lowe Guimera, MD,* Deepa Kulkarni, MD*


*UCLA Mattel Children’s Hospital, Los Angeles, CA.

PRESENTATION
AUTHOR DISCLOSURE Drs. Lowe Guimera A 14-year old girl presents to the emergency department with a 1-day history of
and Kulkarni have disclosed no financial
altered mental status. She has severe autism spectrum disorder, learning
relationships relevant to this article. This
commentary does contain a discussion of an disability, and hyperactive behavior for which she takes risperidone and trans-
unapproved/investigative use of a dermal clonidine. She is ambulatory and nonverbal at baseline, but she has been
commercial product/device. increasingly somnolent and unable to walk or eat over the past few hours. There
is no history of witnessed head trauma or seizure-like activity. She has no
associated fevers, respiratory symptoms, vomiting, or diarrhea.
On physical examination, her temperature is 36.6°C (97.9°F), heart rate is
53 beats per minute, respiratory rate is 10 breaths per minute, blood pressure is
88/52 mm Hg, and oxygen saturation is 100% in room air. She is somnolent,
minimally arousable, and opens her eyes briefly to painful stimuli. Her head is
atraumatic, neck is supple, and pupils are pinpoint and minimally reactive to
light. She has regular cardiac rhythm, unlabored shallow breathing, and clear
lungs to auscultation. Her extremities are well perfused and atraumatic. Her
reflexes are normal.
Laboratory evaluation reveals a normal complete blood cell count and
complete metabolic panel. Electrocardiography shows sinus bradycardia.
Computed tomography scan of the brain yields results within normal limits.
A urinary toxicology screen is negative. After administration of 2 normal
saline boluses and 2 doses of naloxone, she has minimal improvement of
hypotension and mental status. Additional evaluation and history reveal the
diagnosis.

DISCUSSION

After further questioning, her parents report that she frequently ingests nonfood
objects, and closer physical examination revealed that her transdermal clonidine
patch was absent, with evidence of excoriation at its previous site. She was
admitted to the hospital with presumed clonidine poisoning due to ingestion of
the patch.
Clonidine is an a-2 agonist that acts centrally in the brainstem to reduce
sympathetic outflow, resulting in decreased peripheral vascular resistance, renal
vascular resistance, heart rate, and blood pressure. Clonidine has multiple clinical
indications for use in the pediatric population, including treatment of hyperten-
sion, attention-deficit/hyperactivity disorder, conduct disorder, and Tourette
syndrome. It is frequently used off-label for other behavioral problems, sleep

Vol. 37 No. 4 APRIL 2016 175


disturbance, headaches, postoperative nausea and vomiting, Rapid identification is essential because several of the
opioid withdrawal, and pain modulation. initial treatments are time-sensitive. Specifically, acti-
Clonidine is available in regular and extended-release vated charcoal, if indicated, is most effective when given
oral formulations as well as transdermal patches for more within 1 hour of exposure to prevent systemic absorption.
sustained release. The transdermal patch contains a reser- In addition, dermal and ocular decontamination must be
voir of medication that moves down a concentration gradi- performed expeditiously for toxins that have rapid ab-
ent toward the patient. To maintain a sufficient gradient for sorption. Other methods of decontamination that should
prolonged drug delivery, the reservoir contains an excess be considered include whole bowel irrigation, which is
amount of drug. In fact, more than 50% of the active drug is effective for nonadsorbed agents as well as sustained-
present in patches removed after 7 days of wear, making release preparations. Certain agents may also benefit
even disposed patches a threat for toxic ingestion. from urine alkalization or hemodialysis to enhance
elimination.
The Condition This patient’s altered mental status and depressed vital
Clonidine overdose can occur from exploratory ingestion in signs raised the possibility of a toxidrome. Her blood
young children, suicidal ingestion, or administration error. pressure was initially stabilized with normal saline boluses.
The severity of symptoms in an overdose is highly variable Atropine can be administered to reverse symptomatic bra-
and depends on the child’s age, the formulation, the route of dycardia caused by atrioventricular node depression in
exposure, and the dose delivered. Most signs of toxicity clonidine toxicity, but it was not needed in this case.
develop 1 to 4 hours after oral formulation ingestion and Naloxone was administered but had minimal effect, mak-
may continue past 4 hours for extended-release oral formu- ing opioid toxicity less likely. Clonidine toxicity has a
lations or patch exposure. The symptomatic period follow- variable response to naloxone, mediated by clonidine’s
ing an ingested transdermal patch is unpredictable because cross-reactivity with potassium channels on opioid m recep-
medication may be erratically released while passing through tors. Dermal decontamination for this patient was ensured
the gastrointestinal tract. with a thorough physical examination, which demonstrated
Classically, clonidine overdose presents with symptoms the absence of additional clonidine patches. Whole bowel
of early hypertension followed by hypotension, bradycardia, irrigation was required to remove the clonidine source and
respiratory depression, central nervous system depression, prevent further gastrointestinal absorption. Elective intuba-
and miosis. In larger overdoses, cardiac conduction defects, tion would have been required for safe administration via
apnea, coma, and seizures may occur, but death is rare. the nasogastric route, given this girl’s depressed mental
These symptoms can easily be confused with other toxic status and risk of aspiration. Therefore, an enema was
ingestions, including ethanol, opiates, benzodiazepines, performed instead, followed by a continuous docusate rectal
barbiturates, and sedative agents. Clonidine overdose also infusion. Two days after admission, the clonidine patch was
has features similar to hypoglycemia, traumatic brain injury, found in her stool. She recovered well and was discharged
the postictal state, meningitis or encephalitis, acidemia, and the following day with normal vital signs and baseline
uremia. Given the broad differential diagnosis, a thorough mental status.
history and physical examination are essential before pur-
suing diagnostic studies. Lessons for the Clinician
• A thorough history and physical examination are essential
Management for all patients presenting with a suspected ingestion
Initial management of all toxic ingestions begins with stabi- because initial laboratory data may not lead to a definitive
lizing the patient, focusing on airway, breathing, and circula- diagnosis.
tion. Evaluation should include blood glucose, complete • Transdermal medications, even after discarding, may be
metabolic panel, electrocardiography, urine toxicology screen, applied or ingested by young children, leading to inad-
and urine pregnancy test in adolescent females. All patients vertent overdoses.
also should be evaluated for coingestants such as aspirin and • Clonidine poisoning is rarely life-threatening and is
acetaminophen. However, results vary, depending on time and treated with decontamination, naloxone, and supportive
type of exposure, and may even be normal. Consultation with care.
poison control is, therefore, important if clinical suspicion is Suggested readings for this article are at http://pedsinreview/
high to assist with the recognition of a potential toxidrome. aappublications.org/content/37/4/175.

176 Pediatrics in Review


in
Brief
Chagas Disease
Aaron W. Tustin, MD, MPH,* Natalie M. Bowman, MD, MPH†
*Department of Environmental Health Sciences, Johns Hopkins Bloomberg School of Public Health, Baltimore, MD.

Department of Medicine, Division of Infectious Diseases, University of North Carolina at Chapel Hill, Chapel Hill, NC.

AUTHOR DISCLOSURE Dr Tustin has Approximately 8 million people worldwide are infected by the protozoan parasite
disclosed no financial relationships relevant to
Trypanosoma cruzi, the causative agent of Chagas disease. After a latent period that
this article. Dr Bowman has disclosed that she
receives grant support from NIH (NIAID) and can last years or decades, 10% to 30% of infected people develop serious compli-
BWF/ASTMH to research Chagas disease cations, such as cardiomyopathy or gastrointestinal dysfunction. Contrary to popular
(Grants K23 AI113197-02, P30 AI50410, and
belief, Chagas disease is not solely a vector-borne infection of Latin America.
others). This commentary does contain a
discussion of an unapproved/investigative Clinicians in nonendemic regions must be aware of the potential for childhood
use of a commercial product/device. T cruzi infections.
Humans typically acquire the parasite through contact with the infected feces
of blood-feeding triatomine insects. Vector-mediated transmission occurs in
endemic regions that extend from the southern United States to the southern
cone of South America. T cruzi can also be transmitted in food and beverages
contaminated with triatomine feces, via infected organ transplants and blood
transfusions, and vertically from mother to child. In the United States, human
contact with triatomines is minimal, and blood donations are screened for T cruzi.
However, more than 300,000 US residents, mostly immigrants from Latin
America, may harbor chronic T cruzi infections. Many of those infected are
children and reproductive-age women.
Vertical transmission is an underrecognized problem. Children born to
infected mothers have an approximately 5% chance of acquiring the parasite.
Although there has been only one reported case of congenital T cruzi infection in
the United States (in an infant born in 2010 to a Bolivian mother), several
hundred undetected congenital T cruzi infections are estimated to occur in the
United States each year. Diagnosis is difficult because most infected newborns are
asymptomatic. A minority of infected infants demonstrate nonspecific signs and
symptoms, including low Apgar scores, low birthweight, respiratory distress,
Congenital Chagas Disease:
Recommendations for Diagnosis,
hepatosplenomegaly, anasarca, pericardial and pleural effusions, and meningo-
Treatment and Control of Newborns, encephalitis. The presentation can easily be confused with neonatal sepsis or
Siblings and Pregnant Women. Carlier Y, TORCH infections (which include toxoplasmosis, rubella, cytomegalovirus, and
Torrico F, Sosa-Estani S, et al. PLoS Negl Trop
herpes viruses). Clinicians may overlook T cruzi in the differential diagnosis,
Dis. 2011;5(10):e1250
especially in nonendemic areas.
Trypanosoma cruzi and Chagas Disease in
To improve detection of congenital infections, the World Health Organization
the United States. Bern C, Kjos S, Yabsley MJ,
Montgomery SP. Clin Microbiol Rev. recommends antenatal T cruzi screening in all pregnant women who have ever
2011;24(4):655-681 lived or received a blood transfusion in an endemic region. Infected women are
Congenital Transmission of Chagas not treated prenatally to prevent vertical transmission because antitrypanosomal
disease - Virginia, 2010. Centers for Disease medications are contraindicated during pregnancy and breastfeeding. Instead,
Control and Prevention (CDC). MMWR Morb children born to T cruzi-infected mothers should be tested during infancy and
Mortal Wkly Rep. 2012;61(26):477-479
treated if necessary. Unfortunately, prenatal screening recommendations are not
Congenital Chagas Disease: An Update. widely followed, even in endemic countries. Many at-risk infants are born to
Carlier Y, Sosa-Estani S, Luquetti AO, Buekens
P. Mem Inst Oswaldo Cruz. 2015;110 mothers with unknown T cruzi status. Pediatricians should consider testing these
(3):363-368 newborns for Tcruzi, especially if they are symptomatic. Diagnostic testing should

Vol. 37 No. 4 APRIL 2016 177


be offered to siblings of any child diagnosed with Chagas adults. Cure rates of nearly 100% are observed in congenitally
disease, and the mother should also be referred for treat- infected infants who receive treatment within the first post-
ment evaluation. natal year. The cure rate decreases to approximately 60% in
Newborns at risk for T cruzi infection should be tested older children with chronic infections. Prompt detection and
with polymerase chain reaction (PCR), the preferred diagnos- treatment of congenital T cruzi infection is, therefore, essen-
tic technique in children younger than age 9 months. PCR is tial to avoid future complications.
more sensitive than microscopy-based techniques to detect COMMENT: I found this In Brief fascinating because I
T cruzi in blood, although it may not be available in resource- have not been familiar with the presentation of Chagas
limited areas. If the initial test result is negative, PCR should disease nor considered it in the differential diagnosis of
be repeated at 4 to 6 weeks of age because parasitemia may patients. Yet, the World Health Organization estimates that
increase over the first postnatal month. Infants who lack Chagas disease is the most prevalent parasitic disease in the
detectable parasitemia should undergo serologic testing after American continents, including North, Central, and South
age 9 months to confirm the absence of infection. Serologic America.
methods such as enzyme-linked immunosorbent assay As the authors noted, the presentation in newborns is
(ELISA) and immunofluorescent antibody test (IFA) should diverse, making diagnosis challenging for those who are not
not be used in children younger than 9 months, in whom familiar with the wide range of possibilities. Congenital
circulating maternal antibodies can cause false-positive Chagas disease must be considered in infants born to moth-
results. After 9 months of age, serology becomes the diag- ers who are from endemic areas, such as the southern portion
nostic method of choice because parasitic DNA is often of South America: Brazil, Paraguay, Argentina, and Bolivia.
undetectable by PCR during the chronic stage of infection. Early identification and treatment can prevent progression to
Positive serology should be confirmed with a second assay more serious manifestations associated with chronic infec-
because these tests have imperfect sensitivity and specificity. tion. Chronic Chagas disease can affect the heart, leading to
All children diagnosed with T cruzi infection should cardiomyopathy, associated arrhythmias, and cardiac dys-
receive antitrypanosomal therapy with either benznidazole function. Although less common, gastrointestinal involve-
or nifurtimox. Although these medications are not approved ment can affect the esophagus or colon and lead to motility
by the US Food and Drug Administration (FDA) in the disorders. Because treatment is available and effective,
United States, they can be obtained from the Centers for diagnosis in both congenital and chronic states is impor-
Disease Control and Prevention under investigational per- tant to patient quality of life and prevention of resultant
mits. Standard therapy is a 60- to 90-day course of benzni- morbidity.
dazole 5 to 7 mg/kg per day or nifurtimox 10 to 15 mg/kg per
day divided in 2 to 3 daily doses. Both drugs have a high rate – Janet Serwint, MD
of adverse effects but are tolerated better in children than Associate Editor, In Brief

178 Pediatrics in Review


in
Brief
Dengue and Chikungunya
Paul J. Lee, MD,* Leonard R. Krilov, MD†
*Children’s Medical Center, Winthrop-University Hospital, Mineola, NY.

State University of New York at Stony Brook School of Medicine, Stony Brook, NY.

AUTHOR DISCLOSURE Dr Lee has disclosed Dengue is caused by 4 related but distinct flaviviruses, DENV1 – 4, which result
that he was a speaker for Novartis Vaccines.
in 390 million infections annually, with one-third of the world’s population at
His relationship with them ended in June
2015. Dr Krilov has disclosed that he is site risk. Most flaviviruses are transmitted by arthropods, such as mosquitoes and
principal investigator for a meningococcal B ticks, and are responsible for diseases such as yellow fever, West Nile fever, and
vaccine trial for Pfizer and that Pfizer provides
Japanese encephalitis. DENV is primarily transmitted by the bite of an infected
funding to his institution; site principal
investigator for an observational study of female Aedes aegypti mosquito, which is endemic to every global region except
respiratory syncytial virus (RSV) infections in Europe and Antarctica. Dengue primarily occurs in South and Southeast Asia,
preterm infants for AstraZeneca and that
Africa, tropical South and Central America, and the Caribbean. In the United
AstraZeneca provides funding to his
institution; and site principal investigator for a States, dengue is a nationally notifiable disease, with 677 cases reported to the
clinical trial of a humanized monoclonal Centers for Disease Control and Prevention (CDC) in 2014. It is endemic in
antibody to prevent RSV in high-risk preterm Puerto Rico, the US Virgin Islands, and American Samoa. Most US cases result
infants for Regeneron. This commentary does
not contain discussion of an unapproved/ from travel to an endemic area, but locally acquired dengue has been documented
investigative use of a commercial product or in Texas and south Florida.
device. Seventy-five percent of DENV infections in all age groups are asymptomatic,
but a spectrum of disease is seen in the remaining 96 million patients, ranging
from self-limited fever to life-threatening shock. Classic dengue fever begins 4 to 7
days after an infected mosquito’s bite and typically presents as a flulike illness
with a temperature greater than 38.5°C (101.3°F), headache, retro-orbital pain,
and severe myalgias, from which its eponym, “break-bone fever,” arose. A fine,
transient macular rash, nausea, and vomiting may also occur. However, infection
may present atypically; the classic findings occur in fewer than 60% of patients.
With the exception of high temperatures, children are often less symptomatic
CDC Health Information for International than adults during the first phase of infection, which lasts 3 to 7 days. Physical
Travel 2016. Centers for Disease Control and
Prevention. New York, NY: Oxford University examination may reveal a palpable liver and petechiae and bruising, caused by
Press; 2016. Also available at: http://wwwnc. moderate thrombocytopenia. Other laboratory findings include leukopenia and
cdc.gov/travel/page/yellowbook-home. an aspartate aminotransferase value less than 1,000 U/L (16.7 mkat/L).
Accessed October 31, 2015
Most patients subsequently recover without complications, but about 1%,
Dengue. Guzman MG, Harris E Lancet. usually children and young adults, develop systemic vascular leak after defervesc-
2015;385(9966):453–465 doi: 10.1016/S0140-
ing. This leakage causes increased hemoconcentration, hypoproteinemia, pleural
6736(1460572-9
effusions, and ascites, resulting in more serious disease, referred to as dengue
Dengue. Simmons CP, Farrar JJ, Nguyen V,
hemorrhagic fever (DHF) or dengue shock syndrome (DSS). DHF is defined by
Wills B. N Engl J Med. 2012;366(15):1423–1432.
doi: 10.1056/NEJMra1110265 a triad of findings: plasma leakage, thrombocytopenia, and hemorrhage. The
bleeding is most notable in the skin, but hematemesis, melena, menorrhagia, and
Infectious Disease Emergencies in Returning
Travelers: Special Reference to Malaria, epistaxis can also occur. Children rarely have clinically significant bleeding,
Dengue Fever, and Chikungunya. Wattal C, but adults can have substantial hemorrhage, even with minor plasma leakage.
Goel N. Med Clin North Am. 2012;96 DSS occurs in patients with DHF when the pulse pressure narrows to less than
(6):1225–1255. doi: 10.1016/j.mcna.2012.08.004
20 mm Hg from plasma volume loss and peripheral vascular collapse begins. DSS
Chikungunya Virus and the Global Spread is particularly insidious because patients initially appear well, with normal or
of a Mosquito-borne Disease. Weaver SC,
Lecuit M. N Engl J Med. 2015;372(13): elevated systemic blood pressure. However, once they develop hypotension,
1231–1239. doi: 10.1056/NEJMra1406035 irreversible shock and death can occur despite aggressive resuscitation. Clinicians

Vol. 37 No. 4 APRIL 2016 179


should closely monitor patients for signs of impending vulnerability of a population that has no prior immunity
shock, such as persistent vomiting, increasingly severe to CHIKV, efficient transmissibility has led to large-scale
abdominal pain, tender hepatomegaly, mucosal bleeding, outbreaks. One outbreak in northeastern Italy in 2007
lethargy, or restlessness. Helpful diagnostic indicators are was caused by the local Aedes aegypti and Aedes albopictus
the presence of effusions and increasing or high hematocrit mosquitoes biting an infected traveler and then infecting
as the platelet count rapidly declines. The altered vascular the local population with an adapted CHIKV that had
permeability quickly resolves after 48 to 72 hours, and its enhanced transmissibility. Both of these mosquito spe-
pathogenesis remains unclear. DHF and DSS do not occur cies are present in the United States, and at least 11
with the initial DENV infection. endemic cases have been documented. Estimates of CHIKV
Immunity to one serotype after infection may predispose incidence and prevalence are not as well defined as for
individuals infected with another serotype to severe dengue. DENV.
Therefore, most travelers, except for those frequently vis- In sharp contrast to DENV, 85% of CHIKV infections are
iting family and relatives, are unlikely to develop what the symptomatic, with rapid onset of a temperature of 39° to 40°C
World Health Organization now classifies as severe dengue. (102.4° to 104°F) after a brief incubation of 2 to 4 days.
Dengue is usually diagnosed clinically because most Polyarthralgia begins 2 to 5 days after fever, frequently
endemic areas lack access to clinical laboratories. Even in involving more than 10 joint groups. The arthralgia tends
developed countries, confirmation is slow and limited to to be symmetric and more common in distal than proximal
specialized reference laboratories. The gold standard for joints, especially large joints, although the axial skeleton
diagnosis, although it is not helpful in patient care, remains may be involved. Patients are often immobilized by pain.
testing paired acute and convalescent (obtained 10 to 14 days Macular or maculopapular rash is also common, appearing
later) serum with a DENV hemagglutinin inhibition assay. 3 or more days after fever and lasting 3 to 7 days. The rash
Newer tests, such as DENV nonstructural protein 1 (NS1) begins on the trunk or extremities, can be patchy or diffuse,
or immunoglobulin (Ig)M enzyme-linked immunosorbent may involve the face as well as the palms and soles, and may
assays or RNA reverse transcription-polymerase chain reac- be pruritic in up to 50% of patients. Children may develop
tion (RT-PCR), can now diagnose dengue quickly. bullous lesions, but petechiae and bruising are uncommon.
Dengue management is largely supportive. Early recog- Gastrointestinal symptoms and erythematous ears from
nition of shock and careful, frequent assessment of pulse, CHIKV chondritis occur as well.
blood pressure, hematocrit, and urine output, with admin- Similar to DENV, CHIKV is usually diagnosed clinically.
istration of oral and intravenous fluids and blood products to Marked lymphopenia is characteristic as well as throm-
maintain adequate intravascular volume, is critical. Cur- bocytopenia, elevated aspartate aminotransferase/alanine
rently there is no effective antiviral agent for DENV. Pre- aminotransferase, and hypocalcemia. RT-PCR can diagnose
vention is based on mosquito control measures, although a CHIKV in the first week of illness, and IgM can be positive
recombinant live attenuated tetravalent vaccine (CYD-TDV) before the fifth day of illness and persist for months. A
may be approved in 2016. fourfold rise in IgG between acute and convalescent serum
Chikungunya fever is caused by an alphavirus (CHIKV), also confirms CHIKV. Treatment of CHIKV is supportive,
which was isolated during an epidemic in 1952 in Tanzania. and prevention is through mosquito control, with vaccine
Its name is the Makonde word for “that which bends up- research still in early stages.
ward,” describing the bent posture patients that assume Differentiating DENV and CHIKV can be difficult be-
from the severe arthralgia CHIKV causes. In 2004, CHIKV cause both are acute systemic febrile illnesses with rash that
made headlines when it spread from Africa to South Asia, occur in the same world regions. The Table highlights im-
causing a number of epidemics affecting millions of people. portant differences between the two. The differential diag-
In October 2013, CHIKV appeared in St. Martin and rapidly nosis also includes malaria, rickettsial disease, typhoid,
spread throughout the Caribbean and Central America, with leptospirosis, Epstein-Barr virus infection, measles, rubella,
1.7 million cases reported within 2 years, 3,270 of which and other hemorrhagic fever viruses.
were reported in the United States. COMMENT: Dr Anthony Fauci, Director of the National
Although CHIKV is spread by the same mosquitoes as Institute of Allergy & Infectious Diseases, has called mos-
DENV, it can be more readily transmitted. Paired with the quitoes the most destructive animals on earth, without any

180 Pediatrics in Review


TABLE. Differentiating Dengue from Chikungunya
DENGUE CHIKUNGUNYA

Fever Longer duration (5–7 days) High-grade, shorter duration (3–5 days)
Arthralgia Common Common, but characteristically severe polyarthralgia
Rash Fine, faint transient macules; spares the Maculopapular, can be bullous, pruritic, and on the
palms/soles face, palms/soles
Petechiae/ecchymoses Common Uncommon
Joint swelling Very uncommon Common
Abdominal pain Common Uncommon
Respiratory symptoms Can occur Not seen
Leukopenia Common Uncommon
Symptoms Resolve within 1 week, except for fatigue Frequently persist
Severe complications Can occur Rare

apparent redeeming characteristics. Within the family of newly infected travelers return to homelands infested with
mosquitoes, the Aedes, along with the Anopheles (the vector Aedes, the epidemic becomes a pandemic. More than 20
for malaria), is at the top of the list of villains. In addition to countries in the Americas have now reported indigenous
dengue and chikungunya, Aedes species carry the arbovi- cases of Zika virus. As of this writing, no locally acquired
ruses responsible for yellow fever, West Nile fever, eastern Zika infections have been reported in the United States, but
equine encephalitis, and now the most recent addition to the several travelers returning from areas of active infection
catalog of pandemic threats: Zika virus. have been identified with the virus, and possibly some
Initially identified serendipitously in Uganda in 1947, the southern states harbor mosquitoes that could become a
Zika virus spread locally in West Africa and then eastward reservoir for the regional spread of Zika.
to the Pacific, causing outbreaks of infection in Micronesia The Centers for Disease Control and Prevention have
and Southeast Asia that were characterized by fever, rash, urged women who are pregnant or trying to become preg-
arthralgias, and conjunctivitis. Until recently it raised no nant to avoid travel to areas where Zika virus has been
special concern because the virus appeared to cause only identified and has issued guidelines for the management
transient illness, if any at all; up to 80% of infected indi- of pregnant women who may have been infected and for
viduals remained asymptomatic. However, in 2015, if I may the evaluation of infants with possible congenital infection.
paraphrase W.B. Yeats, all changed, changed utterly; a ter- Early in 2016, the World Health Organization (perhaps
rible pandemic was born. belatedly) declared the neurologic complications associated
By the year’s end, Brazil reported more than 1 million with Zika virus infection to be an international health
cases of Zika infection and with them came an alarming emergency, which is the same classification it gave to the
rise in cases of microcephaly as well as of Guillain-Barré Ebola epidemic in West Africa several years ago. As pedi-
syndrome. Brazil hosted the World Cup in 2014, and some atricians, we know the devastating consequences of micro-
have speculated that one of the myriad visitors from around cephaly. It is a terrible price to pay, even for the miracle of
the globe transported the Zika virus as unwanted baggage. modern air travel.
When an infected human is bitten by an Aedes mosquito, the
insect ingests the virus, and its next bite passes the virus on —Henry M. Adam, MD
to another human host – and so an epidemic is born. When Associate Editor, In Brief

Vol. 37 No. 4 APRIL 2016 181


Stay current
with these new pediatric
resources!

Visit

to order today!
ASAP!
Every article. Any device.
Anytime. Anywhere.

Want A Better

Wrangle
Way to

Thousands of
Pediatrics-Focused Studies? AAP GRAND ROUNDS
www.aapgrandrounds.org
Mission: To provide pediatricians with timely synopses and critiques of important new studies relevant to
pediatric practice, reviewing methodology, significance, and practical impact, as part of ongoing CME activity.

PEDIATRIC PULMONOLOGY & SLEEP MEDICINE


Vol. 30 No. 5 | Pages 49-60 | November 2013

Desktop. Tablet. Mobile.

Adenotonsillectomy for Obstructive Sleep Apnea


It’s powerful and it’s here, now! It’s a smart new benefit, free
Source: Marcus CL, Moore RH, Rosen CL, et al. A randomized behavior, and quality of life among school-aged children with OSA. Watchful
trial of adenotonsillectomy for childhood sleep apnea. N Engl J observation may be an alternative for childhood OSA given the absence of
Med. 2013;368(25):2366-2376; doi:10.1056/NEJMoa1215881 significant cognitive improvement and the fact that polysomnography normal-
ized in about half of the children in the watchful observation group.

I
nvestigators from multiple in-
stitutions conducted a random- PICO Commentary by
ized controlled trial to assess Question: Among children aged 5 to 9 Catherine Kier, MD, FAAP, Pediatrics, Stony Brook School of
the efficacy of adenotosillectomy in years with obstructive sleep apnea, does Medicine, Stony Brook, NY

A single, integrated network of journals and periodicals — AAP News,


adenotonsillectomy, compared to watchful Dr Kier has disclosed no financial relationship relevant to this commentary. This commentary does not contain a
children with obstructive sleep ap-

to all AAP members. Log on


observation, result in improved outcomes? discussion of an unapproved/investigative use of a commercial product/device.
nea (OSA) as part of the Childhood
Question type: Intervention Research on pediatric OSA has exponentially increased in the past decade.
Adenotonsillectomy Trial (CHAT).
Study design: Randomized controlled A recent systematic review of the epidemiology of pediatric OSA indicated a
Children 5 to 9 years old with a
score ≥2 on an obstructive apnea- prevalence of 4% to 11% based on parent questionnaire and 1% to 4% based
on diagnostic studies, with higher prevalence among boys, obese children,

Nearly 100 medical journals scanned


hypopnea index (AHI) or ≥1 on the obstructive apnea index were enrolled.
and black children.1 Neurocognitive, cardiovascular, metabolic, and quality

Pediatrics, Pediatrics in Review, Hospital Pediatrics, NeoReviews and


Study participants were randomized to early adenotonsillectomy (within
1 month after randomization) or to watchful observation. The primary of life consequences have been associated with pediatric OSA, especially if
outcome measure was change in attention and executive function based left untreated. Adenotonsillectomy is considered the first line of treatment.
on the Developmental Neuropsychological Assessment score. Secondary About half a million tonsillectomies in children are performed in the United

using your AAP ID and visit


outcome measures included polysomnographic, behavioral, symptomatic, States annually, many of these for OSA.2 The CHAT is the first multicenter
and quality of life measures collected at the beginning of the study and at randomized trial comparing adenotonsillectomy and watchful observation
7 months. Outcomes were compared between the 2 groups; outcomes were for pediatric OSA. CHAT has replicated the findings of a meta-analysis of 14

AAP Grand Rounds — all available in one intuitive interface that


also compared in several “high-risk” subgroups. case series, which showed normalization of polysomnography in about 80%

monthly by 49 pediatric subspecialty A total of 464 children were enrolled; outcome data were collected on 397
including 194 participants randomized to early adenotonsillectomy. There
was no significant difference between the 2 study groups with regard to
change in attention or executive function scores at 7 months post interven-
tion. However, the adenotonsillectomy group demonstrated significantly
of children following adenotonsillectomy.3
The multicenter CHAT study was well-designed, utilized standardized
measurements, and closely followed participants. Since younger children
and those with very severe OSA, prolonged desaturations, and ADHD on
medication were excluded, the findings cannot be generalized to these
groups. In addition, since reevaluation occurred at 7 months, it is possible

gateway.aap.org.
greater improvement in behavioral, quality of life, polysomnographic, and

works on all of your devices.


symptomotology measures than the watchful observation group. Polysom- that improvements in attention and executive function may not be evident
nography normalized in 79% of the adenotonsillectomy group compared to in that short time.4

experts resulting in 134 subject collections 46% of the watchful waiting group. While the CHAT study certainly provides promising evidence that
Analyses in higher risk subgroups including black children, obese children, adenotonsillectomy improves behavioral, symptom, quality of life, and
and children with AHI scores above the median showed that polysomnogra- polysomnographic measures in at least some children with OSA, it leaves
phy did not normalize as frequently many questions unanswered. Future studies will need to assess the value of
compared to those in the correspond- adenotonsillectomy in younger children and those with more severe OSA as
ing lower risk group. This was true in well as evaluating the impact over a longer follow-up period.
INSIDE both adenotonsillectomy and watch- References
Cranial Autonomic Symptoms in ful observation groups. However, the 1. Lumeng JC, et al. Proc Am Thorac Soc. 2008;5(2):242-252; doi:10.1513/pats.200708-

and an archives vault with over 2,000 articles.


Pediatric Migraine adenotonsillectomy group improved 135MG
Long-term Benefits of Selective Dorsal more with regard to polysomnogra- 2. Baugh RF, et al. Otolaryngol Head Neck Surg. 2011;144(1 Suppl):S1-S30;
Rhizotomy doi:10.1177/0194599810389949
phy, behavior, and clinical measures as
Juvenile Idiopathic Arthritis and Live 3. Brietzke SE, et al. Otolaryngol Head Neck Surg. 2006;134(6):979-984; doi:10.1016/j.
Virus Vaccines
compared to the watchful observation otohns.2006.02.033

It’s tailored to your interests.


Cognitive and Behavior Outcome 8
group in these higher risk subgroups. 4. Brouillette RT. N Engl J Med. 2013;368(25):2428-2429; doi:10.1056/NEJMe1305492
Years Following ECMO in the NICU The authors conclude that although
Key words: obstructive sleep apnea, adenotonsillectomy, watchful observation
Disparity in Receipt of Educational adenotonsillectomy did not result in
Services for Autism significantly greater cognitive improve-
Weighing the Evidence: Logistic ments compared to watchful observa- Visit www.GrandRoundsBlog.org
Regression
tion, it resulted in significant improve- to read a post about this article
Is School an Infection Risk for Children appearing this month.
With Cancer? ments in polysomnography, symptoms,
The Costs of Child Maltreatment

AAP Gateway will help you manage AAP journals and periodicals,
What Is Different About Children Who
The AAP Policy on Disclosure of Financial Relationships and Resolution of Conflicts of Interest for AAP CME Activities is designed to issue of AAP Grand Rounds unless noted on the article or below. The AAP has taken steps to resolve any potential conflicts of interest.
Receive Palliative Care? ensure quality, objective, balanced, and scientifically rigorous AAP CME activities by identifying and resolving all potential conflicts of
Joseph Geskey, DO (Editorial Board Member) disclosed a Speaker’s Bureau with GlaxoSmithKline.
interest prior to the confirmation of service of those in a position to influence and/or control CME content.
Lane Palmer, MD (Editorial Board Member) disclosed a Speaker’s Bureau with Laborie.
Back Page: New Guidelines for All individuals in a position to influence and/or control the content of AAP CME activities, including editorial board members, authors, Philip Rosenthal, MD (Editorial Board Member) disclosed Research Grants with Bristol-Myers Squibb and Roche.
Diagnosis and Treatment of C difficile and staff, are required to disclose to the AAP and subsequently to learners that the individual either has no relevant financial Robert R. Wittler, MD (Editorial Board Member) disclosed a Speaker’s Bureau with Sanofi Vaccines.
relationship or any financial relationships with the manufacturer(s) of any commercial product(s) and/or provider(s) of commercial
services discussed in CME activities. None of the editors, authors, or staff had any relevant financial relationships to disclose for this

news, updates and clinical research that’s most relevant and


Subscribe to 2016 AAP Grand Rounds, important to you, based on your specialty and interests.
the Monthly Pediatric Literature Review
from the American Academy of Pediatrics It’s simple, intuitive and accessible.
Take full advantage of AAP Gateway and easily find, filter and sort
Choose an individual
subscription in Print and
OR Get a site license for your organization.
Order directly through the AAP.
over 50,000 articles with the all new search platform — now
Online or Online Only formats. PHONE 847/434-4000
including PubMed.
shop.aap.org/2016-AAP- EMAIL institutions@aap.org
Grand-Rounds ONLINE QUOTE go.aap.org/sitelicenseinquiryform
Over 70%
of children are
admitted to
general APRIL 2016

hospitals
Vol. 37 No. 4
www.pedsinreview.org

rather than
Hematopoietic Stem Cell
children’s Transplantation in
Children and Adolescents
hospitals. Guilcher

Does your pediatric patient care fit? Physical Abuse of Children


Glick, Lorand, Bilka

Syncope
Cannon, Wackel
AN OFFICIAL JOURNAL OF THE AMERICAN ACADEMY OF PEDIATRICS
JANUARY 2016 • VOLUME 6 • NUMBER 1
w w w.hospi t alpedia trics.or g

Toward High-Value Care:


COMMENTARY
A Quality Improvement
Initiative to Reduce
AN OFFICIAL JOURNAL OF THE AMERICAN ACADEMY OF PEDIATRICS
N O V E M B E R 2 015 • V O L U M E 5 • N U M B E R 11
w w w.hospi t alpedia trics.or g
Unnecessary Repeat
Complete Blood Counts
and Basic Metabolic Panels
on a Pediatric Hospitalist
Service
JOHNSON ET AL
Zika Virus – Pediatricians
Parent and Provider
Perspectives on Pediatric
Readmissions: What Can
We Learn About Readiness
Reducing Overutilization
of Testing for Clostridium
difficile Infection in a
Pediatric Hospital System:
A Quality Improvement
Initiative
Be Aware
for Discharge?

Rathore
BRITTAN ET AL KLATTE ET AL

An Examination of Reasons for Refusal


Physician-, Caregiver-, and of Newborn Vitamin K
Disease-Related Factors Prophylaxis: Implications for
Associated With Readmission Management and Education
From a Pediatric Hospital HAMRICK ET AL
Medicine Service
WALLACE ET AL

The Med-Peds Hospitalist


Workforce: Results From
the American Academy of
Pediatrics Workforce Survey
DONNELLY ET AL

Subscribe to Hospital Pediatrics®, the


Monthly Peer-Reviewed Journal from
the American Academy of Pediatrics
Choose a 1- or 2-year individual
subscription terms in Online Only
OR Get a site license for your organization.
Order directly through the AAP.
now indexed
in MEDLINE or Print and Online formats. PHONE 847/434-4000
shop.aap.org/Hospital-Pediatrics EMAIL institutions@aap.org
ONLINE QUOTE go.aap.org/sitelicenseinquiryform
Complementary, Integrative, and Holistic
Medicine: Integrative Approaches to Pediatric
Irritable Bowel Syndrome
Alycia Leiby, MD,*† Minal Vazirani, MD‡{
*Department of Pediatrics, Division of Pediatric Gastroenterology and Nutrition, Goryeb Children’s Hospital, Atlantic Health System, Morristown, NJ.

Department of Pediatrics, Sidney Kimmel Medical College of Thomas Jefferson University, Philadelphia, PA.

Siegler Center for Integrative Medicine, Barnabas Health ACC Barnabas Health System, Livingston, NJ.
{
Rutgers – New Jersey Medical School, Newark, NJ.

INTRODUCTION

Irritable bowel syndrome (IBS) occurs commonly in pediatrics, with the preva-
lence estimated at 6% to 14%. (1)(2) The Rome criteria define pediatric IBS as
abdominal pain that improves with defecation and/or onset associated with a
change in frequency or form of stool. (3) In addition, no evidence of an in-
flammatory, anatomic, metabolic, or neoplastic process explains the symptoms.
The cause of IBS is attributed to a combination of factors, including an altered gut
microbiome, low-grade mucosal inflammation, visceral hypersensitivity, abnor-
mal motility, psychosocial stressors, and genetic predisposition. (4) Recognition
of these factors serves as the foundation for the biopsychosocial approach to
treating pediatric IBS.
A successful treatment plan starts with a strong patient-parent-physician
relationship in which the child’s pain is validated and the physician compas-
sionately approaches the distress that IBS can cause. Multiple studies have
shown that children with IBS have a lower quality of life (QOL) that is
comparable to that seen with nonfunctional gastrointestinal conditions. (2)
(5) Conventional treatment includes education and reassurance, cognitive
behavioral therapy (CBT) for some patients, and consideration of antispasmodic
or antidepressant medications. In practice, the lack of strong evidence, potential
for adverse effects, and parental concern limit the utility of conventional
pharmacologic options.
Complementary and alternative medicine (CAM) use in pediatric gastroen-
terology is common and higher in patients with IBS than nonfunctional gastro-
intestinal conditions. (6) This article reviews the evidence for CAM therapy in IBS
based on adult and pediatric data available in the English language.

PROBIOTICS

The World Health Organization defines probiotics as “live micro-organisms


which, when administered in adequate amounts, confer a health benefit on
AUTHOR DISCLOSURE Dr Leiby has disclosed
the host.” Probiotics adhere to the intestinal epithelium and produce antibiotic that she is on the Speaker’s Bureau for Nestle
substances that alter the cytokine profile to an anti-inflammatory state as well as Infant Nutrition. Dr Vazirani has disclosed no
financial relationships relevant to this article.
short-chain fatty acids from poorly digested carbohydrates that affect gut motility.
This commentary does not contain a
(7) A recent systematic review found the number needed to treat (NNT) to improve discussion of an unapproved/investigative
IBS symptoms with probiotics was 7. (8) use of a commercial product/device.

e10 Pediatrics in Review


“VSL#3” is a multistrain probiotic preparation studied in global improvement of IBS (NNT ¼ 7) but bran did not.
IBS and inflammatory bowel disease that contains Bifido- (16) Starting with a low dose of soluble fiber and titrating
bacterium breve, B longum, B infantis, Lactobacillus paracasei, up while watching for increased gas may be helpful.
L bulgaricus, and Streptococcus thermophilus. This prepara- Adverse reactions to food proteins can represent a food
tion, as well as the strains B infantis and L rhamnosus GG, allergy, with a specific immune response, rather than food
have the strongest data to support their use in both adult and intolerances/sensitivities, which cause symptoms but do
pediatric IBS (Table 1) not have an established immunologic mechanism. (17) A
Probiotic use is based on the concept that the pathophys- recent study showed a greater number of children reporting
iology of IBS involves an altered microflora, (9) although recurrent abdominal pain as a symptom of food intolerance
current study data are limited by the heterogeneity of the (20%) compared with those who had a true allergy (2.3%).
strains and their dosing, which future research may clarify. (18) Understanding the mechanism of food intolerance
Despite these limitations, probiotic use for IBS is consid- presents many challenges. Multiple macronutrients in
ered a safe and well tolerated treatment option, based on one food may produce symptoms, such as the carbohydrate
Grade B evidence. (lactose), protein (casein and whey), and fat in dairy products
and gluten protein versus wheat starch in bread. A few
studies have suggested that gluten may be a cause of IBS
DIET
symptoms in patients without celiac disease. (19)(20) How-
More than 90% of adolescents with IBS report that their ever, a more recent study did not show a gluten effect after
symptoms are triggered by ingestion of certain foods, with reduction of fermentable short-chain carbohydrates, sug-
common culprits being large meals, fatty foods, and dairy gesting that IBS symptoms may be due to the poorly
products. (10) Parents often report that their child’s symp- absorbed starches in wheat. (21)
toms are food-related, and a recent study demonstrated that Recently, high-quality evidence has emerged suggesting
food-related IBS symptoms were associated with lower QOL that a low-FODMAPs (fermentable oligosaccharides [ fruc-
as compared to patients who had IBS without food-related tans, such as in wheat], disaccharides [lactose], monosac-
symptoms. (11) charides [ fructose] and polyols [sugar alcohols: sorbitol])
Fiber consists of unabsorbed carbohydrates that increase diet reduced IBS symptoms in adults. (22) FODMAPs are
fecal water content and bulk. Fiber is fermented to varying poorly absorbed, fermentable short-chain carbohydrates
degrees in the colon, thus influencing the microbiome. The that act as substrates for bacterial metabolism that produces
differences in fermentation and solubility alter laxation and gas, leading to colonic distention and resulting in bloating
gas production, with the potential adverse effects of bloating and pain. Results of the first double-blind, randomized trial
and flatus. (12) Early studies of fiber for IBS treatment sug- in pediatrics are encouraging. (23) A total of 45 children with
gested that although bowel frequency may have improved, IBS were randomized to a high-FODMAP (50 g/day) ver-
pain did not. (13)(14)(15) However, a recent meta-analysis that sus low-FODMAP (9 g/day) diet. (23) Thirty-three patients
included 14 randomized, controlled trials showed that solu- completed the crossover design and reported significant
ble fiber, such as psyllium or wheat dextrin, had benefit for improvement in bloating, nausea, and abdominal pain

TABLE 1. Probiotics Studied for Irritable Bowel Syndrome


PROBIOTIC DOSE AND ADMINISTRATION STUDY DESIGN RESULTS

VSL#3 43
450-900  10 CFU vs placebo 
8
Randomized, double-blind, placebo- Improved global assessment of relief
6 wk controlled (pediatric) (P < .05)
Bifidobacterium 1  108 CFU once daily vs placebo  Randomized, double-blind, placebo- Reduced abdominal pain (P < .03)
infantis44 4 wk controlled (adult)
Lactobacillus 3  109 – 1  1010 CFU twice daily Meta-analysis (pediatric) Relative risk 1.7, 95% confidence
rhamnosus GG45 vs placebo for 4-8 wk interval 1.27-2.27, number needed
to treat 4, 95% confidence interval
3-8

CFU¼colony-forming units.

Vol. 37 No. 4 APRIL 2016 e11


(P < .01). The low-FODMAP diet eliminates certain fruits response to dietary elimination and assist in avoiding
and vegetables, wheat, high-fructose corn syrup, legumes, overly restrictive diets.
and any lactose-containing dairy products. A strategy to allow
the least restrictive diet may involve initial strict elimination
HERBS/BOTANICALS
of high-FODMAP foods with gradual re-challenge to deter-
mine the level of intake that maintains symptom control. A variety of herbs have been studied for IBS, including
Food-specific immunoglobulin G (IgG) antibodies are turmeric, peppermint, and artichoke leaf extract, with some
physiologically produced in response to food exposure. suggestion of benefit. Iberogast, a combination of 9 herbs
However, one randomized, controlled trial (24) and two (chamomile flowers, bitter candytuft, angelica root, caraway
smaller nonrandomized trials (25)(26) showed some evi- fruits, milk thistle, lemon balm leaves, greater celandine,
dence of symptom reduction using IgG-specific elimination licorice root, and peppermint leaves) has been used for more
diets in adult patients with IBS. These data are preliminary, than 30 years in Germany for functional dyspepsia. Recently
and further research is needed before recommending clin- it has been studied for use in IBS, with positive preliminary
ical use of IgG testing in pediatrics. results. The lack of supplement ingredient regulation and
Dairy and higher-fat foods are common symptom trig- few studies in this area limit the ability to make recom-
gers for IBS. Affected patients are more sensitive to the mendations for pediatric IBS (Table 2).
effects of lactase deficiency that lead to symptoms of lactose
intolerance, such as bloating, increased flatus, and diarrhea. BIOBEHAVIORAL
(27) Intestinal sensitivity is increased by lipids in the small
bowel, causing discomfort for patients who are viscerally Cognitive Behavioral Therapy
hypersensitive at baseline. (28) The normal changes in mo- A number of biobehavioral options have been evaluated to
tility triggered by lipids (delayed gastric emptying, delayed manage IBS, including yoga, hypnotherapy, and CBT. CBT
small bowel transit, and accelerated colonic transit) also are aims to identify symptom triggers and help patients develop
heightened in those who have IBS. coping skills. This is a commonly accepted therapy for IBS,
When foods are convincingly linked with a patient’s with the American Academy of Pediatrics concluding that
symptoms and the family can manage the complexities of short-term use may improve pain and disability (29) and the
the diet while providing adequate nutrition, it is reason- American College of Gastroenterology (ACG) meta-analysis
able to limit offending foods. However, clinicians and concurring that it is an effective therapy for IBS with a NNT
families must consider the psychosocial challenges of of 3 to 4. (30) Evidence is rated as Grade A.
strict elimination diets and the risk of nutritional defi-
ciencies. Therefore, collaboration with a registered die- Yoga
titian may help to maintain a balanced diet. Further, Yoga has been studied in children for cardiorespiratory
although dietary restrictions may benefit some patients, fitness, mental health conditions, and IBS. (31) In 2006,
entirely skipping meals has been shown to worsen IBS 25 adolescents with IBS were randomized to a yoga inter-
symptoms. Based on Grade B evidence, reintroduction of vention that consisted of 1 live class and 4 weeks of a home
the “offending” foods may be considered to assess true instructional video. The yoga group had lower levels of

TABLE 2. Herbs and Botanicals Studied for Irritable Bowel Syndrome (IBS)

HERB DOSE AND ADMINISTRATION STUDY DESIGN RESULTS

Turmeric 46
73 mg vs 144 mg standardized extract Partially blinded, randomized, two-dose, Decreased IBS prevalence (P < .001),
tablets  8 wk pilot study (adult) trend to reduced pain (P ¼ .071)
Peppermint47 187 mg (<45 kg) or 374 mg (‡45 kg) vs Randomized, double-blind, placebo- Decreased severity of pain (P < .03)
placebo capsules thrice daily  2 wk controlled (pediatric)
Iberogast48 Liquid extract 20 drops thrice daily vs Multicenter randomized, double-blind, Decreased abdominal pain (P ¼ .0009)
placebo  4 wks placebo-controlled (adult)
Artichoke leaf 320 mg or 640 mg standardized extract Postmarketing surveillance in IBS with Reduction in dyspepsia (P < .001),
extract49 once daily  8 wk concomitant dyspepsia (adult) normalization of bowel pattern
(P < .001)

e12 Pediatrics in Review


functional disability and anxiety (P < .10) but no change in for many children and families, but psychological, phar-
overall gastrointestinal symptoms. (32) A pilot study in the macologic, or dietary treatment may also be needed for
Netherlands of 20 children ages 8 to 18 years showed a some. Strong evidence supports the efficacy of CBT, and
significant decrease in pain frequency and intensity after 10 emerging data suggest that clinical HT and yoga also may
sessions of yoga. (33) A recent study of adolescents and be beneficial. Probiotics and certain herbs may be useful
young adults practicing a yoga protocol for 3 hours per week adjuncts to biobehavioral therapy. Above all, acknowledg-
over 6 weeks demonstrated lower physical disability in the ing that the patient’s symptoms are genuine is most im-
adolescents and decreased IBS symptoms and psychological portant for strengthening the patient-parent-physician
distress in the young adults. (34) The overall evidence for bond, assuring the patient of the physician’s belief in the
yoga efficacy in IBS is rated as Grade B. patient’s pathology, and initiating discussions about the
“brain – gut” connection.
Hypnotherapy
Hypnotherapy (HT) has been used for pain control in
References
medicine since the 19th century and studied for IBS since
1984, with Grade B evidence suggesting that it decreases 1. Hyams JS, Burke G, Davis PM, Rzepski B, Andrulonis PA.
Abdominal pain and irritable bowel syndrome in adolescents: a
stress and somatization. (35) Brain imaging has demon- community-based study. J Pediatr. 1996;129(2):220–226
strated that HT reduced activity in the anterior cingulate 2. Sagawa T, Okamura S, Kakizaki S, Zhang Y, Morita K,
cortex, a region shown to be overreactive in adults with IBS. Mori M. Functional gastrointestinal disorders in adolescents
(36) The 2014 ACG systematic review and meta-analysis of and quality of school life. J Gastroenterol Hepatol. 2013;28
(2):285–290
psychological therapies for IBS pooled data from 5 adult
3. Rasquin A, Di Lorenzo C, Forbes D, et al. Childhood functional
randomized, controlled trials and found substantial benefit
gastrointestinal disorders: child/adolescent. Gastroenterology.
for HT over control therapy with an NNT of 4. (30) Vliegar 2006;130(5):1527–1537
et al randomized 53 patients ages 8 to 18 years to either 4. Zeisler B, Hyams JS. Irritable bowel syndrome. In: Faure C, ed.
standard treatment (education and supportive therapy) or 6 Pediatric Neurogastroenterology. New York, NY: Springer;
2013:367–376
HT sessions consisting of relaxation and suggestions to
5. Varni JW, Lane MM, Burwinkle TM, et al. Health-related
control abdominal pain, gut function, and ego strengthen-
quality of life in pediatric patients with irritable bowel
ing. (37) At both 1 year (P < .001) and 5 years (P ¼ .005), syndrome: a comparative analysis. J Dev Behav Pediatr. 2006;
results with HT were superior to those with standard 27(6):451–458
therapy. (38) Results of 2 other studies of children with 6. Vlieger AM, Blink M, Tromp E, Benninga MA. Use of
functional abdominal pain favored the use of HT for reduc- complementary and alternative medicine by pediatric patients with
functional and organic gastrointestinal diseases: results from a
tion of pain and absenteeism. (39)(40) It is reasonable to multicenter survey. Pediatrics. 2008;122(2):e446–e451
include these 2 studies in this review because pediatric 7. Brenner DM, Moeller MJ, Chey WD, Schoenfeld PS. The utility of
functional abdominal pain and IBS are similar in presenta- probiotics in the treatment of irritable bowel syndrome: a systematic
tion and response to psychological therapies. (41) Although review. Am J Gastroenterol. 2009;104(4):1033–1049, quiz 1050

sample sizes are small, the available literature indicates that 8. Ford AC, Quigley EM, Lacy BE, et al. Efficacy of prebiotics,
probiotics, and synbiotics in irritable bowel syndrome and chronic
HT is more effective than usual treatment in children.
idiopathic constipation: systematic review and meta-analysis. Am J
Limitations include whether results are generalizable to Gastroenterol. 2014;109(10):1547-1562
hypnotherapists outside of a research setting and the 9. Kassinen A, Krogius-Kurikka L, Mäkivuokko H, et al. The
availability/insurance coverage of hypnotherapists. An fecal microbiota of irritable bowel syndrome patients differs
significantly from that of healthy subjects. Gastroenterology.
ongoing pediatric study comparing HT via a CD audio
2007;133(1):24–33
recording listened to in the home with individual therapy
10. van Tilburg M, Squires M, Blois-Martin N, Zucker NL, Bulik C,
should address some of these issues. (42) Chitkara D. Diet and eating associated symptoms in adolescents
with IBS. Gastroenterology. 2012;142(5 suppl 1):S381
11. Carlson MJ, Moore CE, Tsai CM, Shulman RJ, Chumpitazi BP.
CONCLUSION
Child and parent perceived food-induced gastrointestinal
symptoms and quality of life in children with functional
Once a diagnosis of IBS is established, a suggested approach
gastrointestinal disorders. J Acad Nutr Diet. 2014;114(3):403–413
is to help the family understand that coping strategies are
12. Eswaran S, Muir J, Chey WD. Fiber and functional gastrointestinal
important for symptom management because psycholog- disorders. Am J Gastroenterol. 2013;108(5):718–727
ical and environmental factors play a role in the disease. 13. Prior A, Whorwell PJ. Double blind study of ispaghula in irritable
Validation and reassurance may be sufficient treatment bowel syndrome. Gut. 1987;28(11):1510–1513

Vol. 37 No. 4 APRIL 2016 e13


14. Lambert JP, Brunt PW, Mowat NA, et al. The value of prescribed bowel syndrome: systematic review and meta-analysis. Am J
‘high-fibre’ diets for the treatment of the irritable bowel syndrome. Gastroenterol. 2014;109(9):1350–13651366
Eur J Clin Nutr. 1991;45(12):601–609 31. Birdee GS, Yeh GY, Wayne PM, Phillips RS, Davis RB, Gardiner P.
15. Francis CY, Whorwell PJ. Bran and irritable bowel syndrome: time Clinical applications of yoga for the pediatric population: a
for reappraisal. Lancet. 1994;344(8914):39–40 systematic review. Acad Pediatr. 2009;9(4):212-220.e1-9
16. Moayyedi P, Quigley EM, Lacy BE, et al. The effect of fiber 32. Kuttner L, Chambers CT, Hardial J, Israel DM, Jacobson K, Evans K.
supplementation on irritable bowel syndrome: a systematic review A randomized trial of yoga for adolescents with irritable bowel
and meta-analysis. Am J Gastroenterol. 2014;109(9):1367–1374 syndrome. Pain Res Manag. 2006;11(4):217–223
17. Boettcher E, Crowe SE. Dietary proteins and functional 33. Brands MM, Purperhart H, Deckers-Kocken JM. A pilot study of
gastrointestinal disorders. Am J Gastroenterol. 2013;108 yoga treatment in children with functional abdominal pain and
(5):728–736 irritable bowel syndrome. Complement Ther Med. 2011;
18. Gijsbers CF, Kneepkens CM, Schweizer JJ, Benninga MA, Büller 19(3):109–114
HA. Recurrent abdominal pain in 200 children: somatic causes and 34. Evans S, Lung KC, Seidman LC, Sternlieb B, Zeltzer LK, Tsao JC.
diagnostic criteria. Acta Paediatr. 2011;100(11):e208–e214 Iyengar yoga for adolescents and young adults with irritable
19. Biesiekierski JR, Newnham ED, Irving PM, et al. Gluten causes bowel syndrome. J Pediatr Gastroenterol Nutr. 2014;59(2):
gastrointestinal symptoms in subjects without celiac disease: a 244–253
double-blind randomized placebo-controlled trial. Am J 35. Whorwell PJ, Prior A, Faragher EB. Controlled trial of hypnotherapy
Gastroenterol. 2011;106(3):508–515 in the treatment of severe refractory irritable-bowel syndrome.
20. Carroccio A, Mansueto P, Iacono G, Soresi M, D’Alcamo A, Cavataio Lancet. 1984;2(8414):1232–1234
F, et al. Non-celiac wheat sensitivity diagnosed by double-blind 36. Rutten JM, Reitsma JB, Vlieger AM, Benninga MA. Gut-directed
placebo-controlled challenge: exploring a new clinical entity. Am J hypnotherapy for functional abdominal pain or irritable bowel
Gastroenterol. 2012;107(12):1898–1906; quiz 1907 syndrome in children: a systematic review. Arch Dis Child. 2013;
21. Biesiekierski JR, Peters SL, Newnham ED, Rosella O, Muir JG, 98(4):252–257
Gibson PR. No effects of gluten in patients with self-reported non- 37. Vlieger AM, Menko-Frankenhuis C, Wolfkamp SC, Tromp E,
celiac gluten sensitivity after dietary reduction of fermentable, Benninga MA. Hypnotherapy for children with functional
poorly absorbed, short-chain carbohydrates. Gastroenterology. abdominal pain or irritable bowel syndrome: a randomized
2013;145(2):320–328.e1-3 controlled trial. Gastroenterology. 2007;133(5):1430–1436
22. Halmos EP, Power VA, Shepherd SJ, Gibson PR, Muir JG. A diet low 38. Vlieger AM, Rutten JM, Govers AM, Frankenhuis C, Benninga MA.
in FODMAPs reduces symptoms of irritable bowel syndrome. Long-term follow-up of gut-directed hypnotherapy vs. standard care
Gastroenterology. 2014;146(1):67–75.e5 in children with functional abdominal pain or irritable bowel
23. Chumpitazi BP, Tsai CM, McMeans AR, Shulman R. A Low syndrome. Am J Gastroenterol. 2012;107(4):627–631
FODMAPS diet ameliorates symptoms in children with irritable 39. van Tilburg MA, Chitkara DK, Palsson OS, et al. Audio-
bowel syndrome: a double blind, randomized crossover trial. recorded guided imagery treatment reduces functional
Gastroenterology. 2014;146(5):S-144 abdominal pain in children: a pilot study. Pediatrics. 2009;124(5):
24. Atkinson W, Sheldon TA, Shaath N, Whorwell PJ. Food elimination e890–e897
based on IgG antibodies in irritable bowel syndrome: a randomised 40. Weydert JA, Shapiro DE, Acra SA, Monheim CJ, Chambers AS, Ball
controlled trial. Gut. 2004;53(10):1459–1464 TM. Evaluation of guided imagery as treatment for recurrent
25. Guo H, Jiang T, Wang J, Chang Y, Guo H, Zhang W. The value of abdominal pain in children: a randomized controlled trial. BMC
eliminating foods according to food-specific immunoglobulin G Pediatr. 2006;6:29
antibodies in irritable bowel syndrome with diarrhoea. J Int Med 41. Rutten JM, Benninga MA, Vlieger AM. IBS and FAPS in children: a
Res. 2012;40(1):204–210 comparison of psychological and clinical characteristics. J Pediatr
26. Zar S, Mincher L, Benson MJ, Kumar D. Food-specific IgG4 Gastroenterol Nutr. 2014;59(4):493–499
antibody-guided exclusion diet improves symptoms and rectal 42. Rutten JM, Vlieger AM, Frankenhuis C, et al. Gut-directed
compliance in irritable bowel syndrome. Scand J Gastroenterol. hypnotherapy in children with irritable bowel syndrome or
2005;40(7):800–807 functional abdominal pain (syndrome): a randomized
27. Chey WD. The role of food in the functional gastrointestinal controlled trial on self exercises at home using CD versus
disorders: introduction to a manuscript series. Am J Gastroenterol. individual therapy by qualified therapists. BMC Pediatr.
2013;108(5):694–697 2014;14:140
28. Accarino AM, Azpiroz F, Malagelada JR. Modification of small 43. Guandalini S, Magazzù G, Chiaro A, La Balestra V, Di Nardo G,
bowel mechanosensitivity by intestinal fat. Gut. 2001; Gopalan S, et al. VSL#3 improves symptoms in children with
48(5):690–695 irritable bowel syndrome: a multicenter, randomized, placebo-
29. Di Lorenzo C, Colletti RB, Lehmann HP, et al; AAP Subcommittee; controlled, double-blind, crossover study. J Pediatr Gastroenterol
NASPGHAN Committee on Chronic Abdominal Pain. Chronic Nutr. 2010;51(1):24–30
Abdominal Pain In Children: a Technical Report of the American 44. Whorwell PJ, Altringer L, Morel J, Bond Y, Charbonneau D,
Academy of Pediatrics and the North American Society for O’Mahony L, et al. Efficacy of an encapsulated probiotic
Pediatric Gastroenterology, Hepatology and Nutrition. J Pediatr Bifidobacterium infantis 35624 in women with irritable bowel
Gastroenterol Nutr. 2005;40(3):249–261 syndrome. Am J Gastroenterol. 2006;101(7):1581–1590
30. Ford AC, Quigley EM, Lacy BE, et al. Effect of antidepressants 45. Horvath A, Dziechciarz P, Szajewska H. Meta-analysis:
and psychological therapies, including hypnotherapy, in irritable Lactobacillus rhamnosus GG for abdominal pain-related functional

e14 Pediatrics in Review


gastrointestinal disorders in childhood. Aliment Pharmacol Ther. 48. Madisch A, Holtmann G, Plein K, Hotz J. Treatment of irritable
2011;33(12):1302–1310 bowel syndrome with herbal preparations: results of a double-blind,
46. Bundy R, Walker AF, Middleton RW, Booth J. Turmeric extract randomized, placebo-controlled, multi-centre trial. Aliment
may improve irritable bowel syndrome symptomology in Pharmacol Ther. 2004;19(3):271–279
otherwise healthy adults: a pilot study. J Altern Complement Med. 49. Bundy R, Walker AF, Middleton RW, Marakis G, Booth JC.
2004;10(6):1015–1018 Artichoke leaf extract reduces symptoms of irritable bowel
47. Kline RM, Kline JJ, Di Palma J, Barbero GJ. Enteric-coated, pH- syndrome and improves quality of life in otherwise healthy
dependent peppermint oil capsules for the treatment of irritable volunteers suffering from concomitant dyspepsia: a subset analysis.
bowel syndrome in children. J Pediatr. 2001;138(1):125–128 J Altern Complement Med. 2004;10(4):667–669

Parent Resources from the AAP at HealthyChildren.org


• English only: https://www.healthychildren.org/English/health-issues/conditions/abdominal/Pages/Irritable-Bowel-Syndrome-IBS-and-
Inflammatory-Bowel-Disease-IBD.aspx

Vol. 37 No. 4 APRIL 2016 e15


Two Cases of Abdominal Pain after
Trauma
Stephanie Hartman, MD,* Robin Petroze, MD,† Eugene McGahren, MD†
*Department of Pediatrics, University of Virginia, Charlottesville, VA.

Department of Surgery, University of Virginia, Charlottesville, VA.

CASE 1 PRESENTATION

A previously healthy 2-year-old girl presents to the emergency department (ED)


with a 4-day history of 1 to 2 daily episodes of nonbloody, nonbilious (NBNB)
vomiting. She is accompanied by her father, who relates an unwitnessed fall from
a playground slide 4 days ago and a sick contact with recent gastroenteritis. That
night in the ED, she appears well and has normal vital signs. Clinicians attempt an
oral challenge. She has 1 episode of NBNB emesis after eating an ice pop but later
tolerates ice chips and water. Because she is 4 days removed from the reported
trauma, the ED staff does not believe the emesis is due to any injuries. Her
physical examination findings are benign and vital signs are stable, so she is sent
home with the diagnosis of likely viral gastroenteritis rather than postconcussive
syndrome.
Because she has emesis with every meal the next day at home and becomes
more tired, her grandmother brings her back to the ED. The girl is somnolent,
with dry mucous membranes and decreased urine output. Vital signs include
a heart rate of 124 beats per minute, blood pressure of 99/53 mm Hg, and
temperature of 36.6°C (97.9°F). Among the results of serum laboratory tests are
alanine aminotransferase of 478 U/L (7.98 mkat/L), aspartate aminotransferase
of 90 U/L (1.5 mkat/L), hemoglobin of 9.5 g/L (95 g/L), hematocrit of 29.9%
(0.3), and anion gap of 25 mEq/L (25 mmol/L). She is given multiple intravenous
(IV) fluid boluses. Computed tomography (CT) scan is ordered and reveals the
diagnosis.

CASE 1 DISCUSSION

CT scan of her abdomen and pelvis (Fig 1) documents duodenal hematoma.


Duodenal hematomas are caused by compression of the duodenum against
the vertebral column during trauma. This leads to tearing of blood vessels
between the submucosa and the muscularis layers of the intestinal wall.
The resulting hematoma can partially or completely obstruct the bowel
lumen, resulting in emesis, particularly after eating. In addition to trauma,
AUTHOR DISCLOSURE Drs Hartman, Petroze, duodenal hematomas can be a complication of upper endoscopy and
and McGahren have disclosed no financial biopsy.
relationships relevant to this article. This
Duodenal hematomas are usually managed conservatively, only requiring
commentary does not contain a discussion of
an unapproved/investigative use of a surgical management in 6% of cases. Treatment involves placing a nasogastric
commercial product/device. tube (NGT) and making patients nil per os (NPO) with IV nutrition to allow the

e16 Pediatrics in Review


She was made NPO and an NGT placed. She was started
on parenteral nutrition due to the anticipated prolonged
course of being NPO. She had decreased NGT output on
hospital day 3 (7 days after injury). Repeat upper gastroin-
testinal tract study on hospital day 4 showed only partial
obstruction, which prompted initiation of a liquid diet. A
detailed CPS evaluation determined that she was safe to go
home with her father. She was discharged on hospital day 8
on a full liquid diet. She was seen in clinic 1 week later and
advanced to a full diet, which she tolerated well.

Figure 1. Computed tomography scan showing duodenal hematoma CASE 2 PRESENTATION


(black arrow).
A 10-year-old previously healthy boy presents to his pedia-
trician 1 day after a bicycle accident in which he struck his
hematoma to resolve. Affected children resume eating an abdomen against the handlebars. In the first hour after the
average of 16 days after injury. Once the hematoma has accident, he vomited twice, and he refused any oral intake
reduced in size, a feeding tube can be advanced beyond the for the rest of the day and the next morning. He continues
hematoma to allow enteral nutrition distal to the injury. to have abdominal pain and anorexia. He has abdominal
Multiple case series have demonstrated that duodenal pain and tenderness to palpation but otherwise normal
hematomas in children younger than 4 years of age are physical examination results and stable vital signs. His
almost always caused by nonaccidental trauma (NAT). pediatrician recommends continued observation. Two
Gaines et al published an 8-year retrospective review of days later, his father calls the pediatrician due to the
all pediatric trauma admissions to a level I trauma center. boy’s continued anorexia, abdominal pain, nausea, and
Of the 8,968 admissions, 30 (0.3%) had duodenal injury. vomiting. The pediatrician prescribes ondansetron for
Eight children younger than age 4 years with duodenal nausea and acetaminophen with codeine for pain. Due to
injury were victims of NAT. Therefore, such injury in this his continued refusal of food, the father takes the boy to
age group should raise the suspicion of NAT, and Child the ED for a second opinion. The boy’s vital signs upon
Protective Services (CPS) should be involved in determin- presentation to the ED are stable, but he exhibits sig-
ing whether the patient would be returning to a safe living nificant abdominal tenderness on examination. Imaging
situation upon discharge. studies lead to the diagnosis.
In this patient, head CT scan yielded normal results. She
subsequently had an upper gastrointestinal tract study, which
showed complete obstruction of the duodenum (Fig 2). CASE 2 DISCUSSION

CT scan of the abdomen and pelvis reveals duodenal per-


foration (Fig 3). Duodenal perforations are less common
than hematomas and are caused by a “blowout” injury due
to rapid compressive forces. They are always managed
surgically. Surgical management can range from simple
repair, which is effective for most injuries, to more complex
techniques, which are required for severe or complicated
injuries. The most common complications after surgical
repair include pancreatitis, pleural effusion, sepsis, rerup-
ture of the duodenal repair, abscess formation, and wound
dehiscence. The typical mechanisms of injury leading to
duodenal perforation are crashes involving motor vehicles,
all-terrain vehicles, and bicycles as well as NAT. The patient
was taken to the operating room for primary repair of
Figure 2. Upper gastrointestinal tract study showing complete
obstruction of the duodenum (white arrow). The black arrow indicates
duodenal perforation and placement of an omental patch
the pylorus of the stomach. and a jejunal tube (J tube) distal to the injury site. He was

Vol. 37 No. 4 APRIL 2016 e17


injury, large forces are required to cause damage. The ret-
roperitoneal location of the duodenum also means that
patients who have substantial duodenal injury can have a
surprisingly benign abdominal examination because peri-
tonitis is not typically evident. Primary care clinicians need
to be aware that signs of peritonitis are not always present
on physical examination, even with substantial abdominal
injury. Delays in diagnosis and treatment are associated
with increased complication rates and longer hospital stays,
illustrating the need for clinicians to be diligent. The pres-
ence of continued abdominal pain, nausea, vomiting, and
anorexia in pediatric patients multiple days after abdominal
trauma requires further investigation.

Summary
Figure 3. Computed tomography scan showing duodenal perforation
(black arrow) and free air within the retroperitoneum (white arrow) as • Duodenal injuries are uncommon and difficult to diagnose; this
well as leakage of contrast into the abdominal cavity. diagnosis should be considered in a child who has a history of
abdominal trauma and extended nausea/vomiting and anorexia.
made NPO and an NGT placed. On postoperative day • Clinicians should always consider nonaccidental trauma when
(POD) 3, he started trophic feedings through the J tube patients with a duodenal hematoma are younger than age 4
and tolerated them well. These were increased to full years.
J-tube feedings by POD6. An upper gastrointestinal tract • The management of duodenal hematoma is almost always
follow-through procedure on POD7 showed no evidence conservative.
of obstruction or leak in the duodenum. Several days later • The management of duodenal perforation is always surgical.
he was started on a clear liquid diet by mouth, which was
slowly advanced to a full diet that he tolerated well. He
was discharged on POD16.

Suggested Reading
DISCUSSION OF DUODENAL INJURIES Carrillo EH, Richardson JD, Miller FB. Evolution in the management of
duodenal injuries. J Trauma. 1996;40(6):1037–1046
These two cases represent the spectrum of duodenal in-
Clendenon JN, Meyers RL, Nance ML, Scaife ER. Management of
juries that can occur after trauma, ranging from duodenal
duodenal injuries in children. J Pediatr Surg. 2004;39(6):964–968
hematoma to duodenal perforation. Both patients were seen
Degiannis E, Boffard K. Duodenal injuries. Br J Surg. 2000;
by at least one clinician before their diagnosis and were 87(11):1473–1479
not believed to have significant injury at their initial visits. Desai KM, Dorward IG, Minkes RK, Dillon PA. Blunt duodenal injuries
Both cases were somewhat unusual because no additional in children. J Trauma. 2003;54(4):640–645, discussion 645–646
injuries were found within the abdomen. In the case series Gaines BA, Shultz BS, Morrison K, Ford HR. Duodenal injuries in
children: beware of child abuse. J Pediatr Surg. 2004;
by Gaines et al, 25 of 30 patients with duodenal injury had
39(4):600–602
an associated injury, with fractures, head injuries, and
Shilyansky J, Pearl RH, Kreller M, Sena LM, Babyn PS. Diagnosis and
pancreatic injuries being the most common. Because the management of duodenal injuries in children. J Pediatr Surg.
duodenum’s retroperitoneal location offers protection from 1997;32(6):880–886

Parent Resources from the AAP at HealthyChildren.org


• https://www.healthychildren.org/English/health-issues/conditions/abdominal/Pages/Abdominal-Pain-in-Children.aspx
• Spanish: https://www.healthychildren.org/spanish/health-issues/conditions/abdominal/paginas/abdominal-pain-in-children.aspx

e18 Pediatrics in Review

You might also like

pFad - Phonifier reborn

Pfad - The Proxy pFad of © 2024 Garber Painting. All rights reserved.

Note: This service is not intended for secure transactions such as banking, social media, email, or purchasing. Use at your own risk. We assume no liability whatsoever for broken pages.


Alternative Proxies:

Alternative Proxy

pFad Proxy

pFad v3 Proxy

pFad v4 Proxy